Текст
                    Министерство образования Российской Федерации
Южно-Уральский государственный университет
Кафедра прикладной математики
519.1(07)
Э157
А.Ю. Эвнин
ЗАДАЧНИК
ПО ДИСКРЕТНОЙ МАТЕМАТИКЕ
Издание второе, переработанное и дополненное
Челябинск
Издательство ЮУрГУ
2002

УДК 519.1(076.1)4-511.2(076.1)+510.6(076.1) Эвнин А.Ю. Задачник по дискретной математике. 2-е изд., пе- рераб. и доп. - Челябинск: Издательство ЮУрГУ, 2002. - 164 с. Сборник задач соответствует курсу дискретной математики для сту- дентов специальностей ’’Прикладная математика”, ’’Прикладная матема- тика и информатика” и ’’Программное обеспечение вычислительной тех- ники и автоматизированных систем”. Задачник может быть использован также для проведения практикумов по решению олимпиадных задач. Первое издание вышло в 1998 г. Ил. 27, табл. 9, список лит. — 63 назв. Одобрено научно-методическим советом по математике. Рецензенты: д. ф.-м. н. М.М. Кипнис, ЧГПУ, к. ф.-м. н. С.М. Воронин, ЧелГУ. ISBN 5-696-02145-Х ©А.Ю. Эвнин, 2002. ©Издательство ЮУрГУ, 2002.
Оглавление Предисловие................................................5 1 Предварительные сведения 6 1.1. Множества и операции над ними .................... 6 1.2. Высказывания и предикаты ......................... 7 1.3. Метод математической индукции..................... 8 1.4. Правило произведения ............................ 10 2 Элементы теории чисел 13 2.1. Наибольший общий делитель. Простые числа..........13 2.2. Сравнения по модулю.............................. 15 2.3. Китайская теорема об остатках ................... 16 2.4. Теоремы Эйлера, Ферма, Вильсона.................. 17 2.5. Квадратичные вычеты и невычеты................... 19 2.6. Уравнения в целых числах..........................20 2.7. Мультипликативные функции.........................22 3 Начальные понятия общей алгебры 24 4 Элементы математической логики 27 4.1. Формулы и их преобразования. Двойственность.......27 4.2. Полные системы связок.............................29 4.3. Теорема Поста.....................................30 4.4. Нормальные формы..................................31 4.5. Контактные схемы..................................32 4.6. Булева алгебра....................................34 4.7. Аксиоматические теории............................35 4.8. Исчисление высказываний...........................35 4.9. Исчисление предикатов.............................38
4 Оглавление 4.10. Рекурсивные функции..............................42 4.11. Машина Тьюринга..................................46 5 Комбинаторика 50 5.1. Сочетания.........................................50 5.2. Полиномиальная формула. Комбинаторные тождества .... 52 5.3. Формула включения-исключения......................53 5.4. Задача о беспорядках и встречах ..................54 5.5. Числа Фибоначчи...................................55 5.6. Производящие функции..............................58 5.7. Рекуррентные соотношения..........................60 6 Теория Пойа 64 7 Введение в теорию графов 67 7.1. Определения и примеры ............................67 7.2. Гамильтоновы и эйлеровы графы.....................70 7.3. Деревья...........................................72 7.4. Укладки графов....................................75 7.5. Ориентированные графы. Алгоритмы..................77 7.6. Турниры ..........................................79 7.7. Доминирование, независимость, покрытия, паросочетания . . 81 8 Дополнительные задачи 86 8.1. Инвариант и полуинвариант.........................86 8.2. Задачи с целыми числами ..........................89 8.3. Числа Кармайкла...................................92 8.4. Формула обращения Мёбиуса ........................93 8.5. Бинарные операции и отношения.....................96 8.6. Разные комбинаторные задачи.......................97 8.7. Тождества .......................................100 8.8. Две классические задачи..........................103 8.9. Теорема Рамсея ..................................103 8.10. Ожерелья........................................106 8.11. Графы...........................................107 Ответы. Указания. Решения.................................109 Литература...............................................160
5 ПРЕДИСЛОВИЕ По темам задач и по структуре, а также по терминологии и обозначениям данная книга соответствует учебному пособию [9]. Необходимость составления задачника объясняется тем, что в насто- ящее время практически отсутствуют сборники задач, в которых были бы представлены все темы, составляющие современный курс дискретной математики (в первую очередь, математическая логика, комбинаторный анализ и теория графов). Наиболее полно отвечающий учебной програм- ме указанной дисциплины задачник [20] издан тиражом менее 5 тыс. экз. и малодоступен. Кроме того, в нем отсутствует материал, соответствующий первым трем главам, §7.2, §7.7 и большей части последней главы (в т.ч. §8.4, §8.9) настоящего издания. Названия первых семи глав сборника совпадают, в основном, с соответ- ствующими заголовками из [9]. Весьма обширная последняя глава, содер- жащая задачи повышенной трудности и дополнительный теоретический материал, может быть использована для кружковой работы. Во втором издании наибольшей переработке подверглась четвертая глава, при этом существенным образом использованы материалы лекций по математической логике Ю.П. Нестеренко. Основными источниками задач послужили книги [1], [2], [4], [5], [6], [8], [23], [24], [27]. Результаты, сформулированные в задачах 327 и 385, принад- лежат моему учителю и коллеге М.М. Гольденбергу. Некоторые задачи, в том числе 77, 123.6, 126, 140, 290, 315, 325.2, 369, 370, 374, 502, 540, 553, 556, 609, 612, 647, 660, 669, 671, 677, 678, 696, придуманы (или ’’переоткрыты”) составителем сборника. Формулировки ряда задач возникли как (обрат- ный) перевод с языка олимпиад на язык научный. В частности, широ- ко использованы материалы Всероссийской, Соросовской и Путнамовской олимпиад последних лет, а также заданных разделов журналов ’’Квант” и ’’Математика в школе”. Указать (и даже установить) авторство всех за- дач сборника не представляется возможным, но все-таки назову авторов нескольких красивых задач, придуманных совсем недавно и вошедших во второе издание сборника: С.Л. Берлов (задача 731), А.С. Голованов (135), В.М. Гуровиц (514), Д.В. Карпов (12), С.И. Токарев (733). Электронный адрес автора: evnin@prima.susu.ac.ru. Буду благодарен за замечания и советы. А.Ю. Эвнин, 10 апреля 2002 г.
Глава 1 Предварительные сведения 1.1. Множества и операции над ними 1. Какие из следующих утверждений верные? 1) Ф е Ф', 2) фсф- 3) a е {а, Ь}-, 4) а С {а, Ь); 5){а}е{а,Ь}; 6) {а} С {а, 6}; 7) {а, Ь} е {а, Ь, {а, Ь}}; 8) {a, b} С {а, Ь, {а, Ь}}. 2. Пусть А = [0,1] х R, В = R х [0,1]. Найти множества АС\ В, A U В, А\В, В \ А, изобразить их на координатной плоскости. 3. Пусть Ап = [—п; п] (отрезок числовой прямой). Найти: 1) и Ап (к е N)- 2) и Ап- 3) п Ап (к е N); 4) п Ап. п=1 п=1 п=1 п=1 4. Пусть Ап = (0; (интервал числовой прямой). Найти: 1) и Ап (к е N)- 2) и Ап- 3) п Ап (к е N); 4) п Ап. п=1 п=1 п=1 п=1 5. Доказать: А с В <=> А П В = А <==> A U В = В. 6. Доказать дистрибутивные законы: (А п в) и С = (A и С) П (В U С); (A и В) П С = (А п с) и (В П С). 7. ( Законы двойственности де Моргана). Рассматривается се- мейство подмножеств некоторого множества U: для i = 1,... ,п А^ С U. Пусть для любого множества В С U запись В означает дополнение к В: В = U \ В. Доказать, что 1) (П”=1 Aj) = up=1Aj (дополнение к пересечению есть объединение допол- нений) ; 2) (U”=1 Aj) = np=1Aj (дополнение к объединению есть пересечение допол- нений) .
7 1.2. Высказывания и предикаты 8. Определить местность предикатов. 1) х2 — 2ж — 15 = 0; 2) \/х ж2 — 2ж — 15 = 0; 3) Зх х2 — 2х — у = 0. 9. Связать свободную переменную квантором так, чтобы получить ис- тинное высказывание (предметная область — множество действительных чисел). 1) |ж| = —ж; 2) х2 > 0; 3) sin ж ^2; 4) Зж ж2 — 2х — у = 0. 10. Пусть Р(ж, у) — двухместный предикат ’’Окружность ж вписана в треугольник ?/”. Прочитать следующие высказывания, определить значения их истинности, построить отрицания данных высказываний. 1)\/ж \/з/ Р(х,у\, 2)ЭхЧу Р(х,у\, 3) \/ж Зу Р(х,у). 11. Сформулировать отрицания следующих высказываний в утверди- тельной форме (т.е. так, чтобы отрицание не начиналось со слов ”не” или ’’неверно, что”). 1) В любом городе есть район, в каждой школе которого есть класс, все ученики которого учатся без троек. 2) Существует город, в каждом районе которого есть футбольная ко- манда, все игроки которой не старше 18 лет. 3) В каждом городе есть улица, на которой по крайней мере в одном доме все окна выходят на юг. 4) Существует книга, на каждой странице которой есть не менее чем одна строка, в которой буква ”ы” встречается по меньшей мере два раза. 5) В каждом городе хотя бы одна улица застроена только такими до- мами, в которых есть однокомнатные квартиры. 12. Бизнесмен Вася вывесил в своем супермаркете четыре рекламных лозунга: (1) Всё дешёвое невкусно! (2) Всё невкусное дёшево! (3) Всё вкусное недёшево! (4) Не всё вкусное дёшево! Борющийся за экономию коммерческий директор заметил, что два ло- зунга утверждают одно и то же. Какие? Задачи о рыцарях и лжецах Рыцари всегда говорят правду, а лжецы всегда лгут.
8 Глава 1. Предварительные сведения 13. Жители некоторого государства делятся на рыцарей и лжецов. Как-то в комнате собралось 10 жителей этого государства, и каждый из них сказал, обращаясь к остальным: ’’Все вы — лжецы”. Сколько среди этих людей было рыцарей и сколько лжецов? 14. В другой комнате собралось четверо жителей того же государства (А, В, С, D), и между ними произошел такой разговор: А: Все мы рыцари. В: А лжёт. С: По крайней мере двое из нас — лжецы. D: По крайней мере трое из нас — лжецы. Кто лжец, а кто рыцарь? 15. В думе государства рыцарей и лжецов 101 депутат. Каждый из них заявил, что если его выведут из думы, то среди оставшихся лжецы составят большинство. Сколько рыцарей в думе? 16. По кругу сидят рыцари и лжецы. Каждый из них сказал: ’’Все, кро- ме, быть может, меня и тех, кто сидит рядом со мной, — лжецы”. Сколько рыцарей сидит за столом? 17. Вокруг стола расселись рыцари и лжецы. Каждый из них сказал о своем соседе справа, правдив тот или лжив. Известно, что на основании этих заявлений можно однозначно определить, какую долю от присутству- ющих составляют рыцари. Чему она равна? 1.3. Метод математической индукции Простейший вариант метода математической индукции состоит в следую- щем. Рассматривается некоторый предикат А(п), где п — натуральное число. Пусть известно, что 1) [база индукции] высказывание А(1) истинно; 2) [индукционный шаг] для любого k Е N из истинности А(к) следует истинность А(к + 1). Тогда для любого п G N высказывание А(п) истинно. Действительно, поскольку А(1) истинно, а А(1) влечет А(2), выска- зывание А(2) также истинно. А(2), в свою очередь, влечет А(3). Значит, и высказывание А(3) истинно. Продолжая эти рассуждения, можно "до- браться” за конечное число шагов до высказывания А(п), где п — любое
9 наперед заданное натуральное число, и это высказывание оказывается ис- тинным. 18. Доказать тождества: 1) l + 2 + ... + n = ^tl); 2) 12 + 22 + ... + п2 = ^+iX2"+i); 3) I3 + 23 + ... + n3 = (1 + 2 + ... + п)2; 4) J_ + J_ + _| 1_= _JL_ ) 1-2 ' 2 3 ' • • • ' п-(п+1) П+1 19. Методом математической индукции доказать, что п-элементное множество имеет 2” подмножеств. 20. Плоскость поделена на части несколькими а) прямыми; б) окруж- ностями. Доказать, что эти части можно раскрасить в два цвета так, что любые две смежные (то есть имеющие общий участок границы) части бы- ли разного цвета. 21. Доказать, что при п > 3 существует п различных натуральных чисел таких, что их сумма делится на каждое из этих чисел. 22. Выпуклый 2п-угольник с равными сторонами можно разрезать на ромбы. Доказать. 23. Пусть а0 = 2000, ап = ^2000 + у/ап-\. Доказать, что при п > 1 целая часть числа ап есть величина постоянная. В следующих задачах используются более сложные схемы индукции (где другой вид имеют индукционный шаг и база индукции). 24. Пусть х + - — целое число. Доказать, что для любого натурального п число х + также целое. 25. Доказать, что для любого п > 5 квадрат можно разрезать на п квадратов. 26. Доказать, что для любого п > 70 куб можно разрезать на п кубов. 27. Доказать, что многочлен fn(x) = 1 + х + + ... + при нечётном п имеет один действительный корень, а при чётном п не имеет действи- тельных корней. 28. Доказать, что многочлен, принимающий только неотрицательные значения, представим в виде суммы квадратов многочленов.
10 Глава 1. Предварительные сведения 1.4. Правило произведения 29. Из города А в город Б ведут 5 дорог, а из города Б в город В - 7 дорог. Сколько есть различных маршрутов из города А в В через Б? 30. В меню столовой 3 первых, 5 вторых и 3 третьих блюда. Сколькими способами можно выбрать обед из трёх блюд (первое, второе и третье)? 31. Сколько есть двузначных чисел, не содержащих цифр 0, 2, 5? 32. Сколько есть двузначных чисел, не содержащих цифр 1, 3, 6? 33. Номер автомашины состоит из трёх букв латинского алфавита (со- держащего 26 букв) и трёх цифр. Сколько можно составить различных номеров автомашин? 34. У рояля 88 клавиш. Сколькими способами можно извлечь после- довательно 6 звуков? 35. Сколько натуральных делителей имеет число 23 • З4 • 56? 36. Сколько натуральных делителей имеет число 23 • З4 • 45? 37. Сколько есть пятизначных чисел, 1) оканчивающихся двумя семёрками? 2) начинающихся с двух одинаковых цифр? 3) в каждом из которых нет одинаковых цифр? 4) в каждом из которых соседние цифры различны? 5) делящихся на 4 и не содержащих цифр 0, 4, 6, 8? 6) в записи которых есть одинаковые цифры? 7) в записи которых есть хотя бы одна чётная цифра? 38. Сколько есть перестановок цифр 0, 1, 2, ..., 9, в которых 1) цифра 3 занимает третье место, а цифра 5 — пятое? 2) цифра 1 следует непосредственно за цифрой 0? 3) цифра 0 занимает одно из первых трёх мест, а цифра 1 — одно из последних четырёх мест? 4) цифра 0 занимает одно из первых пяти мест, а цифра 1 — одно из первых трёх мест? 5) между цифрами 0 и 1 стоят ровно три цифры? 6) цифра 0 расположена левее цифры 1?
11 7) цифра 1 расположена между цифрами 0 и 2? 8) хотя бы одна из первых трёх цифр делится на 3? 39. Сколькими способами можно рассадить за десятью партами 10 мальчиков и 10 девочек так, чтобы за каждой партой сидели а) мальчик слева, а девочка справа? б) мальчик и девочка? 40. Сколькими способами можно прочитать слово ПАРУС, двигаясь вправо или вниз по каждой из следующих таблиц? п А Р У С А Р У С Р У С У С С п п А П А Р П А Р У П А Р У С 41. Сколькими способами можно расставить на шахматной доске 8 одинаковых ладей так, чтобы никакие две из них не били друг друга? 42. Сколькими способами можно расставить на шахматной доске 8 одинаковых ладей так, чтобы они били все поля? 43. На координатной плоскости рисуются всевозможные несамопере- секающиеся ломаные, все вершины которых имеют целые координаты, а звенья параллельны координатным осям; Ln — число таких ломаных, вы- ходящих из начала кординат и имеющих длину п. Доказать, что 4 • 2n-1 < Ln < 4 • Зп-1. 44. Ключом шифра, называемого ’’решётка”, является трафарет, сде- ланный из квадратного листа клетчатой бумаги размером п х п (п — чётное число). Некоторые из клеток вырезаются с тем, чтобы в получившиеся отверстия на чистый лист бумаги того же размера можно было вписы- вать буквы шифруемого текста. Одна из сторон трафарета помечается; при наложении трафарета на чистый лист бумаги четырьмя возможны- ми способами (помеченной стороной вверх, вправо, вниз, влево) его выре- зы полностью покрывают всю площадь квадрата, причем каждая клетка оказывается под вырезом ровно один раз. Буквы сообщения, имеющего длину п2, последовательно вписываются в вырезы трафарета при каждом из четырёх его возможных положений. После снятия трафарета на листе бумаги оказывается зашифрованное сообщение. Найти число различных ключей шифра для произвольного чётного числа п.
12 Глава 1. Предварительные сведения 45. При каком наибольшем m с помощью п гирь на чашечных весах можно взвешивать грузы в 1,2,3,...,тг? Какими дожны быть при этом массы гирь? Рассмотреть два случая: а) гири могут быть только на одной чашке весов; б) гири могут быть на разных чашках. 46. Каково наибольшее число подмножеств n-элементного множества, любые два из которых имеют непустое пересечение?
Глава 2 Элементы теории чисел Математика — царица наук, арифметика — царица математики. К. Гаусс Господь Бог создал целые числа, все остальное — дело рук человеческих. Л. Кронекер Целые числа составляют костяк дискретной математики. Д. Кнут 2.1. Наибольший общий делитель. Простые числа 47. Является ли число 57599 простым? 48. Доказать, что нечётное число п > 1 является составным тогда и только тогда, когда оно представимо не менее чем двумя способами в виде разности двух квадратов неотрицательных целых чисел. 49. Два соседних натуральных числа взаимно просты. Доказать. 50. Числа тип взаимно просты. Доказать, что числа тп и т + п также взаимно просты. 51. Числа тип взаимно просты. Какие общие делители могут иметь числа т + п и т — п? 52. Числа тип взаимно просты. Доказать, что числа тп и т2 + п2 также взаимно просты. 53. Числа тип взаимно просты. Какие общие делители могут иметь числа т + п и т2 + п2? 54. Найти наибольший общий делитель чисел 1) 321 и 843; 2) 2166 и 6099; 3) 6787 и 7194; 4) 23521 и 75217. 55. Любые два соседние числа Фибоначчи (определение см. §5.5) вза- имно просты. Доказать.
14 Глава 2. Элементы теории чисел 56. Пусть гп — n-значное число 11... 1. Доказать: (rn,rm) = Г(П)ГП). 57. Последовательность (ап) задается соотношениями ао = 0, ап = P(an_i), где Р(ж) — многочлен с целыми коэффициентами. Доказать, что 58. Показать, что простое число р является делителем п! с кратностью jt>i 59. Записать 20! в виде произведения степеней простых чисел. 60. Доказать, что кратность простого числа р в каноническом разло- жении п! не превосходит а) б) 61. Доказать, что у/тА < Пр|п Р^1 (произведение берется по всем про- стым делителям п). 62. Доказать неравенство Пр|пР^ — Показать, что из него следует бесконечность множества простых чисел. 63. Пусть (т, п) — наибольший общий делитель чисел тип, [т, п] - их наименьшее общее кратное. Аналогичный смысл имеют обозначения (т,п,к) и [т, п, к]. Доказать, что имеет место тождество (т,п)[т,п\ = тп. Верно ли, что(т, п, /с)[ш, п, к] = тпк? 64. Пусть р — простое число. Доказать, что для n = 1, 2,... ,р — 1 числа п и р — п взаимно просты. 65. Доказать, что для любого п найдётся п последовательных нату- ральных чисел, каждое из которых — составное. 66. Доказать, что для любого п > 3 между пип! найдётся по крайней мере одно простое число. 67. Доказать, что для любого п > 1 между п и 2п найдётся по крайней мере одно простое число. 68. Доказать, что если 2п > (1 + n)k, то среди чисел 1, 2, 3,..., 2п суще- ствует по крайней мере к + 1 простое число. Показать, что отсюда следует бесконечность множества простых чисел. 69. Доказать, что среди чисел {1, 2,..., п} не менее четверти свободны от квадратов. Получить из этого утверждения доказательство бесконеч- ности множества простых чисел.
15 2.2. Сравнения по модулю 70. Найти остаток от деления 1) 6100 на 7; 2) 6100 на 35; 3) 1О10 + 1О100 + 1О1000 + ... + Ю101° на 7. 71. Доказать, что 1) 30" + 61100 делится на 31; 2) 43101 + 23101 делится на 66; 3) II10 — 1 делится на 100; 4) 7120 — 1 делится на 143. 72. Пусть кип — натуральные числа. Доказать, что кп+2 + (к + l)2n+1 делится на к2 + к + 1. 73. Доказать, что если сумма квадратов двух целых чисел делится на 3, то и каждое из них делится на 3. 74. Доказать, что если сумма квадратов двух целых чисел делится на 7, то и каждое из них делится на 7. 75. Двузначные числа от 19 до 80 выписаны подряд. Доказать, что полученное число 192021... 7980 делится на 1980. 76. Можно ли все двузначные числа от 32 до 86 выписать в некотором порядке одно за другим так, чтобы получилось простое число? 77. Можно ли числа 145, 146, ..., 151 выписать в некотором порядке одно за другим так, чтобы получилось простое число? 78. Пусть п = 2(mod3), п = 3(mod5). Найти остаток от деления п на 15. 79. Существует ли в сутках момент, когда расположенные на общей оси часовая, минутная и секундная стрелки правильно идущих часов об- разуют попарно углы в 120°? 80. Выяснить, при каких к < 11 сумма квадратов к последовательных натуральных чисел может быть квадратом натурального числа. 81. Число 1144664411 представить в виде суммы наименьшего возмож- ного числа четвёртых степеней натуральных чисел.
16 Глава 2. Элементы теории чисел 82. Можно ли множество натуральных чисел разбить на три подмно- жества так, чтобы для любого п 6 N числа п, 2п, Зп принадлежали разным подмножествам? 2.3. Китайская теорема об остатках Теорема, которой посвящен параграф, открыта в Китае в первом (по дру- гим сведениям, в четвертом) веке нашей эры. Она имеет много приложе- ний. Например, она используется в разработанном в 60-х годах двадцатого века алгоритме быстрого преобразования Фурье. См. об этом в статье М. Кельберт. Что такое преобразование Фурье? //Математическое про- свещение. - 2000. Сер. 3, вып. 4. - С.188-202. Мы рассмотрим два доказательства этой теоремы — комбинаторное и конструктивное. См. также лабораторную работу Мб в рамках практикума, разработанного под руководством известного специалиста по теории чисел X. Монтгомери (программы, составляющие Computational Laboratories In Number Theory (CLINT) имеются на кафедре прикладной математики ЮУрГУ). 83. [Китайская теорема об остатках.] Пусть • • -М попарно взаимно простые натуральные числа; целые числа Ti,r2,... ,гп таковы, что Mi 0 < ri < qj. Тогда существует такое целое число х, которое для любого i имеет остаток гг от деления на qi. Доказать. Доказательство. Обозначим через N произведение чисел qp N = щ • <?2 • --qn- Заметим сначала, что если два числа имеют одинаковые наборы остатков при делении на qi,q2,... ,qn, то их разность кратна N. Действительно, если Mi а = 6(mod%), т.е. Mi а — b : qi, то в силу того, что числа q^,q2,... ,qn попарно взаимно просты, а — b делится и на их произведение N. Рассмотрим множество чисел А = {0,1,..., TV — 1}. Разность любых двух (различных) чисел из этого множества по абсолютной величине мень- ше N, поэтому она не кратна N, и, следовательно, любые два числа из А имеют разные наборы остатков при делении на щ, q2, , qn. Подсчитаем теперь, сколько всего может быть различных наборов остатков при деле- нии числа на щ, q2, , qn. При делении на остаток принимает одно из qi значений: 0,1,..., ф — 2 или qi - 1. Согласно правилу произведения всего наборов будет щ • q2 • ... • qn, т.е.
17 как раз TV. Таким образом, для любого набора остатков от деления на qi, q2,.. , qn в множестве А найдётся (и при том ровно одно) число, обла- дающее им. 84. Пусть qi,q2,... ,qn попарно взаимно простые натуральные числа. Для каждого i найти число ij, удовлетворяющее следующим условиям: 1) / г ij = O(mod^); 2) ti = l(mod%). 85. На основе предыдущей задачи получить конструктивное доказа- тельство китайской теоремы об остатках. 86. Найти наименьшее натуральное число, которое даёт при делении на 3 остаток 2, на 4 — остаток 1, на 5 — остаток 2. 87. Пусть qi,q2,... ,qn попарно взаимно простые натуральные числа; целые числа и, г2...., гп таковы, что Уг 0 < < qi. Доказать, что суще- ствует такое целое число ж, что х = ri(mod^i), х 4- 1 = r2(mod<72),... , ж + п - 1 = rn(modgn). 88. Доказать, что для любого п найдётся п последовательных нату- ральных чисел, каждое из которых делится на квадрат простого числа. 2.4. Теоремы Эйлера, Ферма, Вильсона 89. Найти остаток от деления 1) 2100 на 101; 2) З102 на 101; 3) 8900 на 29; 4) 7120 на 143. 90. Доказать, что для любого натурального п число п73 — п37 делится на 10. 91. Доказать, что если п не делится на 17, то либо п8 — 1, либо п8 4-1 делится на 17. 92. Пусть р — нечётное простое число. Доказать, что п^~ при делении на р дает остаток 0, 1 или р — 1. 93. Доказать, что если сумма квадратов двух целых чисел делится на простое число р = 4к 4- 3, то и каждое из них делится на р.
18 Глава 2. Элементы теории чисел 94. Пусть а и b — взаимно простые числа, р — нечётный простой дели- тель числа а2 + Ь2. Доказать, что для некоторого натурального п выпол- няется равенство р = 4п + 1. 95. Пусть а — целое число, р — нечётный простой делитель числа а4 + 1. Доказать, что р имеет вид р = 8n + 1. 96. Пусть а — целое число, р > 3 простой делитель числа а2 + а + 1. Доказать, что р имеет вид р = 6n + 1. 97. Пусть а и b — взаимно простые числа. Доказать, что а^ь) +^(«) = l(modab). 98. Пусть а > 1 — произвольное натуральное число, р > 2 — простое число, взаимно простое с а2 - 1. Положим п = "Тд1 • Доказать, что 1) п — составное число; 2) an-1 = l(modn). Пусть f(x) = аохп + й1Жп-1 + ... + ап — многочлен с целыми коэффи- циентами. Решить сравнение /(ж) = O(modm) (1) означает найти все целые значения переменной х, ему удовлетворяющие. Как известно, если х = a?o(modm), то и f(x) = f (ж0) (mod m). Поэтому в качестве решений уравнения (1) можно рассматривать классы вычетов по модулю т. Будем говорить, что сравнение (1) имеет столько решений, сколько классов вычетов по модулю m ему удовлетворяют. Степень мно- гочлена f(x) называют степенью сравнения (1). 99. Пусть р — простое число. Тогда сравнение У(ж) = aoxn + aiTn-1 + ... + an = O(modp) (2) равносильно сравнению степени не выше р — 1. 100. Если сравнение n-й степени (2) по простому модулю р имеет более п решений, то все коэффициенты многочлена /(ж) кратны р. 101. Пусть п — составное число. Доказать, что (п — 1)! + 1 не делится на п. 102. Пусть п > 4 — составное число. Доказать, что (п — 1)! делится на п. 103. Если р — простое число, то (р — 1)! + 1 делится на р. Замечание. Задачи 101 и 103 показывают, что имеет место следующий
19 критерий простоты числа. Теорема Вильсона. Для того, чтобы число р было простым, необходи- мо и достаточно, чтобы (р — 1)! + 1 делилось на р. Другое доказательство этой теоремы (не использующее малую теорему Ферма) можно найти в [60] (с.23). 104. [Теорема Лейбница.] Число р > 2 является простым тогда и только тогда, когда (р — 2)! — 1 делится на р. Доказать. 105. Если р = 4/с + 1 — простое число, то (^у^)!2 + 1 делится на р. Доказать. 106. Доказать, что если р > 2 — простое число, то 1) I2 • З2 • 52 •... • (р — 2)2 = (—l)2^ (modp), 2) 22 • 42 • 62 • ... • (р — I)2 = (-l)2^ (modp). 2. 5. Квадратичные вычеты и невычеты Пусть р — простое число. Число а, не делящееся на р, называется ква- дратичным вычетом по модулю р, если разрешимо (относительно х) сравнение х2 = a (modp), и квадратичным невычетом в противном случае. 107. Пусть р — нечётное простое число. Доказать, что среди чисел 1, 2,... ,р — 1 квадратичных вычетов ровно половина. Для простого числа р символ Лежандра ( М определяется так: О, если а делится на р, < 1, если a — квадратичный вычет, — 1, если a — квадратичный невычет. 108. [Лемма Лежандра.] Если р — нечётное простое число, то = oft 1^//2(modp). Р/ 109. Доказать, что 110. Найти все простые р > 2, для которых число —1 является ква- дратичным вычетом.
20 Глава 2. Элементы теории чисел 111. Решить сравнение х2 = 5 (mod 19). 112. [Лемма Гаусса.] Пусть р — нечётное простое число, q = (р — 1)/2, Р = {1,2,и для каждого k Е Р число е^ € { —1,1} выбрано так, что число akEk сравнимо по модулю р с каким-нибудь числом из Р. Тогда 113. Найти все простые р > 2, для которых число 5 является квадра- тичным вычетом. 2.6. Уравнения в целых числах 114. Решить в целых числах уравнения: 1) 16ж + 4у = 1830; 2) 13ж + 7?/= 1; 3) 21ж + 19т/ = 5; 4) 1994ж - 171?/= 1. 115. Пусть а и b — взаимно простые натуральные числа. Найти наи- меньшее натуральное п такое, что при любом натуральном т, большем п, разрешимо в натуральных числах уравнение ах + by = т. 116. Пусть а и b — взаимно простые натуральные числа. Сколько су- ществует натуральных чисел, не представимых в виде п = ах + by, где х и у — неотрицательные целые числа? 117. Один фермер потратил 1000 долларов на покупку 100 различных домашних животных. Каждая корова обошлась ему в 100 долларов, сви- нья — в 30 долларов, а овца — в 5 долларов. Сколько голов скота каждого вида он купил? 118. Другой фермер потратил 1800 долларов на покупку 100 различ- ных домашних животных. Каждая корова стоила 72 доллара, свинья — 36 долларов, овца — 6 долларов. Известно, что коров было куплено больше, чем свиней. Сколько голов скота каждого вида купил фермер? 119. Три мужа — Андрей, Иван и Степан пошли со своими жёнами - Анной, Екатериной и Ольгой за покупками. Каждый платил за каждую вещь по стольку рублей, сколько он купил вещей. Андрей купил больше Анны на 23 вещи, Иван — больше Екатерины на 11 вещей. Определить, кто
21 на ком женат, если каждый из мужей израсходовал на 63 рубля больше своей жены. (Задача экзамена Кишиневской гимназии за 1879 г.). 120. Пусть pi,P2, -,Рп ~ различные нечётные простые числа. Дока- зать, что их произведение можно представить в виде разности квадратов 2”-1 способами. 121. Доказать, что следующие уравнения не имеют решений в целых числах, выбрав подходящим образом модуль для сравнения. 1) у2 = 5ж2 + 6; 2) х2 + у2 = 4z — 1; 3) 15ж2 — 7у2 = 9; 4) х2 — 7у = 10; 5) х3 + 21г/2 + 5 = 0; 6) х2 + у2 + z2 = 8t — 1. 122. Решить в целых числах уравнения 1) х + у = х2 — ху + у2'. 2) у2х = 9999г/ + ж; 3) 2ху — Ьх + у = 55. 123. Решить в натуральных числах уравнения: 1) х2-у2 = 1988; 2) Зт + 7 = 2П; 3) 3 • 2т + 1 = п2- 4) 19х + 94?7 = 1993*; 5) 2Х + 3 = 1Р; 6) 2х + Зу = 52; 7) хУ+1 - (х + 1)^ = 2001; 8) 1 + 3* = 3^ + 52. 124. Решить в натуральных числах уравнение 1 1 1 7Г arctg —h arctg —h arctg - = —, x у z 4 считая, что x > у > z. 125. Найти натуральные числа х < у < г, для которых 1 1 1 _ 19 х ху xyz 97 126. Решить в целых числах уравнение х2 + у2 = 31 (г2 + i2). 127. Найти все пары целых чисел таких что сумма их суммы, разности, произведения и частного равна 150. 128. Даша гадает на ромашке: ’’Любит — не любит — плюнет — поце- лует — к сердцу прижмёт — к черту пошлёт”. Глаша при гадании к этим шести вариантам добавляет ещё один: ’’своей назовёт”. На ромашках с п и 2п лепестками у Даши хорошее предсказание, а у Глаши плохое. Чему равно п, если считать, что на ромашке не может быть более 100 лепестков?
22 Глава 2. Элементы теории чисел 129. На плоскости проведено п прямых. Каждая прямая пересекается ровно с 1999 другими. Найти все возможные значения п. 130. На клетчатой бумаге нарисован прямоугольник. Количество его клеток, примыкающих к границе прямоугольника, равно количеству остальных его клеток. Найти размеры прямоугольника. 131. Первоначально во всех клетках таблицы 100 х 100 записаны плю- сы. Допускается операция одновременного изменения знака во всех клет- ках одной строки или одного столбца. Можно ли получить таблицу, в ко- торой а) 2001; б) 1990; в) 2002 минусов? 132. Найти все натуральные п, при которых уравнение пХ1 + ... + п®100 = 1997 разрешимо в целых числах ад,..., адоо- 133. Пусть Ьо, &!, 62, • ~ геометрическая прогрессия, к < пт — взаимно простые числа, Ьо, Ьь и bm — натуральные числа. Доказать, что при любом i < m число bi также натуральное. Верно ли, что все члены прогрессии натуральные числа? 134. Пусть р и г, q и s — две пары взаимно простых чисел. Доказать, что любое положительное рациональное число представимо в виде , где a, b. c.d - натуральные числа. 135. Если бесконечная возрастающая арифметическая прогрессия со- держит точный квадрат и точный куб, то она содержит и 6-ю степень некоторого целого числа. Доказать. Большое количество задач по элементарной теории чисел собрано в [60]. См. также §§8.2, 8.3 настоящего сборника. Ряд задач из других разделов также тесно связаны с теорией чисел. 2.7. Мультипликативные функции 136. Пусть ш(п) — число различных простых делителей п. Доказать, что функция /(и) = c^n'J — мультипликативная. 137. Доказать, что произведение мультипликативных функций муль- типликативно. 138. Найти количество чисел, не превосходящих m и взаимно простых с пт для пт = 25, 60, 250, 1000 000. 139. Сколько существует правильных несократимых дробей со знаме- нателем 288?
23 140. Пусть число а оканчивается на 1, 3, 7 или 9. Доказать, что а400000 оканчивается на 000001. 141. Доказать тождество ср(т)ср(п) = п))<Д[т, п]). 142. При п > 2 число <Дп) чётно. Доказать. 143. Доказать, что сумма всех натуральных чисел, не превосходящих п и взаимно простых с п, равна тир(п)/2. 144. Из равенства <Дщр2 -Ps) = (pi - 1)(р2 - 1)... (ps ~ 1) (pi,p2,... ,ps — различные простые числа) вывести бесконечность множе- ства простых чисел. т(п) есть число (натуральных) делителей числа п, s(n) — их сумма. 145. Доказать: т(п) нечётно <=> п — квадрат. 146. Доказать: s(ri) нечётно <=> п — квадрат или удвоенный ква- драт. 147. Доказать, что произведение всех делителей числа п равно пт^п^2. 148. Каким количеством нулей может оканчиваться произведение всех делителей числа? п 149. Исходя из соотношения 53 | = Inn + 0(1), докажите, что fc=i = n(lnn + 0(1)). k=i Таким образом, на долю каждого их первых п натуральных чисел при- ходится в среднем примерно In п делителей. 150. Доказать неравенство s(n) -I- <Дп) > 2п. 151. Доказать, что при n > 1 справедливо неравенство s(n) < n(lnn + 1). Существует ли такая константа С, что для всех п справедливо s(n) < Сп? Продолжение темы мультипликативных функций — в §8.4.
Глава 3 Начальные понятия общей алгебры 152. Какими свойствами (рефлексивность, антирефлексивность, сим- метричность, антисимметричность, транзитивность) обладают следующие бинарные отношения на множестве действительных чисел? 1) хду <= v 2 2 => яг = у'; 5) хду <= => х2 + х = у2 + у; 2) хду <= => х2 + у2 = 1: ; 6) хду <= => х3 + х = у3 + у; 3) хду <= => ху > 1; 7) хду <= => х-у е Z- 4) хду Ф= => У = И; 8) хду <= => х — у е N. 153. Какие из отношений предыдущей задачи являются отношениями эквивалентности? Для каждого из таких отношений выяснить, что пред- ставляют собой классы эквивалентности и сколько элементов они содер- жат. 154. На множестве учеников класса введем отношение ’’учится лучше”. Будем говорить ’’Ученик А учится лучше ученика В”, если по большин- ству контрольных работ А имел оценки выше, чем В. Обладает ли данное отношение свойством транзитивности? 155. На множестве А введено симметричное и транзитивное отношение д такое, что Уа 36 agb. Доказать, что отношение д рефлексивно. Соглашение. В задачах данного раздела е обозначает нейтральный элемент группы. Пусть А = {«1, а?,... ,an} — конечное множество, на котором опреде- лена бинарная операция *. Таблица из п строк и п столбцов, в которой на пересечении i-й строки и j-го столбца стоит элемент множества А, равный Hi * aj, называется таблицей умножения, или квадратом Кэли. 156. На множестве {1, 2, 3, 4, 6,12} определим две бинарные операции: 1) a * b = (а, Ь) (наибольший общий делитель); 2) a * b = [а, Ь] (наименьшее общее кратное). Составить для этих операций квадраты Кэли.
25 157. Составим матрицу коэффициентов дробно-линейной функции f („Л — dix+bj . ~ ax+di ‘ д __ f (h bi \ — | i I • \ Ci di J Какая матрица будет соответствовать сложной функции 158. На множестве функций {х, —ж} выберем в качестве бинарной операции композицию функций (будем считать, что областью определения всех функций является множество R\{ —1,0,1}). Составить квадрат Кэли для данной операции. Доказать, что рассматри- ваемая алгебраическая структура является группой. 159. На множестве (Q \ 0) х Q введена операция (а, 6) (с, d) = (ас, be + d). Доказать, что данная алгебраическая структура является группой. 160. Доказать, что в квадрате Кэли конечной группы каждый элемент группы встречается в каждой строке (и каждом столбце) ровно один раз. 161. Составить квадрат Кэли для следующих групп: 1) вращений правильного треугольника; 2) вращений квадрата; 3) вращений правильного пятиугольника; 4) симметрий ромба, не являющегося квадратом; 5) симметрий правильного треугольника; 6) симметрий прямоугольника, не являющегося квадратом; 7) симметрий квадрата. 162. Доказать, что группа из задачи 158 изоморфна группе симметрий квадрата. 163. Какие из следующих числовых множеств образуют аддитивные группы? Z, 2Z, N, 2Z + 1, Q+, Q, R \ Q, {-1, 0,1}. 164. Какие из следующих числовых множеств образуют мультиплика- тивные группы? R, R \ {0}, R+, Z, 2Z + 1, Q, Q \ {0}, {1, -1}, {1, 2, R \ Q, {2П | n е Z}. 165. Доказать, что если в группе каждый элемент себе обратен (Vа а * а = е), то группа — абелева.
26 Глава 3. Начальные понятия общей алгебры 166. Найти с точностью до изоморфизма все группы, состоящие не более, чем из 4 элементов. 167. Пусть </? : G —> Н — сюръективное гомоморфное отображение абелевой группы G на группу Н. Доказать, что Н — абелева группа. 168. Пусть < G, • > группа, д Е G. Доказать, что отображение ipg : G —> G, заданное правилом рд(х) = д~4хд, является изоморфизмом. 169. Пусть < G, • > конечная группа. Доказать, что \/gEG 3neN gn = e. Наменьшее n > 0, при котором дп = е, называют порядком элемента д. 170. Доказать, что конечная группа четного порядка обязательно со- держит элемент второго порядка. 171. Пусть группа обладает единственным элементом второго порядка. Доказать, что этот элемент перестановочен с каждым элементом группы. 172. Пусть х и у — элементы (мультипликативной) группы с указан- ными ниже свойствами. Доказать, что ys = е. 1) х2 = е, хух-1 = т/3; s = 8; 2) х2 = е, хух~4 = yn; s = п2 — 1; 3) х3 = е, хух~4 = у4‘, s = 63; 4) xm = е, хух~4 = yk‘, s = кт — 1. 173. Пусть G — группа с единичным элементом е; ф : G —> G функция такая, что ф(д1)ф(д2)Ф(9з) = всякий раз, когда 9192дз = е = hihzha. Доказать, что существует такой элемент a Е G, что функция ^(х) = аф(х) есть гомоморфизм (т.е. Ух,у ф(ху) = (я) V* (?/))• 174. Элемент х / 0 кольца < К, +, • > называется нильпотентным, если хп = 0 для некоторого n Е N. Доказать, что нильпотентность эле- мента х влечет обратимость элемента 1 — х в любом кольце с единицей. 175. Пусть для элементов а, Ь, с кольца с единицей выполняются ра- венства (1 — ab)c = 1 = с(1 — аЬ). Показать, что если d = 1 + bca, то (1 — ba)d = 1 = cZ(l — ba). Таким образом, из обратимости элемента 1 — ab следует обратимость элемента 1 — Ьа. 176. Доказать, что кольцо идемпотентов < К, +, • > (Va Е К а2 = а) коммутативно. 177. Найти все гомоморфизмы поля действительных чисел в себя.
Глава 4 Элементы математической логики 4.1. Формулы и их преобразования. Двойственность 178. По мишени произведено три выстрела. Пусть Л^ есть высказы- вание: ’’Мишень поражена при г-м выстреле”. Что означают следующие высказывания: 1) А± V Л2 V Л3; 2) А^А2^А^ 3) (Mi V Л2)&Л3? 179. По обвинению в ограблении перед судом предстали А, В и С. Установлено следующее: 1) если А не виновен или В виновен, то С виновен; 2) если А не виновен, то и С не виновен. Виновен ли Л? 180. Определить, кто из четырёх подозреваемых участвовал в ограб- лении банка, если известно: 1) если Л участвовал, то и В участвовал; 2) если В участвовал, то или С участвовал, или Л не участвовал; 3) если D не участвовал, то Л участвовал, а С не участвовал; 4) если D участвовал, то и Л участвовал. 181. В санатории на берегу моря отдыхают отец, мать, сын и две до- чери. До завтрака члены семьи часто купаются в море. Известно, что 1) если купается отец, то обязательно купаются мать и сын; 2) если купается сын, то обязательно купается старшая дочь; 3) мать и младшая дочь порознь не купаются; 4) кто-то из мужчин обязательно купается. Однажды утром из дочерей купалась только одна. Кто купался в это утро? 182. Выяснить, является ли следующее рассуждение логически пра- вильным; для этого представить каждое предложение пропозициональ-
28 Глава 4. Элементы математической логики ной формой и проверить, является ли заключение логическим следствием конъюнкции посылок. Если инвестиции останутся постоянными, то возрастут правительственные расходы или увеличится безработица. Если правительственные расходы не возрастут, то налоги будут снижены. Если налоги будут снижены и инвестиции останутся постоянными, то безработица не увеличится. Следовательно, правительственные расходы возрастут. 183. В городе А живут люди, всегда говорящие правду. Жители города В, напротив, всегда лгут. У развилки двух дорог, ведущих в Л и В, путе- шественник встречает местного жителя. Какой вопрос, требующий ответа ”да” или ’’нет”, должен задать путешественник, чтобы узнать, какая дорога ведёт в А? 184. Является ли тождественно истинной формула (х -> у) -Д ((гс V z} -Д (у V г))? 185. Равносильны ли формулы f и д? 1) / = (ж V?/Vz)-> ((я V?/)(;r V^)), д = х ~ г; 2) / = (х^у) z, д = х^ (у z)- 3) f = (ж -> у) V ((ж -> z)y), g = xy(y^xz). 186. Проверить, имеют ли место следующие равносильности: 1) х V (у ~ z) = (ж V у) ~ (ж V г); 2) х -Д (у ~ г) = (ж -> у) ~ (ж -> z); 3) х(у ~ z) = ху ~ XZ', 4) х -Д (у V z) = (ж -> у) V (ж -> z); 5) ж -> г/z = (ж -> ?/)(ж -> z); 6) ж ->(?/-> z) = (ж ->?/)-> (ж -> z); 7) z -> (ху -Д ((х Ду)д y)z) =у -Д (х -4- z). Говорят, что логическая функция f сохраняет ноль (единицу), если /(О,..., 0) =0 (соответственно /(1,..., 1) = 1). 187. Подсчитать число логических функций от п переменных, сохра- няющих ноль. Назовем логическую функцию самодвойственной, если она является двойственной к самой себе, т.е. f(x±,..., хп) — самодвойственная функция, если f (xi,..., хп) = .. ,х^). 188. Найти число самодвойственных логических функций от п пере- менных (допуская и фиктивное вхождение переменных).
29 189. Найти все самодвойственные логические функции от двух и трёх переменных. 190. Пусть g1(xl,...,xn),...,gm(xi,...,xn) самодвойственные функции. Доказать самодвойственность сложной функции А(ж1,... ,хп) = f(gi,.. .,дт){хъ ... ,Хп). 4.2. Полные системы связок 191. Доказать, что система связок {&, V, —>} не является полной. 192. Выразить через стрелку Пирса 4- (Л В = А V В) операцию отрицания, конъюнкцию и дизъюнкцию. Многочлены Жегалкина Введем операцию сложения по модулю 2: х + у = х ~ у. 193. Докажите, что {+,&, I}1 — полная система связок. Назовем одночленом конъюнкцию любого числа попарно различных переменных или константу 1 (пустую конъюнкцию), а многочленом Же- галкина сумму по модулю 2 попарно различных одночленов ЖцЖгг • • • 1 где суммирование ведётся по наборам («1,..., г*), среди которых может быть и пустой. Пустую сумму (число одночленов равно нулю), как обычно, полагаем равной нулю. 194. Выписать все одночлены от двух переменных (ж и у) и подсчитать число многочленов от двух переменных. 195. Представить многочленом Жегалкина следующие логические функции: 1) х V у V г; 2) ху V xz V yz\ 3) xyz V xyz V xyz V ж у z. 196. Доказать, что любая логическая функция однозначно представи- ма многочленом Жегалкина. 1 Константу 1 можно считать 0-арной связкой, задающей функцию, которая не имеет аргументов.
30 Глава 4. Элементы математической логики 4.3. Теорема Поста Всякое множество Т логических функций, замкнутое относительно супер- позиции (т.е. такое, что любая суперпозиция функций из Т входит в Т), называется функционально замкнутым классом. 197. Выяснить, какие из указанных ниже множеств являются функ- ционально замкнутыми классами: 1) множество функций от одной переменной; 2) множество функций от двух переменных; 3) множество L линейных функций (линейная функция — это многочлен Жегалкина степени 1); 4) множество S самодвойственных функций; 5) множество Ро функций, сохраняющих ноль; 6) множество Pi функций, сохраняющих единицу; 7)Р0ПР?; 8) множество А всех логических функций. 198. Доказать, что если функционально замкнутый класс не пуст и не совпадает с А, то дополнение к нему не обладает свойством функциональ- ной замкнутости. Пусть Е = {0,1}. Упорядочим это множество, полагая 0 < 0,0 < 1,1 < 1. На единичном n-мерном кубе Еп введем частичный порядок следующим образом: (xi,..., хп) < (гц,..., уп), если для любого i име- ем Xi < уi. Логическая функция f называется монотонной, если условие (Х!,...,хп) < (У1,..., уп) влечет /(хъ ..., хп) < /(т/i, ...,уп). 199. Выяснить, какие из указанных ниже функций монотонные. Г)х\/у, 2) ху, 3) х—Э у; 4)хд(хду); 5) х у ~ (ж V у); 6) ху V уz\! zx\ 7) х + у + ху. 200. Множество М монотонных функций функционально замкнуто. Доказать. Множество логических функций Т называется полным, если любая логическая функция представима суперпозицией функций из Т. Мини- мальное полное множество логических функций (т.е. такое полное множе- ство, что если из него удалить любую функцию, то оно перестанет быть полным), называется базисом. 201. Привести примеры базисов из одной, двух и трёх функций.
31 Функционально замкнутый класс, отличный от ф и А называется предполным, если он не содержится ни в каком функционально замкнутом классе, отличном от себя самого и от А. Известно ([24]), что существует ровно пять предполных классов: Po,Pi,L,M,S. Имеет место следующая теорема: Теорема Поста. Множество логических функций Т полно тогда и только тогда, когда для каждого из классов Pq,Pi,L,M, S в Т есть функция, ему не принадлежащая. 202. С помощью теоремы Поста показать, что множество функций {0,1, х + у + z, ху} является базисом. 203. Из множества логических функций {ху V ху, ху + z, (х + у) ~ z, ху V yz V zx} выделить всевозможные базисы. 204. С помощью теоремы Поста доказать, что базис не может содер- жать более а) пяти; б) четырёх функций. Вопросы, затронутые в настоящем параграфе, обстоятельно изучаются в монографии [24]. 4.4. Нормальные формы 205. С помощью равносильных преобразований привести к ДНФ сле- дующие формулы: 1) (^i V a?2x3)(a;i V ж3); 2) (a?i V a?2^3X4)((^2 V ж4) —> ад^заД V х2х3 V Xi V z4. 206. Привести к СДНФ, КНФ и СКНФ следующие формулы: l)a?i V х2х3; 2)х1х2У х-^ху, 3)ад V V х2х3. 207. С помощью метода Блейка построить сокращённую ДНФ по за- данной ДНФ: 1) ^1^2 V xix3a?4 V х2ж3Х4; 2) xix2x3 V xix2a?4 V а?за?4; 3) a?ix2 V xix3 V xix2x3a?4 V Ж1Ж2ж3а;4. 208. Построить сокращённую ДНФ по заданной КНФ: 1) (ад V х2 V x3)(xi V х2 V a?3)(z2 V ж3); 2) (ад V а?4)(а:2 V х3 V x4)(xi V х2 V ж3); 3) (a?i V х2 V ^3)(^i V х4) (ад V х3 V ж4).
32 Глава 4. Элементы математической логики 209. Построить СДНФ, СКНФ и минимальную ДНФ для функции /(ад, ад, ад) со следующей таблицей истинности. Х1 Х2 хз /(ад, х2, ад) 0 0 0 1 0 0 1 1 0 1 0 1 0 1 1 0 1 0 0 0 1 0 1 1 1 1 0 0 1 1 1 1 210. Построить сокращённую ДНФ и все тупиковые ДНФ для функции f = х у z\/ х yz\/ xyz V xyz V xyz \/ xyz. 211. Привести пример логической функции от п переменных, у кото- рых любая ДНФ и любая КНФ являются совершенными. 212. Пятеро друзей решили записаться в кружок любителей логиче- ских задач: Андрей (А), Виктор (В), Семен (С), Дмитрий (В), Евгений (В). Но староста кружка предложил им выдержать вступительный экза- мен. ”Вы должны приходить к нам по возможности больше вечеров, однако в разных сочетаниях, соблюдая следующие условия: 1) Если А приходит вместе с В, то В должен присутствовать. 2) Если В отсутствует, то В должен быть, а С пусть не приходит. 3) Л и С не могут одновременно ни присутствовать, ни отсутствовать. 4) Если придёт В, то D пусть не приходит. 5) Если В отсутствует, то В должен присутствовать, но это в том слу- чае, если не присутствует С. Если же С присутствует при отсутствии В, то В приходить не должен, a D должен прийти”. Сколько вечеров и в каком составе друзья должны приходить, чтобы выдержать экзамен? 4.5. Контактные схемы 213. Упростить схему до 4-х контактов.
33 214. Упростить схему до 3-х контактов. 215. Упростить схему до 2-х контактов. 216. Упростить схему. У С У < У 5 У 5 - b с ь с С 217. Построить контактную схему, реализующую схему голосования жюри из трёх человек (каждый член жюри голосует ”за”, нажимая свою кнопку, и ’’против”, не нажимая её; лампочка зажигается лишь в том слу- чае, когда большинство членов жюри голосует ”за”). 218. Построить контактную схему, реализующую схему голосования жюри из четырёх человек (предложение принимается, если за него прого- лосовало большинство членов жюри или если голоса разделились поровну, и за предложение подан голос председателя жюри). 219. Спроектировать контактную схему, позволяющую зажигать и ту- шить лампочку с помощью трёх независимых переключателей. Существу- ет ли решение аналогичной задачи для п переключателей?
34 Глава 4. Элементы математической логики 4.6. Булева алгебра Пусть на множестве S = {А,В,С,...} определены две бинарные операции, обозначаемые + и -2, так, что при этом выполняются следующие свойства ("аксиомы булевой алгебры/- 1. Множество S замкнуто относительно операций + и . Для любых А, В, С € S 2. А + В = В + А, А В = В А (коммутативность операций + и ); 3. А + (В + С) = (Л + В) + С, А(ВС) = (АВ)С' (ассоциативность); 4. А + А = А, А А = А (идемпотентность); 5. А(В + С) = АВ + АС, А + ВС = (А + В)(А + С) (дистрибутивность). 6. Существует элемент О € S, называемый нулем, такой, что VAeS А + О = А. 7. Существует элемент I Е S, называемый единицей, такой, что VAeS А1 = А. 8. VA е S ЗА Е S А + А = I, А А = О (существование обратного элемента/ Тогда алгебраическая структура < S;+,- > называется булевой алге- брой. Данная система аксиом не является независимой: некоторые аксиомы являются следствиями других. 220. Вывести аксиому 4 из аксиом 5-8. 221. Используя только аксиомы 5, 7, 8, доказать, что VA А + I = I. 222. Вывести аксиому 6 из аксиом 5, 7, 8. 223. Доказать закон двойственности: если в любом тождестве бу- левой алгебры заменить + на • на +, I на О, О на I, то вновь полу- чится тождество. 224. Вывести аксиому 3 из аксиом 4-8. 2Этот знак иногда будет опускаться.
35 4.7. Аксиоматические теории Формальная аксиоматическая теория определена, если 1) задан алфавит (множество символов); 2) из множества слов (конечных последовательностей символов алфавита) выделено множество формул; 3) из множества формул выделены аксиомы; 4) заданы правила вывода (отношения между формулами). Если g — отношение на множестве формул и (Ai, А2, , Ak, А) Е q, то говорят, что формула А есть непосредственное следствие фор- мул Ai, А2, , Аь, или: формула А непосредственно выводима из гипотез Ai, А2, , Ak- При этом используют такую запись: Ai, А2, , Ak А ' Выводом формальной теории называют последовательность формул, в которой каждая формула — либо аксиома, либо непосредственное след- ствие каких-либо предыдущих формул. Последнюю формулу вывода на- зывают теоремой. Говорят, что формула А выводима из множества формул Г (запись: Г h А), если существует последовательность формул, в которой каждая формула — либо аксиома, либо принадлежит Г, либо непосредственное следствие каких-либо предыдущих формул, и при этом последняя фор- мула последовательности есть А. Если Г = {Ai, А2, ,Ak}, записывают также Ai, Аг,..., Ak Ь А. В случае Г = ф получаем, что А — теорема (запись: h А). 225. Доказать следующие свойства выводимости: I. Г, Ah А II. Если Г h А, то Г, В h А. III. Если Г И А, Г h В и А, В h С, то Г И С. IV. Если Г h А и Г, A h В, то Г h В. 4.8. Исчисление высказываний Алфавит включает в себя логические символы ~ и -д, скобки (и ); пропозициональные переменные х,у,... ,х±,щ,...
36 Глава 4. Элементы математической логики Формулы определяются следующим образом: • любая пропозициональная переменная является формулой; • если А — формула, то А формула; • если А и В — формулы, то (А —> В) — формула. Для сокращения записей внешние пары скобок будем иногда опускать. Аксиомы. Для любых формул А, В и С следующие формулы являются аксиомами: А1. А-д(В-дАу А2. (А -д (В -д ((А -Д В) -д (А -4- С)); АЗ. (В^А)^ ЦВ дА)д В). Правило вывода — modus ponens (m. р.): из формул А и А —> В непосредственным следствием является формула В: А, (А^В) В 226. Доказать, что если Г h (А —> В) и Г h А, то Г h В. 227. Доказать, что Ah (В —> А). 228. Доказать, что h (А —> А). 229. [Теорема дедукции.] Доказать, что Г, А И В тогда и только тогда, когда ГН (А —> В). 230. Получить следующие следствия из теоремы дедукции: 1) если А И В, то И А —> В; 2) [правило транзитивности] А —> В, В -д С h (А -Д С); 3) [правило сечения] А —> (В —> С), В И (А -4 С). 231. Доказать, что 1) И А > А; 2) И А -4- А; 3) И (В -д А) -д (А -д ву 4) И (А -4- В) -д (В -д А); 5) И А-> (А-> В); 6) И А -4- (В -д А^ВУ 7) И (А^В) -д ((А^В)^ВУ
37 232. Доказать, что если Г, А Ь В и Г, А Ь В, то Г Ь 13. 233. Доказать, что если формула А выводима в исчислении высказы- ваний, то она является тождественно истинной в алгебре высказываний. Пусть А = A(zi, а?2,..., хп) — формула исчисления высказываний, со- держащая пропозициональные переменные х^,х2,... ,xn, а а принимает значения 0 и 1. Введём обозначение Ап ( А, если <7 = 1; ( А, если <7 = 0. В частности, для пропозициональной переменной х имеем a ( х, если <7 = 1; ( х, если <7 = 0. 234. Пусть в алгебре высказываний (<7i, <72,..., <7П) — произвольный набор значений переменных (xi, х2, , хп) формулы A(xi, х2,..., хп), а <7 = Л(<71, <72,..., <7П). Доказать, что в исчислении высказываний ,ГЛ1 „СГ2 ,,Л |_ да *^1 5 *//2 5 ' • • 7 *^п * 235. Доказать, что если формула А, содержащая из логических свя- зок лишь — и —>,3 тождественно истинна в алгебре высказываний, то она выводима в исчислении высказываний. Задачи 233 и 235 говорят о полноте исчисления высказываний: множество выводимых формул исчисления высказываний совпадает с множеством тождественно истинных формул алгебры высказываний. Пусть в формальной аксиоматической теории вместе с каждой форму- лой А определена формула А. Теорию называют противоречивой, если найдётся такая формула А, что одновременно выводимыми являются фор- мулы А и А. В противном случае теория — непротиворечивая. 236. Доказать, что исчисление высказываний — непротиворечивая те- ория. Формальную аксиоматическую теорию называют полной в узком смысле, если добавление к списку аксиом любой невыводимой формулы делает теорию противоречивой. 3Напомним, что всякую логическую функцию можно представить в таком виде.
38 Глава 4. Элементы математической логики 237. Доказать, что исчисление высказываний полно в узком смысле. 238. Доказать, что ни одна из аксиом Al, А2 и АЗ не выводима из двух Других. 4.9. Исчисление предикатов Алфавит включает в себя предметные переменные х,у,..., xi,yi,...; предикатные символы P(n\ Q^n\ ..., Д , Q[n\ , п = 0,1, 2,.. .; логические символы ~ и — квантор общности V и квантор существования 3; служебные символы: скобки ( и ) и запятую. Формулы (а также их свободные и связанные переменные) опре- деляются индуктивно следующим образом: 1. Если — предикатный символ, а х^,х^,, xin — предметные переменные, то Р^(х^,х^,... ,xin) — атомарная формула; пред- метные переменные атомарных формул называются свободными. 2. Если А — формула, то А — формула; свободные и связанные пере- менные формулы А остаются такими же для формулы А. 3. Пусть А и В — формулы, и при этом ни одна свободная переменная любой из формул не является связанной переменной другой форму- лы. Тогда (А —> В) — формула, в которой свободные переменные формул А и В остаются свободными, а связанные переменные оста- ются связанными. 4. Пусть А — формула, для которой х — свободная переменная. То- гда \/х А и Зх А — формулы, в которых х — связанная переменная (остальные свободные (связанные) переменные формулы А остают- ся свободными (связанными)). Формулу А называют областью дей- ствия соответствующего квантора. Для упрощения записей верхние индексы у предикатных символов, а так- же некоторые пары скобок будем иногда опускать. Кроме того, определим и другие логические символы: записи А V В, А&В, А ~ В будут обо- значать соответственно А —> В, А В, (В —> А) -д А —> В (именно так
39 выражаются дизъюнкция, конъюнкция и эквиваленция через отрицание и импликацию в алгебре высказываний). 239. Определить, какие из переменных следующей формулы являются свободными, а какие связанными. (Va? Зу P&\x,y,z)) -Д (Va? <?(2)(a?,i)). Аксиомы. Для любых формул А, В и С следующие формулы являются аксиомами: Al. А->(£-> А); А2. (А -д (J3 -д ((А -Д В) -д (А -Д С)); АЗ. (В->А)-> ((В дА)д В); А4. (Дх А(а?)) —> А(у) (здесь формула А(а?) не содержит переменной у); А5. А(у) —> (За; А(а?)) (здесь формула А(у) не содержит переменной х). Правила вывода. 1. Правило modus ponens (m. р.): Л’ . 2. Правило связывания квантором общности: , где формула В не содержит переменной х. 3. Правило связывания квантором существования: (эЛ&Дв’ где фор~ мула В не содержит переменной х. 4. Правило переименования связанной переменной: связанную перемен- ную формулы А можно всюду в этой формуле заменить другой пе- ременной, не являющейся свободной в А. 240. Доказать, что 1) Va? А(а?) h За? А(а?); 2) I- За? \/у А(х, у) —> \/у За? А(х, у). Интерпретация формулы. Равносильные формулы. Интерпре- тация формулы исчисления предикатов состоит в указании множества М (области интерпретации) и соответствия между предикатными символами и предикатами соответствующей местности. При заданной интерпретации считают, что предметные переменные принимают значения из множества М, а логические символы и кванторы имеют обычный смысл. При этом всякая формула становится предикатом от своих свободных переменных (если таковые отсутствуют, то — высказыванием).
40 Глава 4. Элементы математической логики 241. Пусть область интерпретации — множество неотрицательных це- лых чисел, а предикаты и P(x,y,z) означают соответственно x + y = znxy = z. Записать формулу с использованием данных пре- дикатов, истинную тогда и только тогда, когда 1) х = 0; 2) х = 1; 3) х — чётное число; 4) х — нечётное число; 5) х < у; 6) х = у, 7) х < у, 8) у делится на х; 9) z — наибольший общий делитель х и у; 10) х — простое число. 242. Пусть область интерпретации — множество /?(А) всех подмно- жеств множества А, а предикат Р(Х, У) есть X С У. Записать с исполь- зованием данного предиката формулу, означающую, что 1) X и У = Z; 2) X П У = Z; 3) X = ф- 4) X = А; 5) У — дополнение к X. 243. Ввести соответствующие предикаты, и с их помощью записать следующие высказывания: 1) Всякое натуральное число, делящееся на 12, делится на 2, 4 и 6. 2) Жители Швейцарии обязательно владеют французским, или итальян- ским, или немецким языком. 3) Каждый студент группы выполнил по крайней мере одну лабораторную работу. 4) Через любые две различные точки проходит единственная прямая. 5) Если a — корень многочлена с действительными коэффициентами, то сопряженное число a — также корень этого многочлена. 6) Функция, непрерывная на отрезке [0,1], сохраняет на нем знак или при- нимает нулевое значение. 7) Функция f непрерывна, но не является равномерно непрерывной, на интервале (0,1). Пусть формулы А и В имеют одинаковые множества свободных пере- менных и фиксирована некоторая общая для них интерпретация. Форму- лы А и В называются равносильными в данной интерпретации, если они принимают одинаковые значения истинности на любом наборе значе- ний предметных переменных. Формулы А и В называются равносиль- ными на множестве М, если они равносильны в любой интерпретации, для которой множество М является областью интерпретации. Формулы А и В равносильны, если они равносильны в любой интерпретации. 244. Привести пример интерпретации формулы Р(х, у), в которой рав- носильны формулы Р(х, y}SzP{y, z) и Р(х, у)&Р(х, z).
41 245. Привести пример множества М, на котором равносильны форму- лы Var Р(а?) и Эх Р(х). 246. Равносильны ли следующие формулы: 1) \/ж (Р(ж)&<2(ж)) 2) Эх (Р(х) V Q(x)) 3) Va; (Р(а?) V (Да;)) 4) Эх (P(x)&Q(x)) 5) Va; (Р(а;) V Q) 6) Эх (P(x)&Q) 7) Чх (Р(а;) Q) 8) Эх (Р(х) —> Q) 9) Чх (Q —> Р(х)) 10) Эх (Q -> Р(х)) 11) Эх (Р(х) —> Q) и и и и и и и и и и и (Vrc Р(гс))&(\/т/ <?(?/)); (Зге Р(а;)) V (Эу Q(y)); (Чх Р(а;)) V (W <?(?/)); (Зге Р(а;))&(3т/ <?(?/)); (Va; Р(а;)) V Q; (За; P(a;))&Q; (Ух Р(а;)) -> Q; (Зге Р(гс)) -> Q; Q -> (Чх Р(а;)); Q (За; Р(ж)); (За; Р(а;)) -4- Q? Формула называется выполнимой в данной интерпретации, если существует набор значений переменных, на котором она становится истин- ным высказыванием. Формула называется истинной в данной интер- претации, если при любом наборе значений переменных она принимает значение ИСТИНА. Формула называется общезначимой, если она ис- тинна в любой интерпретации. 247. Привести пример интерпретации, в которой истинной является формула Р(а;) V Р(т/). 248. Выполнимы ли в какой-либо интерпретации следующие формулы: 1) Эх Чу Р(х,х)&Р(х,у); 2) Эх Р(ж) Р(у)\_____ 3) Чх Эу Р(х) ~ Р(у\, 4) Эу Чх Р(х) Р(у)^ Теорема о полноте исчисления предикатов ([35]). Множество вы- водимых формул исчисления предикатов совпадает с множеством об- щезначимых формул. 249. Доказать, что исчисление предикатов — непротиворечивая тео- рия. 250. Является ли исчисление предикатов полным в узком смысле?
42 Глава 4. Элементы математической логики 4.10. Рекурсивные функции В этом разделе под множеством натуральных чисел будем понимать мно- жество N = {0,1,2,...}. Функцию f : Nn —> N, где n G N, называют n-местной арифметической функцией. Простейшие функции: 1) О (ж) = 0 (нулевая функция); 2) S(x) = х + 1 (прибавление единицы); 3) I™ (xi,..., xn) = хт (функция проектирования, или выбора аргумента). Операторы. 1. Пусть f — m-местная, a gi,...,gm ~ n-местные арифметические функции, а функция h : Nn -д N задаётся следующим образом: h(xi,..., хп) = /(дфхг,..., xn),..., дт(хг,..., хп)). Тогда говорят, что функция h получена из функций f и д1}..., gm с помо- щью оператора суперпозиции. 2. Пусть f и д — соответственно (п + 2)- и n-местная арифметические функции, а функция h : Nn+1 —> N задаётся следующим образом: h(xi,... ,тп,0) = д(х1,...,хп); h(xi,..., хп, у + 1) = /(Т1,..., хп, у, Гфщ, ..., хп, у)). Тогда говорят, что функция h получена из функций f и д с помощью оператора примитивной рекурсии. 3. Пусть f — (n + 1)-местная арифметическая функция, а функция д : Nn+1 —> N задаётся следующим образом: д(цд,... ,Хп) = у, если при любом t < у f(xi,... ,xn,t) > 0, и /(т1,... ,хп,у) = 0. Тогда говорят, что функция д получена из функции f с помощью оператора минимизации. При этом используют обозначение д(хъ ..., xn) = py[f(x1,..., хп, у) = 0]. Арифметическая функция называется примитивно рекурсивной, если она может быть получена из простейших функций с помощью ко- нечного числа применений операторов суперпозиции и примитивной ре- курсии. 251. Доказать примитивную рекурсивность следующих функций: 1) 0(^1,... ,Хп) = 0;
43 2) /(ж) = х + п, где п = const G N; 3) +(х,у) = х + у; f 0, если х = 0; , х 4) sgo; = | х’ если х > 0’ (сигнум); __ fl, если ж = 0; , х 5) sgz = | о’; если ж > 0’ (антисигнум). 252. Показать, что из примитивно рекурсивных функций с помощью операторов суперпозиции и примитивной рекурсии вновь получаются при- митивно рекурсивные функции. 253. Доказать примитивную рекурсивность следующих функций: , х • f х — а, если х > у- / х 1) х—у = 1 п Д (усеченное вычитание); 2) |ж - у\; 3) \х,у) = х-у; 4) ж®'; 5) ж!; 6) тш(ж,?/); 7) тах(ж, д'). Пусть f — (п + 1)-местная арифметическая функция, а функция д : Nn+i —> N задаётся следующим образом: д(хъ ..., жп, ?/) = ^2 /(жь • • • , хп, г). Тогда говорят, что функция д получена из функции f с помощью опера- тора ограниченного суммирования. 254. Доказать, что оператор ограниченного суммирования не выводит из класса примитивно рекурсивных функций. 255. Доказать примитивную рекурсивность следующих функций: 1) [х/у] (положить, что [ж/0] = ж;); 2) rest (ж, у) — остаток от деления ж на у (положить, что rest (ж, 0) = ж); _ ( 1, если гез1(ж, у) = 0; ’ 1 0, если геэДж, у) Ф 0; 1, если ж — простое число; 0 в противном случае; divk 7/1 = 1 - V, ' \ ,У) | о, если геэДж, у) / 0; ,х р / х _ ( 1, если ж — простое число; ' rW — [ 0 в противном случае; 5) 7г(ж) — количество простых чисел, не превосходящих ж; 6) д(ж) = [д/ж].
44 Глава 4. Элементы математической логики 256. Пусть f — (п+1)-местная, д± и д2 — n-местные примитивно рекур- сивные функции, а функция h : Nn —> N задаётся следующим образом: 92(Х1,—,Хп) Ыдц,... ,жп) = /(ж1,...,жп,г). i=gi(xi,—,xn) Доказать, что функция h — примитивно рекурсивна. Пусть f — (п + 1)-местная арифметическая функция, а функция д : Nn+1 —> N задаётся следующим образом: у д{хъ ..., хп, у) = /(Ж1,..., хп, г). г=0 Тогда говорят, что функция д получена из функции f с помощью опера- тора ограниченного перемножения. 257. Доказать, что оператор ограниченного перемножения не выводит из класса примитивно рекурсивных функций. Арифметическая функция называется частично рекурсивной, если она может быть получена из простейших функций с помощью конечно- го числа применений операторов суперпозиции, примитивной рекурсии и минимизации. Частично рекурсивная функция называется общерекур- сивной, если она всюду определена. Тезис Чёрча. Всякая эффективно вычислимая функция является ча- стично рекурсивной. Тезис Чёрча не является математической теоремой, которую можно доказать (или опровергнуть), поскольку понятие эффективной вычисли- мости носит интуитивный характер. Частичная рекурсивность — одна из возможных формализаций эффективной вычислимости. 258. Доказать частичную рекурсивность функции f(x,y) = | х — у, если х > у; не определена, если х < у. 259. Доказать, что общерекурсивная функция, ограниченная сверху примитивно рекурсивной функцией, является примитивно рекурсивной. 260. Пусть р(п) — n-е простое число (р(0) = 2,р(1) = 3,р(2) = 5,...). Доказать, что р(п) — примитивно рекурсивная функция.
45 Функция Аккермана Пусть /wW =/(/(/(/»)...)). п раз Определим следующую последовательность функций: <то(ж) = х + 1; аДж) = 261. Найти явные выражения для сх1(а;) и «2(ж). 262. Доказать следующие свойства функций оДт): 1) Vi, х ефх) > х', 2) Vi сц(х) — возрастающая функция от ж; 3) Мх сц(х) — возрастающая функция от i; 4) «г (ж) > a'i_1(d'i_1(a;)). 263. Пусть f — примитивно рекурсивная n-местная функция, которая может быть получена из простейших функций применением (в совокуп- ности) менее, чем к операторов суперпозиции и примитивной рекурсии. Доказать, что при всех х±,... ,хп выполняется неравенство /(^1, ...,Хп)< «Дтах^,..., тп)). 264. Доказать, что функция Аккермана А(п) = ап(н) не является примитивно рекурсивной. Рекурсивные и рекурсивно перечислимые множества Характеристической функцией множества А называется функция /а(х) = J 1, если х Е А; [ 0, если х А. Множество А называется примитивно рекурсивным (рекурсивным), если примитивно рекурсивна (общерекурсивна) его характеристическая функция. 265. Доказать, что конечное множество примитивно рекурсивно. 266. Пусть А и В — примитивно рекурсивные множества. Доказать, что множества А П В, A U В, А \ В — также примитивно рекурсивны.
46 Глава 4. Элементы математической логики 267. Пусть f — примитивно рекурсивная функция. Доказать, что мно- жество прообразов любого из её значений А = {ж | /(ж) = «} примитивно рекурсивно. Множество называется рекурсивно перечислимым, если оно пусто или является множеством значений некоторой примитивно рекурсивной функции. 268. Доказать, что всякое примитивно рекурсивное множество рекур- сивно перечислимо. Рекурсивность множества означает существование алгоритма, разре- шающего проблему вхождения произвольного наперед заданного элемента х G N в данное множество. В связи с этим рекурсивные множества также называют разрешимыми. В свою очередь, рекурсивная перечислимость множества говорит о том, что существует алгоритм, в результате работы которого формируется дан- ное множество. Подробно изучает данные понятия теория вычислимых функций[17]. 4.11. Машина Тьюринга Многовековая практика человечества от Евклида до Кнута не встретилась с примером алгоритма, который нельзя было бы записать как программу машины Тьюринга. Н.К. Верещагин, А. Шень [17] Английский математик Алан Тьюринг в 1936 г. с целью формализации по- нятия алгоритма придумал абстрактное вычислительное устройство, на- званное позже в его честь машиной Тьюринга. Пусть имеется лента, разбитая на ячейки, в каждой из которых за- писан один из символов 0 или 1. Имеется также управляющая головка, которая в каждый момент времени находится в одной из ячеек. В каждый момент времени головка находится в одном из внутренних состояний q0, qi,..., qm. Считают, что q-[ — начальное состояние, a qo — конечное. В зависимости от символа, расположенного в данной ячейке, и вну- треннего состояния головка записывает в ячейку некоторый символ и ли- бо передвигается в одну из двух соседних ячеек, либо остается в прежней,
47 переходя при этом в новое состояние (возможно, прежнее). В результа- те работы машина меняется конфигурация ленты (совокупность запи- санных в её ячейках символов) от начальной к конечной, тем самым по некоторому алгоритму осуществляется переработка исходных данных. Дадим теперь формальное описание. Пусть S = {0,1} внешний алфавит; Q — {Qo-> <Zi, , Qm} внутренний алфавит, или множество состоя- ний головки; D = {L,R,C}. Командой называют 6-буквенное слово вида qiSj —> spdqa, где qi,qa е Q, Sj,s0 е S, de D. Машиной Тьюринга называют упорядоченную пару < Q, К >, где Q — внутренний алфавит, а К — программа — множество команд, удо- влетворяющее следующим условиям: 1) оно не содержит команд, начинающихся с символа qo; 2) оно не содержит двух шестерок, начинающихся с двух одинаковых сим- волов. Конфигурацией ленты, или словом Поста называют слово вида W = AqiSjB, где А и В — слова в алфавите S (возможно, пустые). На- чальное слово Поста имеет вид AqiSjB. Заключительное слово Поста имеет вид Aq0SjB. Работа машины Тьюринга состоит в поэтапной переработке слов По- ста (от начального к заключительному) в соответствии с её программой: И/С1) => pjz(2) при этом И/(п+1) = , где W' из W получается по следующему правилу. Пусть W = AqiSjB. Тогда 1) если i = 0, машина останавливается и W — заключительное слово Поста — результат работы машины Тьюринга; 2) если i > 0, но в К нет команды, начинающейся с символов qiSj, то машина сломалась (W' не определено); 3) если i > 0 и в К есть команда qiSj -д Spdqa, то при а) d = R al) W' = AspqaB в случае В ± ф, а2) W' = AspqaQ в случае В = ф, б) d=L 61) W' = ApjaSkSyB в случае А = AiSk, 62) W' = qa0spB в случае А = ф,
48 Глава 4. Элементы математической логики в) d = C W' = AqaspB. Программу, моделирующую работу машины Тьюринга (автор — Максим Абушаев), можно найти в локальной сети кафедры прикладной математики ЮУрГУ Н:/students/Учебные материалы/EVNIN/TIURING. Через ап обозначим п-буквенное слово в алфавите S, все символы ко- торого есть а; при п = 0 будем иметь пустое слово. Пусть f — арифметическая n-местная функция. Говорят, что машина Тьюринга вычисляет функцию f(xi,... ,хп), если любое начальное слово Поста вида щ01Х101Х2.. .0Гп0 она переводит в заключительное слово Если при этом не приписывались нули слева, то будем говорить, что машина Тьюринга правильно вычисляет функцию f(xi,...,xn), или: функция f правильно вычисляется (машиной Тьюринга). 269. Какую функцию f(x) вычисляет машина Тьюринга со следующей программой? щО -д 0Rq2; q<A 0Rq2, 921 -> I-R93; 93О -4- 0Lq6; q3l -Д ILqy q6l -д lLq0; q4l -д 0Rq5', q20 -Д OCqo. 270. Построить машины Тьюринга, правильно вычисляющие следую- щие функции: 1) О(х) = 0; 2) S(x) = х -I- 1; 3) Гфх, у) = ж; 4)/(т)=2т; 5) f(x, у) = х + у; 6) f(x) = х-1. Если машина Тьюринга перерабатывает слово W в слово W, не припи- сывая нули при этом на ленте ни справа, ни слева (т.е. в процессе работы не возникают ситуации а2 и 62), будем использовать запись W|=> W'. 271. Построить машины Тьюринга, следующим образом перерабаты- вающие слова Поста:
49 1) [перенос нуля] 2) [сдвиг вправо] 3) [сдвиг влево] 4) [транспозиция] 5) [удвоение] Z : щООИО^ tfoOl^OO; S+ : 4i001x0h> Ol^oO; S~ : 01^0^ goOl^O; T : 0Fgi0P0h> OPgoOFO; D : ?оО1жО1жОО. Машиной Тьюринга a(M)p называется программа (множество команд), полученная из программы машины М следующим образом: 1) при г А 0 во всех командах qt заменяется на qi+a; 2) q0 заменяется на qp. На множестве машин Тьюринга введем функцию I: если М — машина Тьюринга с множеством состояний Q = {q0, qi, • • •, qm}, то l(M) = т. Композитом MN машин М и N называется машина Тьюринга с мно- жеством состояний {q0,qi,..., qi(M)+i(N)} и программой о(-Л^)цм)+1 Д(м) (^V)o- Введем обозначение Мп следующим образом: М1 = М, а для любого натурального i Мг+1 = МгМ. 272. Пусть машины М и N следующим образом перерабатывают слова Поста. М : А => B'qosaB"-, N : B'qiSaB" => С. Убедитесь в том, что композит MN переводит слово Поста А в слово С. 273. Машина Тьюринга Cn = ((S+T)n~1D(TS~)n~2T)n действует так: Cn : ^Ol^OP2... Ol^O*^ Ol^OF2... OH^oOl^OH2... Ol^OO, где z = + a;2 + • • • + xn + n + 2. Проверьте это. 274. Постройте машину Тьюринга, правильно вычисляющую функцию ... ,хп) = хт. 275. Пусть функции ф(хъ..., жт), дфдд,..., жп),..., дт(хъ ...,хп) пра- вильно вычисляются машинами Тьюринга. Тогда и суперпозиция этих функций f(gi,..., ^m)(rci,... хп) обладает тем же свойством. Доказать. 276. Пусть функция д(х) получена из функции f(x,y) с помощью опе- ратора минимизации д(х) = y/y[f(x, у) = 0]. Доказать, что если функция f правильно вычислима, то и функция д правильно вычисляется машиной Тьюринга. Имеет место следующая теорема[35]. Теорема. Множество частично рекурсивных функций совпадает с множеством функций, которые правильно вычисляются машиной Тью- ринга.
Глава 5 Комбинаторика 5.1. Сочетания 277. Вычислить: С72, С2°0, С* С335, С84, С,13 278. Найти число подмножеств X множества {0,1, 2, 3,4, 5, 6, 7, 8, 9}, обладающих следующими свойствами: 1)|Х|=3; 2) |Х| = 5, 1еХ; 3) |Х|=6, 2^Х; 4) |Х| = 7, {О,1}СХ, 2£Х; 5) множество X состоит из трёх чётных и двух нечётных чисел; 6) |Х| <5. 279. На окружности последовательно отмечены точки Ai,...,Ai2. Сколько существует 1) хорд с концами в отмеченных точках; 2) треугольников с вершинами в отмеченных точках; 3) выпуклых четырёхугольников с вершинами в отмеченных точках; 4) треугольников с вершинами в отмеченных точках, не имеющих общих точек с прямой Л2Л8; 5) треугольников с вершинами в отмеченных точках, имеющих общие точки с прямой Л1Л5? 280. На окружности отмечено п точек. Точки соединяются всевозмож- ными хордами; известно, что никакие три из них не пересекаются в одной точке внутри круга. Найти: 1) число точек пересечения хорд внутри круга; 2) количество частей, на которые хорды делят круг. 281. На прямой I отмечено 8 точек, а на параллельной ей прямой т (7 ф тп) — 11 точек. Сколько существует 1) треугольников с вершинами в отмеченных точках; 2) выпуклых четырёхугольников с вершинами в отмеченных точках?
51 282. п человек в совокупности выписывают к журналов, причем каж- дый выписывает два журнала, каждый журнал выписывают четверо, а каждая пара журналов выписывается только одним человеком. Найти п и к. 283. Две команды играют в волейбол до 4 побед. Сколько существует разных вариантов изменения счёта в игре по партиям? 284. Сколькими способами можно разложить 4 белых и 3 чёрных шара по 6 различным ящикам? 285. Решить предыдущую задачу при дополнительном условии: ни один ящик не должен быть пустым. 286. Сколькими способами можно разложить 20 одинаковых шаров по 5 различным ящикам так, чтобы 1) в каждом ящике оказалось не менее двух шаров; 2) в каждом ящике оказалось не более 5 шаров; 3) оказалось не более двух пустых ящиков? 287. Найти коэффициент при х100 в разложении многочлена (1 + х + х2 + ... + а?100)3. 288. Доказать, что т гп 13 12 «1 «п = 1 »п-1 = 1 42=1*1 = 1*0=1 289. Пусть п — произвольное натуральное число, большее 1. Доказать, что существует бесконечно много натуральных чисел, не представимых в виде суммы 1) п; 2) не более, чем п слагаемых, каждое из которых есть п-я степень натурального числа. 290. Дан квадрат. Каждая его сторона разбита на п равных частей. Че- рез точки деления проведены прямые, параллельные сторонам. Сколько существует 1) прямоугольников, 2) квадратов, ограниченных проведённы- ми линиями? 291. В правлении банка 7 человек. Каково должно быть минимальное число замков от сейфа и как следует распределить ключи между членами правления (каждый член правления может получить ключи от нескольких
52 Глава 5. Комбинаторика прочитать слово ’’абракадабра”, замков), чтобы любое большинство сейф могло открыть, а любое меньшин- ство — не могло? 292. Каким числом способов можно двигаясь вправо или вниз по таблице? 293. На клетчатой бумаге нарисован прямоугольник ABCD, стороны которого лежат на линиях сетки, причём длина отрезка AD в к раз больше длины отрезка АВ (к — натуральное число). Рассматриваются всевозмож- ные пути, проходящие по линиям сетки и кратчайшим образом ведущие из А в С. Доказать, что среди этих путей в к раз больше тех, у которых первое звено лежит на AD, чем тех, у которых первое звено лежит на АВ. 294. Изучите поведение последовательности (щ), где ак = Ск (при фиксированном п), с точки зрения возрастания-убывания. 295. Имеется 12 точек, никакие четыре из которых не лежат в одной плоскости. Доказать, что существует разносторонний треугольник с вер- шинами в этих точках. 296. Имеется карточная колода из 52 карт. Каким числом способов можно раздать по 13 карт четырём игрокам? 5.2. Полиномиальная формула. Комбинаторные тождества 297. Найти коэффициент при хк в разложении многочленов: 1)(х + 2)10, к = 3-, 2) (1 - 2х)7, к = 4; 3)(^-|)8, к = -Ь- 4) (З-у^ — Хд/ж)9, к = 11; 5) [х2 — х + 2)8, к = 7; 6) (i/ж + у/х + лУЁ)6, к = 2.
53 298. С помощью формулы бинома Ньютона п (i+x)” = fc=0 доказать следующие тождества: п 1) Е = 10”; fc=0 п 3) Е^=п2”-!; fc=0 го V с* = 2”+1~1 • °) fc+1 п+1 ’ fc=o In 7) s(-im„)2=(-D fc=0 2) E(-l)n“fe2feC^ = 1; fc=0 4) £(-l)‘-4C* = 0; fc=0 6\ v = л-- 7 fc+i n+l’ fc=0 2n—1 s) £ НЖД = o- fc=0 299. С помощью комбинаторных рассуждений доказать: п п—т+т i) Z^r.2-1; 2) £ к—0 к—т Пусть Е = {0,1}. Назовём n-мерным кубом п-ю декартову степень этого множества Еп (множество всех двоичных последовательностей дли- ны п). Элементы Еп — вершины куба. Множество вершин с фиксиро- ванными значениями п — к координат называют гранью размерности к. 300. Пусть ik — число граней размерности к у n-мерного куба. Вычис- лить ik для всех допустимых значений к и доказать, что io — ii + i2 — • • • -к ( — 1)” 1in-i — 1 — (—!)"• 301. Доказать, что формула (*) имеет место и для n-мерной пирамиды (её задают п+1 вершин, а любые к+1 вершин определяют /с-мерную грань, к = 0,1,..., п). 5.3. Формула включения-исключения 302. На кафедре лингвистики работают 13 человек, причем каждый из них знает хотя бы один иностранный язык. Десять человек знают англий- ский язык, семеро — немецкий, шестеро — французский. Пятеро знают
54 Глава 5. Комбинаторика английский и немецкий, четверо — английский и французский, трое - немецкий и французский. Сколько человек знают 1) все три языка; 2) ровно два языка; 3) только английский язык? 303. В сборнике [53] предлагается для решения следующая задача. В группе из 25 студентов 12 изучают латынь, 10 — греческий и 9 санскрит. Для каждых из двух языков найдётся ровно пять студентов, изучающих оба эти языка. Сколько студентов изучают все три языка? Корректна ли эта задача? 304. 1) Показать, что количество натуральных чисел, делящихся на п и не превосходящих положительного числа х, равно \х/п\. 2) Сколько есть чисел, не превосходящих 10000 и не делящихся ни на 3, ни на 5, ни на 7? 3) Сколько есть четырёхзначных чисел, не делящихся ни на 3, ни на 5, ни на 7? 4) Сколько есть чисел, не превосходящих 10000 и не делящихся ни на одно из чисел 6,10 и 15? 5) Показать, что если п = 30m, то количество натуральных чисел, не превосходящих п и не делящихся ни на одно из чисел 6,10 и 15, равно 22m. 305. Пусть п > 5. Показать, что простых чисел в множестве {п 4- 1, п + 2,..., п 4- 30} не больше восьми. 306. На каждой стороне треугольника АВС отмечено по п точек, раз- бивающих её на п 4-1 равных частей. Рассмотрим всевозможные треуголь- ники с вершинами в отмеченных точках (по одной на каждой стороне). Сколько среди этих треугольников таких, у которых ни одна из сторон не параллельна стороне треугольника АВС? 307. Сколько существует 6-значных номеров (первые цифры могут быть и нулями) с суммой цифр 27? 308. В кошельке лежит по 20 монет достоинством в 1, 2 и 5 рублей. Сколькими способами можно из этих 60 монет выбрать к монет? 5.4. Задача о беспорядках и встречах 309. С помощью рекуррентных соотношений найти число беспорядков Dn для п = 1,..., 8. 310. Доказать, что Dn = [у 4- |].
55 311. Сколькими способами можно расставить на шахматной доске 8 одинаковых ладей так, чтобы никакие две из них не били друг друга и чтобы ни одна ладья не стояла на главной диагонали? 312. Сколькими способами можно раскрасить клетки шахматной дос- ки 8 х 8 в 8 цветов так, чтобы клетки, имеющие общую сторону, были бы окрашены в разные цвета и чтобы в каждом горизонтальном ряду встре- чались все 8 цветов? 313. Две колоды карт, содержащие по 52 карты, тщательно тасуются, после чего сравниваются карта за картой. Какова вероятность того, что не будет ни одной пары совпадающих карт? 314. Для числа перестановок п элементов с к встречами Dnk доказать тождества: п 1) Dn>k = nl; 2) Dn k = (к = 1,..., п); к=0 3) £ kDn>k = п\- 4) £ CknDk = п\- к=\ к=0 5) Ё С“Д.л = д; 6) Ё (к - 1)2р„л = nl. к=т к=0 315. Случайным образом выбирается перестановка чисел Пусть £ — количество элементов, остающихся на своих местах. Найти ма- тематическое ожидание и дисперсию случайной величины 316. Секретарше нужно отправить п различных писем по п различным адресам. Она подписывает конверты и случайным образом вкладывает письма в конверты. Сколько в среднем писем дойдет до своего адресата? 5.5. Числа Фибоначчи Последовательность чисел Фибоначчи (Jn) задается соотношениями: fo = 0, /1 = 1, Vn Е No /п+2 = /п+1 + /п- 317. Прыгун перемещается слева направо вдоль клетчатой ленты, со- вершая прыжки на одну или две клетки. Каким числом способов он может переместиться из 1-й клетки в п-ю? 318. Доказать следующие утверждения: 1) /1 + /з + • • • + /гп+1 = /гп+2! 2) 1 + h + /4 + • • • + Лп = Лп+х;
56 Глава 5. Комбинаторика 3) fi + h + • • • + fn = fn+2 — 1; 4) fkfk+1 — fk—lfk = /fc! 5) fl + /2 + • • • + fn = fnfn+Г, 6) 1 — /5 + 7/7 — тД + . . . + (—l)”j—^—7- = A- / 1'2 2-3 3-5 v ' Jn-lJn Jt 7) fn+m — fn—lfm 4” fnfm+li 8) hn = ff+i - fLr, 9) fn + fn+i = fon+Г 10) /3+1 + f3n - /Li = b; 11) если n делится на m, то fn делится на fm; 12) для наибольшего общего делителя двух чисел Фибоначчи справед- ливо соотношение (/„, /т) = f(n<my, 13) если m > 2 и fn делится на fm, то п делится на тп. 319. Найти все п, для которых fn = п2. 320. Доказать тождество Е)/Г=о ^n-k ~ fn+i- 321. Вычислить трехдиагональный определитель размера п х п 1-1 0 0 ... О О О 1 1-1 0 ... О О О О 1 1 -1 ... О О О О 0 0 0 ... 1 1 -1 О 0 0 0 ... О 1 1 322. Доказать, что f = ( ^n+i . \ 1 0 / \ Jn Jn-l / 323. Доказать тождество [Кассини, 1680 г.] fn+lfn-l - fn = (-!)”• 324. Продолжив последовательность чисел Фибоначчи влево (сохраняя рекуррентное соотношение), докажите, что = (—1)”+1/п- 325. Доказать тождества 1) fn+k ' fm—k fn ' fm — ( 1) ' fm—n—k ' fki 2) 2/2 = fn-2 ' fn+2 + fn-1 fn+Г, fn+l fn+2- fnfn+3 = (-!)"•
57 326. Доказать для чётного п + arctg Вывести отсюда равенство ОО 1 V arctg ------ „-1 hn+l 7Г 4‘ 327. Доказать для нечётного п In /та + 1\ fn-lj = 1П /п+1 + 1\ /п +1- V . , / fn+2 + 1 \ +ln(j7Tl) Вывести отсюда равенство fan + 1 /2п-1 = 3. 328. Найти число двоичных последовательностей длины п, не содер- жащих единиц ни в каких двух соседних позициях. 329. Доказать, что из р нулей и q единиц можно составить С?+1 различ- ных последовательностей, в каждой из которых нигде не окажется рядом две единицы. 330. С помощью двух предыдущих задач дать комбинаторное доказа- тельство тождества задачи 320. 331. Найти число различных покрытий прямоугольника 2 х п прямо- угольниками 1x2. 332. Требуется составить набор из 10 гирек (каждая гирька весит целое число граммов), с помощью которых можно взвесить любой вес, выража- ющийся целым числом от 1 до 55, даже если любая из гирек может быть потеряна. Предполагается, что гирьки кладутся на одну чашку весов, а взвешиваемый груз — на другую. 333. Имеется а) 76, б) 199 карточек, на которых написаны различные числа. Карточки разложены на столе по окружности числом вниз. Требу- ется найти какие-нибудь три идущие подряд карточки такие, что число, написанное на средней из этих трех карточек, больше, чем на каждой из
58 Глава 5. Комбинаторика двух соседних. Перевернуть одну за другой можно не более а) 10, б) 12 карточек. Как надо действовать, чтобы наверняка добиться успеха? 334. Доказать, что ^2X1 < 4. 335. Доказать, что 2nfn = 2 Cnm+15m. 336. Пусть р — простое число, большее 5. Доказать, что 1) 5^ = /Дшобр); 2) fp = l(modp). 337. Доказать, что если р — простое число, большее 5, то ровно одно из чисел fp-i или fp+1 делится на р. 338. Можно ли из последовательности чисел, обратных натуральным, выбрать а) сколь угодно длинную; б) бесконечную подпоследовательность, в которой каждое число равно разности двух предшествующих? 339. Найти все функции /(гг), непрерывные на R и для которых имеет место тождество /(/(х)) = /(ж) + х. 340. Назовем множество эгоцентричным (или э-множеством), если оно содержит свою мощность (число элементов). (Например, {2,3}, {3,5,8} — э-множества, а {3,5}, {2,5,8} не являются таковыми). Найти число подмножеств множества {1,2, являющихся минимальными э-множествами (или мэ-множествами), т.е. такими эгоцентричными множествами, чьи собственные подмножества — не э-множества. (Пример. {2,3} — мэ-множество в отличие от {1,2}). С числами Фибоначчи тесно связаны числа Лукаса, о которых см. конец §5.7. 5.6. Производящие функции Производящая функция является устройством, отчасти напоминаю- щим мешок. Вместо того, чтобы нести отдельно много предметов, что могло бы оказаться затруднительным, мы собираем их вместе, и тогда нам нужно нести лишь один предмет — мешок. Д. Пойа Найти производящие функции следующих последовательностей: 341 а = / 1’ п = п 1 0, п> N.
59 342. ап 343. ап ( п + 1, п = 0,1,..., N, [ О, п > N. ( (п + 1)(п + 2), п = 0,1,..., N - 1, 1 О, п > N. 344. ап = а", п = 0,1, 2,... 345. ап = п2, п = 0,1, 2,... Пусть (а„), (Ь„) — последовательности, А(х) и В(а?) — соответствующие производящие функции. Выразить А(х) через В(х) при следующих соот- ношениях между последовательностями: 346. По — 0, — Ьг/_\. п — 1,2,... 347. ап — bn+i. 348. ап = Ьп+к, к 6 N. 349. ап = апЬп. 350. По — 0, ап — bn bn_\j п — 1,2,... 351. ап — bn_\-i bn. 352. ап = £^=0 353. ап = bk. 354. ап = nbn. 355. ап = Ьксп_к (С(х) = спхп). 356. ап = Y^=nCkbk. Доказать, что Ьп = ^=Л-^п~кСкак- 357. Пусть ап = Y^=oCn+r п е No; bo = 0, bn = Y^j=o cn+j\ n 6 N- Доказать, что 1) (^n+l — й'п 4“ bn-\-i, ^n+1 — й'п + Ьщ 2) A(a;) — 1 = xA(x) + B(x), B(x) = x(A(x) + B(x)). 3) Найти Л(а?) и B(x). 4) Получить явные формулы для ап и Ьп. 358. Показать, что функция (1 — 4х)-1/2 является производящей для последовательности ап = С%п. 359. Доказать тождество = •
60 Глава 5. Комбинаторика 5.7. Рекуррентные соотношения 360. Последовательность (а„) удовлетворяет соотношению Ни-|-2 — CV^n+1 4~ уравнение х2 — ах — /3 = 0 имеет два различных ненулевых корня Xi и х2. Доказать, что имеет место тождество ап = щж” + с2а?2 для некоторых щ и с2, однозначно определяемых аг и а2. 361. Найти формулу общего члена последовательности: 1)®п+2 — Зни, Hi — 10, н2 — 16, 2)ап+2 = 2 cos аап+[ — ап; ср = cos а, н2 = cos 2а. 362. Найти количество n-значных чисел, состоящих из цифр 1, 2, 3, в которых первая и последняя, а также любые две соседние цифры различ- ны. 363. Сколько существует раскрасок вершин n-угольника, если соседние вершины должны быть разного цвета, а всего имеется k цветов? 364. Пусть п-и член последовательности задается формулой ап = С13?” 4- с2х%, где xY / х2. Доказать, что для последовательности имеет место рекуррентное соотно- шение ап+2 = 4- /Зап, где а = Xi -h х2, /3 = — Ж1Ж2. 365. Найти нж4 4- by4, если а + b =1, а + b = 1, а 4- b = 23, ах + ЬУ = 2Н + Ьу2 = 2, ах 4- by = 2> зн ах 4- by = 79, 2 ах ах2 4- by2 = 5, 3) ах2 4- by2 = 217, ах3 + Ьу3 = 3; ах3 4- by3 = 14; ах3 4- by3 = 691. 366. Решить задачу 24 с помощью задачи 364. 367. Доказать, что для любого натурального п число
61 является целым и нечётным. 368. Найти первые 100 знаков после запятой у числа (5 + -\/26)101. 369. Доказать, что числа -*-= ((1 + д/2)" - (1 - v^)99} и -^= ((14- л/2)100 - (1 - д/2)100>) 2у2 ' / 2у2 ' / являются целыми и взаимно простыми числами. 370. Доказать, что целая часть числа ^^(1 + л/2)1000 является чётным числом. 371. Доказать, что число [(3 + \/5)n] + 1 делится на 2П. 372. Пусть =4, &2 = 14, &п+2 = 46п+1 — Ьп. Обозначим через Sn площадь треугольника со сторонами bn — 1, bn, bn 4- 1, а через гп радиус вписанной в него окружности. Получить рекуррентные формулы для Sn и гп. 373. Доказать, что для любого чётного п число tgn 15° + ctgn 15° пред- ставимо в виде суммы квадратов трех последовательных целых чисел. 374. Пусть к 6 N, к > 1. Доказать периодичность функции, для кото- рой при любом х + 1) + /(ж - 1) = 2 cos гь 375. Найти число двоичных последовательностей длины 11, не содер- жащих единиц ни в каких трех соседних позициях. 376. Найти общие решения рекуррентных соотношений: 1) ^п+2 4“ 2нп+1 4- ап — 0, 2) ип+3 + 10нп+2 + 32ап+1 4- 32ап = 0; 3) Щц-з + Зап+2 + Зап+1 4- ап = 0. 377. Найти ап по рекуррентным соотношениям и начальным условиям: 1) ®п+з — Зап+2 4- an+i ~ Зап = 0, щ = 3, а2 = 7, аз = 27; 2) ^п+з 3fln-|-i -|- 2ап — 0, а। — <z, п2 — ®з — с. 378. Пусть Pi(x) = (1 + х + х2 4- ... + ж9)г = Y^k=o ai,k%k- 1) Доказать, что = а^-к, где 0 < к < 9г. 2) Доказать, ЧТО Н2г,9г = Y^k=oalk- 3) С помощью тождества Д+1(а;) = (1 4- х 4- х2 4-... 4- а?9) • Д(х) выразить через ^г,1? • • • ? ^i,k- 4) Вычислить (26,27-
62 Глава 5. Комбинаторика 379. Найти коэффициент при х2 в многочлене Рп(х) = (... ((х — 2)2 — 2)2 — ... — 2)2 (здесь п пар скобок). 380. Последовательность (ап) задана соотношениями Hq = О, щ = 1 и Vft 6 N CZ71+I — Пп—\ И- "\/12(2п((2п + 1) + 1. Доказать, что все члены этой последовательности — целые числа. 381. Пусть р / 0,1/2,1, q = 1 — р. Доказать, что [п/2] г=0 _qn-д p-q 382. Чёрт предложил карточному шулеру: ’’Каждый раз, как ты пе- рейдешь мост и вернешься обратно, твои деньги в кармане удвоятся, но за это ты мне заплатишь в первый раз 50 руб, во второй — 100 руб, в третий - 150 руб и т.д.” Шулер прошел по мосту 17 раз вперед и назад. Сна- чала он значительно обогащался, однако затем не только спустил то, что приобрел, но даже остался должен чёрту 360 руб 72 коп. Сколько денег у шулера было изначально? Как бы изменилась ситуация, если бы этих денег было на копейку больше? (Журнал ’’Математический вестник”, 1914 г.) 383. Бесконечная последовательность нулей и единиц 01101001100101... строится так: первая цифра — нуль; следующий шаг повторяется бесконечное число раз: к уже построенному куску последовательности приписывается новый кусок той же длины, получаемый из него заменой нулей на единицы, а единиц на нули. Является ли эта последовательность, начиная с некоторого места, периодической? Числа Лукаса Последовательность чисел Лукаса (1п) задается соотношениями: /0 = 2, /| = 1, Vn G No ln+2 = ^n+l + In- 384. Пусть a = /3 = Показать, что числа Фибоначчи и Лукаса задаются формулами /П = 4=И-Л; /п = «п + Г- V 5
63 385. Доказать тождества 1) fan = fn In, 2) /2 =/2п + 2-(-1)Д 3) A-/n = |(Un-(-i)4-n). В статье А.Н. Рудакова [43] приведено доказательство следующего ре- зультата. Теорема (Э. Лукас, 1876 г.). Пусть q = 4k + 3 — простое число. Число М = 2q — 1 является простым тогда и только тогда, когда число Лукаса l^q-i делится на М. С помощью этой теоремы Э. Лукас установил простоту числа М = 2127 — 1, которое с 1877 г. по 1951 г. являлось самым большим извест- ным простым числом. Как сообщается в [43], доказательство простоты указанного 38-значного числа М потребовало около 100 часов вычислений (напомним, дело происходило 125 лет назад). 386. Пусть Г{ = l2i. Показать, что и = 3, и имеет место рекуррентное соотношение ri+1 = т[ — 2. 387. Написать программу, определяющую, является ли число 29 — 1 простым, если известно, что q = 4k + 3 — простое число.
Глава 6 Теория Пойа 388. Пусть S = {a, b, с, d} и G = {тл, тг2, тг3, тг4} — группа подстановок, действующая на множестве S: (а b с d \ ( a b с d \ а о с d J у о а с d J (а Ъ с d \ f а Ъ с d \ 7 1 ) 5 ^4 I 7 7 ] • а о а с J \ b а а с J 1) Убедиться в том, что G — группа, составив для неё квадрат Кэли. 2) Отношение эквивалентности на S порождается группой G. С помощью леммы Бернсайда найти число классов эквивалентности. 3) Выписать классы эквивалентности в явном виде. 389. Составляются трехбуквенные слова из букв а и Ь. Два различных слова считаются эквивалентными, если они получаются друг из друга при перемене местами крайних букв; например, abb ~ bba. Определить число классов эквивалентности, пользуясь леммой Бернсайда. Выписать классы эквивалентности в явном виде. 390. Вокруг стола рассаживаются п человек. Сколько существует раз- личных расположений, если отождествлять такие, которые получаются одно из другого сдвигом всех людей по часовой стрелке на произвольное, но одинаковое для всех число мест? 391. На листках бумаги пишут числа от 00000 до 99999 (числа, меньшие 10000, дополняют слева нулями). Будем считать, что при переворачивании цифры 0, 1, 8 не меняются, а цифры 6 и 9 переходят друг в друга. На- пример, для чисел 06981 и 18690 можно приготовить только один листок. Сколько всего листков понадобится? 392. Составляются ожерелья из бусин трех цветов. Каждое ожерелье состоит из 1) 5; 2) 6 бусин. Не будем различать ожерелья, получающиеся друг из друга поворотом в плоскости. Пользуясь леммой Бернсайда, найти число различных ожерелий.
65 393. Решить предыдущую задачу в предположении, что не различа- ются ожерелья, получающиеся друг из друга поворотом в пространстве. 394. Решить задачи 392, 393 с помощью следствия из теоремы Пойа. 395. Сколько ожерелий можно составить из двух красных, двух зелё- ных и двух синих бусин в предположениях задач 392 и 393? 396. Завод выпускает погремушки в виде кольца с надетыми на него р красными и q синими шариками. Сколько различных погремушек мо- жет быть выпущено? Две погремушки считаются одинаковыми, если мо- гут быть получены друг из друга передвижением шариков по кольцу или переворачиванием. Еще один подход к задаче о числе ожерелий продемонстрирован в §8.4. Некоторые общие результаты по подсчету числа ожерелий содержатся в §8.10. 397. Сколькими способами можно раскрасить в к цветов 1) рёбра; 2) грани тетраэдра, который может свободно вращаться в пространстве? 398. Сколькими способами можно раскрасить в к цветов 1) вершины; 2) рёбра; 3) грани куба, который может свободно вращаться в простран- стве? 399. Сколькими геометрически различными способами можно раскра- сить вершины куба в два цвета так, чтобы вершин каждого цвета было поровну? 400. Сколькими способами можно раскрасить 5 рёбер куба в синий цвет, а остальные рёбра в красный цвет? 401. Найти число существенно различных способов размещения вось- ми одинаковых пометок на шахматной доске размера 8x8. Два способа разметки считаются существенно различными, если их нельзя преобразо- вать друг в друга вращением доски или отражением относительно любой из четырёх осей симметрии. 402. Конструктор интегральных схем строит чипы с 16 элементами, расположенными в виде матрицы 4x4. Чтобы реализовать различные схе- мы, эти элементы нужно соединять; непосредственно соединяются только элементы, соседние по горизонтали или по вертикали, например
66 Глава 6. Теория Пойа Чтобы напылить межкомпонентные соединения, необходим фотошаблон рисунка соединений. Для двух рисунков, показанных выше, используется один и тот же фотошаблон (схемы симметричны относительно диагонали). Сколько требуется фотошаблонов для того, чтобы на этих чипах реализо- вать все возможные рисунки соединений? 403. Следующие две картинки называются соответственно ’’Звезда Да- вида” и ’’Мечи Магомета”: Представим себе, что эти фигуры составлены из кусков проволоки, спаян- ных в точках пересечений. Сколько существует различных звёзд и мечей с точки зрения вида пересечений? Две фигуры отождествляем, если одну из них можно переместить в пространстве так, что они становятся нераз- личимыми по виду пересечений. Например, См. также задачи 445, 485 и §8.10.
Глава 7 Введение в теорию графов 7.1. Определения и примеры Напомним некоторые стандартные обозначения. Nn — пустой граф с п вершинами; Кп — полный граф с п вершинами; Kn,m — полный двудольный граф, в долях которого пит вершин; Сп — циклический граф с п вершинами; Wn = Ni + Cn-i — колесо; Еп — граф, соответствующий п-мерному кубу. Простой граф — граф без петель и кратных рёбер. 404. Пусть Gn — простой граф с множеством вершин {щ,...,цп}, в котором вершины vt и Vj смежны тогда и только тогда, когда числа i и j взаимно просты. Изобразить графы G4 и G6 и найти их матрицы смежно- сти. Показать, что если тп < п, то Gm С Gn. 405. Для графов из каждой пары графов, изображенных на рисунке, выяснить, изоморфны ли они. 406. Найти все (с точностью до изоморфизма) простые графы, в ко- торых не более пяти вершин. 407. Сколько существует попарно неизоморфных простых графов с 10 вершинами и 1) 44; 2) 43 рёбрами?
68 Глава 7. Введение в теорию графов 408. Сколько существует помеченных простых графов с п вершинами? Сколько из них имеет m рёбер? 409. Доказать, что в простом графе с не менее чем двумя вершинами найдутся две вершины одинаковой степени. 410. Если в простом графе G с п > 2 вершинами в точности две имеют одинаковую степень, то либо в G, либо в G имеется ровно одна изолиро- ванная вершина. Доказать. 411. Пусть в простом графе никакие две вершины одинаковой степени не соединены цепью длины 2. Доказать, что в графе есть висячая вершина. 412. Для любого конечного множества неотрицательных целых чисел, в котором количество нечётных чисел чётно, существует граф, для кото- рого это множество является множеством степеней его вершин. Доказать. 413. Показать, что рёберные графы L(Kn) и £(КП)ТО) являются регу- лярными. 414. Доказать, что при п > 2 звёздный граф К\^п не является рёберным графом. 415. Пусть pi, р2,..., рп — степени вершин графа G. Сколько вершин и рёбер содержит рёберный граф L(G)? 416. Доказать, что простой граф изоморфен своему рёберному гра- фу тогда и только тогда, когда он является дизъюнктным объединением циклических графов. 417. Привести примеры (когда это возможно) 1) двудольного графа, являющегося регулярным; 2) кубического графа с 9 вершинами; 3) (для каждого п) простого графа с п вершинами и (w-1An~2) рёбрами; 4) (для каждого п) простого графа с п вершинами, изоморфного своему рёберному графу; 5) связных графов, являющихся регулярными графами степени 4. 418. Какие из платоновых графов являются двудольными? 419. [Теорема Кёнига.] Граф является двудольным тогда и только тогда, когда все его циклы имеют чётную длину. Доказать. 420. Доказать, что в Еп нет циклов нечётной длины. 421. Доказать, что в непустом двудольном регулярном графе доли со- держат равное число вершин.
69 422. Может ли регулярный степени выше 1 двудольный граф иметь мосты? 423. В связном графе степени четырёх вершин равны 3, а степени остальных вершин равны 4. Доказать, что нельзя удалить ребро так, чтобы граф распался на две изоморфные компоненты связности. 424. Пусть в графе среди любых четырёх вершин найдётся вершина, смежная с тремя остальными. Доказать, что радиус графа равен единице. 425. Найти дополнения к графам, соответствующим тетраэдру, кубу и октаэдру. 426. Вычислить 1) С4 + Я2; 2) Кп + Кт-, 3) Кп<т-, 4) G + Н (G и Н — простые графы). 427. Пусть G, Н и К — простые графы; доказать или опровергнуть следующие равенства: 1) G U (Я + К) = (G U Я) + (G U К); 2) G + (Я U К) = (G + Я) U (G + К). 428. Найти матрицы смежности графов Kn,Nn и Сп. 429. Чем характерна матрица смежности двудольного графа? 430. Какова связь между матрицами смежности простого графа и его дополнения? 431. Пусть А — матрица смежности регулярного графа степени к. До- казать, что к есть собственное значение матрицы А. Найти отвечающий ему собственный вектор. 432. В графе Петерсена найти циклы длины 5, 6, 8 и 9. 433. В графе Петерсена найти разрезы из 3, 4 и 5 рёбер. 434. Доказать, что дополнение к (простому) несвязному графу есть связный граф. 435. Доказать, что рёберный граф связного графа связен. 436. Пусть G — граф с множеством вершин {щ,..., г>п} и матрицей смежности А. Доказать, что число маршрутов длины к из Vi в Vj равно (гД)-му элементу матрицы Ак. Показать также, что если G — простой граф, то число треугольников (циклов длины 3) в G равно tr А3/6 (где tr А = о,ц — след матрицы А). Верно ли, что число циклов длины 4 равно tr А4/8?
70 Глава 7. Введение в теорию графов 437. Основываясь на результате предыдущей задачи, предложите ал- горитм определения диаметра графа по его матрице смежности. 438. Пусть в простом графе G степень каждой вершины не меньше г, где г > 2. Доказать, что в G существует цикл длины > г + 1. 439. Доказать, что при п > 10 верно утверждение: если в простом графе 2п вершин и степень каждой вершины не меньше п, то в графе имеется подграф К3 3. 440. [Экстремальная теорема Турана.] Пусть G — простой граф с 2п вершинами, не содержащий треугольников. Доказать, что в G не более п2 рёбер и привести пример, когда эта верхняя граница достигается. 441. Найти максимальное число рёбер в простом графе с 2п + 1 вер- шинами, не содержащем треугольников. 442. Найти радиус и диаметр графа Петерсена. 443. Графом Клебша называют граф, образованный вершинами, рё- брами и главными диагоналями четырёхмерного куба. Другими словами, в качестве вершин графа Клебша можно рассматривать упорядоченные двоичные наборы 0000, 0001,..., 1111; и вершины являются смежными то- гда и только тогда, когда их двоичные представления различаются либо в одном, либо во всех разрядах. Найти радиус и диаметр графа Клебша. 444. Для каждого п построить пример графа, центр которого состоит из п вершин и не совпадает с множеством всех вершин. 445. Пусть а) п = 4; б) п = 5. 1) Найти цикловой индекс группы подстановок на множестве рёбер полного графа с п вершинами, порождённых перестановками вершин. 2) С помощью теоремы Пойа найти число попарно неизоморфных про- стых графов с п вершинами и тп рёбрами. 7.2. Гамильтоновы и эйлеровы графы 446. Для каких чисел тип следующие графы являются а) эйлеровы- ми; б) гамильтоновыми: 1) Кп; 2) Кгг^п\ 3) Wn; 4) Еп? 447. Привести пример эйлерова графа, не являющегося гамильтоно- вым, и гамильтонова графа, не являющегося эйлеровым.
71 448. Пусть G — двудольный граф, доли которого содержат тип вер- шин соответственно. Доказать, что 1) если G — гамильтонов граф, то т = п; 2) если G — полугамильтонов граф, то \т — n| < 1. 449. Может ли а) конь; б) король; в) ладья побывать на каждой клет- ке шахматной доски размером 8x8 ровно один раз и последним ходом возвратиться в исходную позицию? Решить такую же задачу для доски 7x7. 450. Можно ли прогуляться по парку и его окрестностям (см. рис.) так, чтобы при этом перелезть через каждый забор ровно один раз? 451. При каких п можно пометить вершины выпуклого угольника так, чтобы для каждой пары различных натуральных чисел, не превосходящих п, нашлась сторона, концы которой помечены этими числами? 452. Доказать, что если граф G связен и имеет к > 0 вершин нечётной степени, то минимальное число не имеющих общих рёбер цепей, объеди- нение которых содержит каждое ребро графа G, равно к/2. 453. Дан кусок проволоки длиной 120 см. Какое наименьшее число раз придется ломать проволоку, чтобы изготовить каркас куба с ребром 10 см? 454. Можно ли сетку, составленную из единичных квадратов, представить в виде объединения 1) восьми ломаных длины 5; 2) пяти ло- маных длины 8? 455. С помощью алгоритма Флёри найти эйлеров цикл в графе на рисунке.
72 Глава 7. Введение в теорию графов 456. Любой разрез эйлерова графа состоит из чётного числа рёбер. Доказать. 457. Доказать, что рёберный граф простого эйлерова графа является одновременно эйлеровым и гамильтоновым. 458. Доказать, что рёберный граф простого гамильтонова графа яв- ляется гамильтоновым. 459. Пусть G — простой негамильтонов граф, содержащий п > 3 вер- шин. Доказать, что если несмежные вершины и и v соединяет гамильто- нова цепь, то 1) число вершин, не смежных с и, не меньше числа вершин, смежных с г (и наоборот); 2) р(и) + р(г) < п - 1. 460. С помощью предыдущей задачи доказать теорему О.Оре. Если в простом графе с п вершинами (п>3) для любой пары несмежных вершин и и v выполняется неравенство р(и) + p(v) > п, то граф является гамильтоновым. 461. При каком наименьшем числе рёбер в простом графе с 10 верши- нами этот граф заведомо гамильтонов? 462. [Теорема Л. Пота.] Пусть в графе G имеется п > 3 вершин и для любого натурального к < число вершин со степенью, не превос- ходящей к, меньше к. Тогда G — гамильтонов граф. Доказать. 7.3. Деревья 463. Найти все (с точностью до изоморфизма) деревья, в которых не более семи вершин. 464. Волейбольная сетка имеет вид прямоугольника 50 х 600 клеток. Какое наибольшее количество верёвочек можно перерезать так, чтобы сет- ка не распалась на куски?
73 465. Доказать, что каждое дерево является двудольным графом. Ка- кие деревья являются полными двудольными графами? 466. Если в дереве не менее двух рёбер, то его радиус меньше диаметра. Доказать. 467. Доказать, что центр дерева состоит из одной вершины, если диа- метр дерева есть чётное число, и двух вершин в противном случае. 468. Верно ли, что в дереве с нечётным диаметром любые две простые цепи наибольшей длины имеют хотя бы одно общее ребро? 469. Выразите радиус дерева через его диаметр. 470. Пусть п — количество вершин дерева, г — его радиус. Доказать, что п > 2г. 471. Верно ли, что если диаметр графа равен к > 2, то граф имеет стягивающее дерево диаметра к? 472. Для каждого из указанных ниже графов найти какое-нибудь стя- гивающее дерево и фундаментальную систему циклов относительно него. 1) К5; 2) К3,3: 3) ИД 4) С6; 5) граф Петерсена. 473. Пусть Ti и Т-2 — стягивающие деревья связного графа G. Пока- зать, что для любого ребра е из Т\ существует ребро f из Т2 такое, что по- сле ’’замены” в Д ребра е на ребро f вновь получится стягивающее дерево. (С помощью подобной процедуры можно построить последовательность стягивающих деревьев ТД ... ,Т2, в которой каждое дерево получается из предыдущего заменой одного ребра). 474. Показать, что если щ,..., рп — заданные натуральные числа, то дерево с п > 1 вершинами, в котором степень Vk равна pk (для каждого к), существует в том и только в том случае, если Pk = 2(n — 1). 475. Доказать, что число рёберно-помеченных деревьев с п > 3 вер- шинами (в которых помечены не вершины, а рёбра) равно п”-3. 476. Показать, что при больших п вероятность того, что случайным образом выбранная вершина дерева с п вершинами является висячей, при- ближенно равна 1/е. 477. Найти стягивающее дерево минимального веса для каждого графа на рис.
74 Глава 7. Введение в теорию графов щ 3 v2 1 v3 4 v4 Vi 2 v2 4 v3 1 v4 Корневые деревья Дерево с выделенной вершиной ('корнем) называют корневым дере- вом. 478. Найти число помеченных корневых деревьев с п вершинами. В книгах, посвященных исследованию структур данных (например, [30], [42]), корневое дерево определяют рекурсивно следующим образом. Корневое дерево Т — это непустое конечное множество Т с элемен- тами, называемыми вершинами, такими, что 1) имеется выделенная вершина, называемая корнем данного дерева; 2) если множество остальных вершин непусто, то оно разбивается на m множеств Ti,... ,Tm, каждое из которых в свою очередь является корневым деревом. Деревья Ti,..., Tm называются поддеревьями данного корня. Из опре- деления следует, что каждая вершина дерева является корнем некоторого своего дерева. Число поддеревьев дерева с корнем г — порядок вершины г. Вершины нулевого порядка называют листьями; остальные вершины называют внутренними. 479. Сколько листьев имеет дерево с к внутренними вершинами, по- рядок каждой из которых равен двум? 480. В турнире по олимпийской системе (’’проигравший выбывает”) участвует п человек. Сколько встреч будет проведено? 481. Некто купил курицу. После того, как она снесла два яйца, её съели. Из яиц вывелись цыплята. Петухов съедали сразу, а куриц — после того, как они сносили по два яйца. И т.д. В какой-то момент вывелись одни петухи, и процесс закончился. Сколько куриц было съедено, если съели 97 петухов? 482. Сколько листьев имеет дерево, в котором (кроме листьев) содер- жится П1 вершин порядка 1, п2 вершин порядка 2,..., ns вершин порядка а? Бинарное дерево Т определяется рекурсивно следующим образом: Т = ф (пустое дерево) илиТ =< L,c,R> — упорядоченная тройка, где с -
75 корень дерева (элемент некоторого множества), L и R бинарные деревья; L и R называют соответственно левым и правым поддеревом дерева Т. Вершинами дерева называют его корень, корни его поддеревьев, корни поддеревьев его поддеревьев и т.д. 483. Найти число различных бинарных деревьев с п вершинами. Высота бинарного дерева — наибольшая длина цепи от корня до какой-либо другой вершины. Дерево называют сбалансированным, если для любой его вершины высоты левого и правого поддеревьев различаются не более чем на 1 (например, если одно поддерево пусто, то второе должно состоять только из своего корня). 484. Пусть высота сбалансированного бинарного дерева равна п. Найти наибольшее и наименьшее число вершин в этом дереве. Бинарное дерево назовем полным, если расстояния от всех листьев до корня равны между собой. 485. Будем раскрашивать вершины полного бинарного дерева в два цвета, отождествляя при этом раскраски, которые могут быть получены друг из друга перестановками левого и правого поддеревьев некоторых вершин. Подсчитать число различных раскрасок в два цвета полного би- нарного дерева высоты 2 с помощью а) леммы Бернсайда; б) следствия из теоремы Пойа. 7.4. Укладки графов 486. При каком к можно так расположить 6 точек на плоскости и соединить их попарно непересекающимися отрезками, чтобы каждая точка была соединена ровно с к другими? 487. При каких п графы Gn (определение см. в задаче 404) планарны? 488. Проверить формулу Эйлера, связывающую число вершин, рёбер и граней плоского графа, для следующих графов: 1) Wn; 2) К2,п; 3) графа, соответствующего клетчатому полю s х t. 489. Пусть Д — число А;-угольных граней выпуклого многогранника. Доказать, что ЗД + 2Д + Д > 12 + Д 4- 2Д + ЗД + 4Д0 + .... 490. Пусть в связном плоском графе каждая грань ограничена s рё- брами, а степень каждой вершины равна р. Доказать, что | | где m — количество рёбер графа.
76 Глава 7. Введение в теорию графов 491. С точностью до изоморфизма найти все регулярные связные плос- кие графы, в которых каждая грань ограничена одним и тем же числом рёбер. 492. Всегда ли планарен рёберный граф планарного графа? 493. Пусть в простом графе G не менее 11 вершин. Доказать, что граф G и его дополнение G не могут быть одновременно планарными. 494. Используя тот факт, что в простом плоском графе есть вершина степени не больше 5, доказать, что его вершины можно раскрасить не более чем в 6 цветов так, чтобы смежные вершины были разного цвета. 495. Планарен ли простой регулярный граф степени 5, в котором 10 вершин? 496. Доказать, что не существует плоского графа с пятью гранями такого, что любые две его грани имеют общее ребро. 497. [Теорема Эрдёша.] Пусть R иг — максимальное и минималь- ное расстояния, определяемые п точками на плоскости. Тогда R встре- чается не более п раз, а г — не более Зп — 6 раз. Доказать. 498. В любом плоском графе найдутся две грани с одинаковым числом сторон. Доказать. Назовем картой связный плоский граф без мостов. 499. Грани карты G можно раскрасить в два цвета так, чтобы любые две смежные грани были разного цвета, тогда и только тогда, когда G - эйлеров граф. Доказать. Числом скрещиваний cr(G') графа G называется наименьшее чис- ло попарных пересечений рёбер, получаемых при изображении графа на плоскости. В этом определении пересечение понимается так: жордановые кривые, соответствующие двум рёбрам, пересекаются в точке, которая ли- бо не соответствует никакой вершине графа, либо соответствует вершине, не инцидентной хотя бы одному из данных двух рёбер. Ясно, что cr(G') = 0 тогда и только тогда, когда G — планарный граф. 500. Пусть граф G образован вершинами, сторонами и главными диа- гоналями правильного 2п-угольника. Доказать, что cr(G) = 1. 501. Найти crf/G) и сг(Аз>3). 502. Доказать, что а) при п > 5 сг(Кп) > С^/5; б) при т, п>3 сг(7Гто>„) > С^С^/Э.
77 503. Найти число скрещиваний графов Jf6, и 504. Доказать, что сг(КТО)П) < |>] • [^] • [|] • [^] . Подробно вопрос о числе скрещиваний полного графа и полного дву- дольного графа изучается в книге [44]. 505. Доказать, что в любом графе G порядка п найдётся такая верши- на, что после её удаления (вместе с инцидентными ей рёбрами) возникнет граф G', удовлетворяющий условию cr(G) - > cr(G"). TI 506. Пусть Р — граф Петерсена. Доказать, что cr(F) = 2. 7.5. Ориентированные графы. Алгоритмы 507. Пусть А — матрица смежности орграфа с множеством вершин {щ,..., гп}. Какой смысл имеют суммы строк и суммы столбцов матрицы Л? Доказать, что (г, д)-й элемент матрицы Ак равен числу путей длины к ИЗ Vi В Vj. 508. В дереве с п вершинами рёбра ориентируются случайным обра- зом. Какова вероятность того, что найдётся вершина, из которой ведут пути ко всем остальным вершинам? 509. Пусть G — связный граф. Зафиксируем некоторую его вершину v. Доказать, что можно так ориентировать рёбра графа, что в получившемся орграфе существует путь от v до любой другой вершины. Ориентированный граф называют сильно связным, если для любых его вершин и и v существует путь из и в v. 510. В связном графе степени всех вершин чётны. Доказать, что можно так ориентировать рёбра графа, чтобы 1) получившийся орграф был сильно связным; 2) для каждой вершины полустепень исхода была равна полустепени захода. 511. В ориентированном графе со связным основанием для каждой вершины полустепень исхода равна полустепени захода. Доказать, что ор- граф эйлеров.
78 Глава 7. Введение в теорию графов 512. Доказать, что существует циклическая последовательность длины кп, составленная из к различных символов, в которой встречаются все возможные строки из этих символов длины п. 513. Доказать, что существует последовательность длины кп + к — 1, составленная из к различных символов, в которой встречаются все воз- можные строки из этих символов длины п. 514. По кругу расставлены вазы с конфетами. Ход состоит в том, что из какой-то вазы берутся все конфеты и раскладываются по одной в каж- дую вазу, начиная со следующей вазы (движение происходит по часовой стрелке). Доказать, что за несколько ходов из любого первоначального распределения конфет по вазам можно получить любое другое. 515. В связном графе чётное число рёбер. Доказать, что можно так ориентировать его рёбра, что в получившемся орграфе полустепени исхода всех вершин будут чётными. 516. В орграфе, основанием которого служит колесо Wn, нет источни- ков и стоков. Доказать, что орграф сильно связный. Связный граф называют ориентируемым, если ориентацией его рё- бер можно получить сильно связный орграф. 517. [Теорема Роббинса.] Связный граф является ориентируемым тогда и только тогда, когда в нем нет мостов. Доказать. 518. Найти кратчайший путь от 1-й вершины до всех остальных (см. рис. 1, а, б). 519. Пусть проекты описываются взвешенными графами (см. рис. 2, а, б), где дуги соответствуют операциям (этапам) проекта, а вес дуги обо- значает время выполнения соответствующей операции. Найти наименьшее время выполнения проектов, критические пути и резервы времени для вы- полнения каждой операции.
79 520. Найти максимальные потоки и минимальные разрезы в транс- портных сетях (см. рис. 3, а, б). Число рядом с дугой есть её пропускная способность. 521. Пусть имеется т неженатых мужчин, п незамужних женщин и к свах. У каждой свахи есть список своих клиентов; между любыми муж- чиной и женщиной из этого списка сваха может устроить брак. Для г-й свахи число устроенных ею за год браков не превосходит числа Ь^. Переве- сти задачу нахождения наибольшего числа браков, которые могут устро- ить свахи за год, в задачу нахождения максимального потока в некоторой сети. 7.6. Турниры Орграф, в котором каждые две (различные) вершины соединены ровно одной дугой, называют турниром. Таким образом, турнир получается из полного графа ориентацией рёбер. 522. Доказать, что сумма квадратов полустепеней исхода всех вершин турнира равна сумме квадратов полустепеней захода. 523. Доказать следующие утверждения. 1) Существует турнир, в котором нет ни одного замкнутого пути.
80 Глава 7. Введение в теорию графов 2) При этом орграф обязательно содержит ровно по одному источнику и стоку. 3) Существует ровно п! турниров, имеющих по п вершин и обладающих свойством из и. 1). 524. Доказать, что при любом п, не равном 2 и 4, существует турнир с п вершинами, в котором для любой упорядоченной пары вершин (и, г) найдётся путь из и в г, состоящий из одной или двух дуг. Числом очков вершины турнира называют её полустепень исхода. 525. В турнире все вершины имеют одно и то же число очков. Дока- зать, что для произвольных вершин и и v найдётся путь из и в v из одной или двух дуг. 526. Пусть и — вершина турнира с наибольшим числом очков. Дока- зать, что для любой вершины v есть путь из и в v, состоящий из одной или двух дуг. 527. Будем говорить, что вершины и и v орграфа являются взаимно достижимыми, если существуют пути из и в v и из v в и. Показать, что это отношение является отношением эквивалентности. Показать также, что в случае турнира соответствующие классы эквивалентности можно пронумеровать таким образом, что из вершины, принадлежащей классу с меньшим номером, идут дуги во все вершины из классов с большими номерами. 528. Показать, что если турнир не является сильно связным графом, то в нём существует дуга, изменение ориентации которой приводит к сильной связности графа. 529. Доказать, что любой турнир полугамильтонов. 530. Если турнир с п вершинами сильно связен, то в нём есть циклы длины 3, 4,..., п (в частности, он гамильтонов). Доказать. Треугольники Назовём треугольником замкнутый путь из трёх дуг. 531. У двух вершин турнира одинаковое число очков. Доказать, что в турнире есть треугольник. 532. Доказать, что если турнир — сильно связный граф, то любая его вершина входит в некоторый треугольник.
81 533. Доказать, что число треугольников в турнире с п вершинами рав- но п сп-У2с1, 1=1 где для каждого i через Si обозначено число очков г-й вершины. 534. Доказать, что число треугольников в турнире с п вершинами не больше га2~га при нечётном п и п ~£п при чётном п. 7.7. Доминирование, независимость, покрытия, паросочетания ”Во многих прикладных задачах требуется найти в конечном множестве объектов максимальную систему объектов, попарно не связанных друг с другом, или же выбрать минимальную систему объектов, связанных со всеми другими. Формулировки подобных задач на языке теории графов приводят к понятиям независимости и покрытия.” ([7]) Подмножество V С V вершин графа G =< V, Е > называется доми- нирующим (или внешне устойчивым^), если каждая вершина из V\V смежна с некоторой вершиной из V. Наименьшую мощность доминирующего множества графа называют числом доминирования графа (или числом внешней устойчивости) и обозначают 6(G). 535. Найти число доминирования для следующих графов: Kn; Wn; Kn>m; Сп; графа Петерсена; графа Клебша (определение см. в за- даче 443). 536. Пусть в графе нет изолированных вершин. Доказать, что в нем можно указать такое доминирующее множество, дополнение к которому также является доминирующим множеством. 537. Показать, что для регулярного графа G степени ken верши- нами справедливо неравенство 6(G) > Привести пример графа, для которого в указанном соотношении имеет место равенство. 538. Найти число доминирования cZ-мерного куба при d = 1, 2,3,4, 7. 539. Показать, что 6(E2m-i) = 22т~т~1.
82 Глава 7. Введение в теорию графов 540. Пусть граф G порядка п имеет диаметр 2. Доказать, что 6(G) < \/n In n + 1. Множество попарно несмежных вершин (рёбер) графа называется независимым. Независимое множество — максимальное, если оно не является подмножеством некоторого другого независимого множества. Независимое множество с наибольшим числом элементов называется наибольшим. 541. Доказать, что наибольшее независимое множество является мак- симальным. 542. Доказать, что независимое множество вершин является макси- мальным тогда и только тогда, когда оно доминирующее. 543. Показать на примерах, что доминирующее множество вершин наименьшей мощности может быть независимым, а может и не быть та- ковым. Используется также следующая терминология. Независимое множе- ство вершин называют внутренне устойчивым. Независимое множество рёбер называют паросочетанием. Мощность наибольшего независимого множества вершин (рёбер) графа G обозначают a0(G) (соответственно «1(G)). 544. Доказать, что a0(En) = 2П-1. Говорят, что вершина и ребро графа покрывают друг друга, если они друг другу инцидентны. Подмножество V' С V вершин графа G =< V, Е > называется покрытием графа, если вершины из V покрывают (в совокупности) все рёбра графа (т.е. каждое ребро из Е инцидентно хотя бы одной вершине из V). 545. Доказать, что подмножество вершин графа является его покры- тием тогда и только тогда, когда дополнение к нему — независимое мно- жество. Подмножество Е' с Е рёбер графа G =< V, Е > называется рёбер- ным покрытием графа, если рёбра из Е' покрывают (в совокупности) все вершины графа (т.е. каждая вершина из V инцидентна хотя бы одно- му ребру из Е'). Таким образом, нет рёберных покрытий лишь у графов, имеющих изолированные вершины. Наименьшую мощность покрытия (рё- берного покрытия) графа G обозначают /3q(G) (соответственно /31(G)). 546. Найти O'o(G), /3o(G), од (G), /31 (G) для следующих графов: Cn- Kn; Wn; Кп>т; графа Петерсена.
83 547. Пусть граф G содержит п вершин. Доказать, что O'o(G) + /3q(G) = Ti. 548. Доказать, что для любого графа G справедливо неравенство fio(G) > <м(С9- 549. Пусть граф G порядка п не содержит изолированных вершин. Тогда «1 (G) + /31 (G) = п. Доказать. 550. Для любого двудольного графа G справедливо, что /3o(G) = ai(G). Доказать. Паросочетание называется совершенным, если оно одновременно яв- ляется рёберным покрытием, /с-фактором графа называется его остов- ный регулярный подграф степени к. Таким образом, совершенное паросо- четание и 1-фактор есть совпадающие понятия. 551. Доказать, что если в графе 8 вершин, и степень каждой вершины равна 4, то в графе есть совершенное паросочетание. 552. В графе 2п вершин (п > 2). Какие три вершины ни взять, среди них найдётся смежная с двумя другими. Доказать, что в графе существует совершенное паросочетание. 553. Доказать, что кубический граф, являющийся гамильтоновым, распадается на совершенные паросочетания. 554. Доказать, что граф Петерсена не распадается на совершенные паросочетания. Получить отсюда негамильтоновость графа Петерсена. 555. Привести пример кубического графа, в котором нет совершенного паросочетания. 556. Если граф, все вершины которого имеют нечётную степень, рас- падается на совершенные паросочетания, то в нем либо все рёбра — мосты, либо нет мостов. Доказать. 557. Показать, что граф К2п распадается на 2п — 1 совершенное паро- сочетание. 558. Можно ли раскрасить рёбра графа Кп в п — 1 цветов так, чтобы все рёбра, исходящие из произвольной вершины, были разного цвета? 559. Показать, что число различных совершенных паросочетаний в (помеченном) графе К2п равно = (2п — 1)!!. 560. Показать, что число различных наибольших паросочетаний в (по- меченном) графе ^n+i равно (2n+ 1)!!.
84 Глава 7. Введение в теорию графов 561. Показать, что число различных совершенных паросочетаний в (помеченном) графе Кп<п равно п!(п — 1)!... 2!. Задача о свадьбах Пусть G — двудольный граф с долями Vi и Vj Совершенным паро- сочетанием из Vi в V2 называется паросочетание в G, покрывающее Vi (т.е. для всякой вершины из Vi в паросочетании найдётся инцидентное ей ребро). Пусть Л С V — подмножество вершин графа G =< V, Е >. Окруже- нием множества А называют множество Г(Л)= и Г(г)\л, veA где Г (г) — множество вершин, смежных с v. Имеет место следующая теорема (см., например, [7] или [8]): Теорема [Ф. Холл, 1935 г.] Совершенное паросочетание из Vi в V2 в двудольном графе G существует тогда и только тогда, когда УЛ с Vi |Г(Л)| > |Л|. 562. В любом непустом регулярном двудольном графе существует со- вершенное паросочетание. Доказать. 563. Любой непустой регулярный двудольный граф распадается на совершенные паросочетания. Доказать. 564. Задача о свадьбах. Имеется множество юношей, каждый из которых знаком с некоторыми девушками. При каких условиях можно одновременно женить всех юношей так, чтобы каждый из них женился на знакомой ему девушке? 565. В некотором районе, состоящем из нескольких деревень, число женихов равно числу невест. В каждой деревне общее число женихов и невест не больше половины общего их числа. Доказать: можно всех пере- женить так, что в каждой паре жених и невеста будут из разных деревень. 566. На танцевальном вечере каждый юноша знаком с к девушками, а каждая девушка знакома с к юношами. Доказать, что можно провести к (медленных) танцев так, чтобы каждый участник вечера станцевал со всеми своими знакомыми (противоположного пола). 567. На шахматной доске пометили 16 из 64 клеток так, что на каждой вертикали и горизонтали оказалось по две помеченные клетки. Доказать,
85 что на помеченных клетках можно расставить 8 чёрных и 8 белых фигур так, чтобы на каждой вертикали и каждой горизонтали стояло по одной белой и одной чёрной фигуре. 568. Вуз посылает 8 юношей и 8 девушек на стажировку по восьми спе- циальностям в 8 зарубежных университетов, причем каждый университет принимает по два человека и учит их двум разным специальностям, и каждой из восьми специальностей учат два университета. Всегда ли мож- но так распределить студентов, чтобы в каждом университете стажирова- лись юноша и девушка, и при этом как юноши, так и девушки обучались (в совокупности) по всем восьми специальностям?
Глава 8 Дополнительные задачи Ultra posse пето obligatur. 1 8.1. Инвариант и полуинвариант Ниже предлагаются задачи, в каждой из которых задан некоторый объект и описаны преобразования, которые разрешено производить над этим объ- ектом. Каждый раз задается вопрос, можно ли объект из одного состояния перевести в другое. Предположим, что существует некоторая характеристика объекта, ко- торая не меняется при указанных преобразованиях (это и есть инвариант задачи/ Если для двух состояний объекта значения инварианта различ- ны, то ответ на заданный вопрос будет, очевидно, отрицательным. 569. Дан куб ABCDA^B^CiDy. В вершинах A, B,D и середине М ре- бра Лу/ сидели четыре мухи. В некоторый момент одна из мух взлетела и, пролетев параллельно прямой, соединяющей некоторых двух из оставших- ся мух, села на поверхность куба. Затем снова какая-то муха совершила аналогичный маневр. И т.д. Могут ли мухи после нескольких перелетов оказаться в вершинах А, В, D, МД 570. На острове Серобуромалин обитают 13 серых, 15 бурых и 17 ма- линовых хамелеонов. Если встречаются два хамелеона разного цвета, то они одновременно меняют свой цвет на третий. Может ли случиться так, что через некоторое время все хамелеоны будут одного цвета? 571. Имея числа а и Ь, можно получить число a + b + ab. Можно ли, имея первоначально числа 1 и 4, с помощью (многократного) применения указанной операции, получить а) 1999; б) 2000? 572. Числа а и b можно заменить числами и Можно ли, имея первоначально числа 2, у/2 и 1/Д2, с помощью (многократного) примене- ния указанной операции, получить числа 1, д/2 и 1 + -\/2? дикого нельзя обязать сверх его возможностей (лат.).
87 573. На льду лежат три шайбы. Хоккеист бьет по одной из них так, что она пролетает между двумя другими. Так он делает 25 раз. Могут ли после этого шайбы оказаться на исходных местах? 574. Допускаются следующие преобразования квадратного трехчлена /(а?) = ах2 + Ьх + с: 1) можно менять местами а и с; 2) многочлен /(т) можно заменить на многочлен f(x — к), где к — произвольная константа. Можно ли из х2 — Зх — 4 получить х2 — х — 1? 575. По кругу растут п деревьев. Первоначально на каждом дереве сидит один чиж. Иногда какие-нибудь два чижа перелетают в противопо- ложных направлениях, причем каждый чиж садится на дерево, соседнее с тем, с которого он взлетел. Могут ли чижи собраться на одном дереве? 576. По кругу расположены 12 лампочек, каждая из которых может находиться в двух состояниях (гореть или не гореть). Допускается (многократное) выполнение следующей операции: изменить состояния любых трех рядом расположенных лампочек. Вначале горит одна лампочка. Можно ли зажечь все лампочки? 577. В вершине Mi 12-угольника стоит минус, в остальных — плюс. Можно одновременно менять знак в любых а) 6; б) 4; в) 3 последователь- ных вершинах. Доказать, что нельзя добиться того, чтобы минус стоял в вершине, соседней с Mi, а в остальных вершинах были плюсы. Полуинвариант — (числовая) характеристика объекта, которая меня- ется монотонно (например, увеличивается) при каждом преобразовании. 578. По кругу выписаны п чисел. Если последовательно против часо- вой стрелки стоят числа a, b,c,d и (а — d)(b — с) < 0, то числа b и с можно поменять местами. Доказать, что эту операцию можно проделать лишь конечное число раз. 579. Числа а и b можно заменить числами а + | и b — Можно ли, имея первоначально два ненулевых числа, с помощью (многократного) применения указанной операции, прийти к исходному набору чисел? 580. Каждый член парламента имеет не более 2k +1 врагов. Доказать, что парламент можно разбить на две палаты так, что каждый депутат будет иметь в своей палате не более к врагов. При решении двух следующих задач используется понятие чётности перестановки.
88 Глава 8. Дополнительные задачи 581. Имеются катушечный магнитофон, пять катушек с магнитной лентой и одна пустая катушка. Можно ли добиться того, чтобы каждая лента оказалась на той же катушке, но перемотанная? 582. В плоской квадратной коробке размещены 15 одинаковых фишек квадратной формы, одно место остается свободным. Можно ли, не выни- мая фишек из коробки, а лишь передвигая их друг за другом на свободное место, перейти от одной конфигурации к другой? 1 5 9 13 2 6 10 14 3 7 11 15 4 8 12 2 5 9 13 1 6 10 14 3 7 11 15 4 8 12 583. Двое по очереди пишут цифры 2/с-значного числа, употребляя только цифры 1, 2, 3, 4, 5. Может ли второй игрок добиться того, чтобы полученное число делилось на 9, если первый стремится ему помешать? 1) к = 15; 2) к = 10. 584. Пусть вершины правильного 2п-угольника занумерованы по по- рядку числами от 1 до 2п. Доказать, что нельзя занумеровать вершины другого правильного 2п-угольника так, чтобы при любом совмещении мно- гоугольников (в плоскости) для некоторой вершины совпадали оба номера. 585. При каких п в таблице n х п 1 2 3 п п 1 2 п — 1 п — 1 п 1 п — 2 ; 2 3 4 1 можно выбрать п чисел от 1 до п так, чтобы любые два из них стояли в разных строках и разных столбцах? 586. Окружность разбита 2п точками на равные дуги. Доказать, что у любой замкнутой ломаной с вершинами во всех точках есть параллельные звенья. 587. Имеется бесконечная клетчатая доска. Фигура ’’волк” за один ход перемещается на п клеток в одном направлении (по горизонтали или вер- тикали) инап + 1 клетку в перпендикулярном направлении. За какое наи- меньшее число ходов ’’волк” сможет переместиться из какой-нибудь клетки в соседнюю (т.е. имеющую с исходной общую сторону)?
89 588. В n-мерном кубе покрашено более половины вершин. Доказать, что имеется не менее п ребер, у которых оба конца покрашены. 8.2. Задачи с целыми числами Теория чисел немало служит к изощрению разума начинающих и боль- шое проворство в исчислении приносит. Л. Эйлер 589. Пусть п — совершенное число (т.е. равное сумме всех своих натуральных делителей, отличных от самого числа). Найти сумму 590. Доказать, что число 11... 1, в десятичной записи которого Зп еди- ниц, делится на Зп и не делится на 3n+1. 591. Верно ли, что если сумма цифр числа делится на 81, то и число делится на 81? 592. Найти все п, при которых число ап = 1010... 01 (в его записи п единиц и п — 1 нулей) является простым. 593. В ряду 60 точек, двое поочерёдно заменяют любую точку на циф- ру. Доказать, что второй может сделать так, чтобы полученное число де- лилось на 13. 594. Если число с суммой цифр 2 делится на 19, то оно делится и на 13. Доказать. 595. Для каких натуральных тип число 2П + 1 делится на 2т — 1? 596. Доказать, что 2т — 1 делится на 2п + 1 тогда и только тогда, когда т делится на 2п. 597. Доказать, что при любом натуральном п 1) 2903п - 803п - 464п + 261™ делится на 1897; 2) 1492п - 1770™ - 1863п + 2141™ делится на 1946. 598. Доказать, что при любом нечётном п 1) 46п 4- 296 • 13п делится на 1947; 2) 2269п + 1779п + 1730п - 1776п делится на 2001.
90 Глава 8. Дополнительные задачи 599. Доказать, что если п — нечётное число, кратное 5, то 12п + 9П + 8” + 6П делится на 1991. 600. Доказать, что натуральное число не представимо в виде суммы последовательных натуральных чисел тогда и только тогда, когда оно яв- ляется степенью двойки. 601. Доказать, что (1 + 2)(1 + 22)(1 + 222)(1 + 223)...(1 + 22”-1) = 22” - 1. 602. Доказать, что число 22” — 1 имеет не менее п различных простых делителей. 603. Доказать, что любые два числа последовательности 3,5,17,...,1 + 22П,... взаимно просты. 604. Используя утверждение предыдущей задачи, доказать бесконеч- ность множества простых чисел. 605. Доказать, что число 22” + 22" 1 + 1 имеет не менее п различных простых делителей. Л. Эйлер предложил следующий способ разложения на множители нечётного числа, для которого известны два представления в виде сум- мы двух квадратов ([48]). Пусть п = х2 + у2 = u2 + v2, причем хин — чётные числа, а у и v - нечётные. Тогда х2 — u2 = v2 — у2 и х — u v + у a v — у х + u b ’ где a/Ь — несократимая дробь. Для некоторых целых t\ и t2 имеем равен- ства х — u = ati, v + у = at2, v — у = bti, х + u = bt2. Поскольку числа х ± и и v ± у чётны, а одно из чисел а или b нечётно, заключаем, что числа И nt2 чётны. Разрешив написанные выше уравнения относительно х и у, получим ati + bt2 at2 — bti x = ; y = 2 2
91 откуда п = х2 + у2 = | ((tzii + bi2)2 + (at2 - 6ii)2) = |(a2i? + b2i2 + a2t2 + 62i2) = |(a2 + 62)(i2 + i2) = (a2 + 62) ((ix/2)2 + (i2/2)2). 606. С помощью метода Л. Эйлера разложите на множители следую- щие числа: 1000001 = 1992 + 9802; 1000009 = 2352 + 9722; 1000049 = 6322 + 7752; 1000169 = 1552 + 9882; 1000361 = 6562 + 7552; 1000441 = 2402 + 9712; 1000529 = 6732 + 7402. 607. Обобщите метод Л. Эйлера для нахождения разложения на мно- жители нечётного числа п, для которого известны два представления вида п = х2 + dy2 = и2 + dv2. 608. Разложите на множители число 13 717421 = 7612 + 7 • 13702 = 4392 + 7 • 13902. 609. Пусть ab a2,..., an и &1; &2,..., Ьп — полные системы вычетов по модулю п. Доказать, что если п > 2, числа афц, a2, b2,..., anbn не образуют полной системы вычетов по модулю п. 610. Докажите, что для п > 2 п цифр п — 1 цифр 22 = 22 (mod п). 611. Доказать, что если a — 1 : km, то ak — 1 : km+} (где a,k, m E N). 612. Пусть a — произвольное натуральное число, (un) — последова- тельность, задаваемая рекуррентно = a, Un+i = aUn, n E N. Доказать, что при n > 3 разность двух соседних чисел последовательно- сти (wn+i — wn) делится на 10п-2. Замечание. Из результата данной задачи следует свойство стабилизи- руемое™ цифр ’’сверхстепеней”: какой бы разряд ни взять, начиная с некоторого числа в последовательности a, аа, аа<1, аа° ,... цифра в этом разряде не меняется. 613. Пусть р — простое число, n > 1, (р — 1)п 4-1 : п. Доказать, что п делится на р.
92 Глава 8. Дополнительные задачи 614. Пусть р — простое число, р > 3, к = [2р/3]. Доказать, что сумма биномиальных коэффициентов Ср + С2 + ... + Ср делится на р2. 615. Для каждого натурального п запишем сумму 1/т в виде an/[3n, где an и /Зп — взаимно простые числа. Найти все п, при которых /Зп не делится на 5. 616. Для п = 10 определить, чётно или нечётно число упорядоченных n-элементных наборов натуральных чисел (a?i, х2,..., хД таких, что Xi Х2 Хп 8.3. Числа Кармайкла Составное число m такое, что для любого числа а, взаимно простого с т, выполняется соотношение am~l = l(modm), называется числом Кар- майкла. 617. Доказать, что любое число Кармайкла нечётно. 618. Доказать, что числа 561, 46657 = б6 + 1,101101 являются числами Кармайкла. 619. Пусть тп — свободное от квадратов составное число, обладающее тем свойством, что тп — 1 : р — 1 для произвольного простого числа р, делящего т. Доказать, что т — число Кармайкла. Справедливо и обратное утверждение, что доказывает следующая се- рия задач. 620. Пусть число тп делится на квадрат простого числа р и a"1-1 = l(modm). Доказать, что сгр-1 = l(modp2). 621. Верно ли, что (р 4- 1)р-1 = l(modp2) (где р — простое число)? 622. Доказать, что число Кармайкла свободно от квадратов. 623. Пусть тп — число Кармайкла, р — простой делитель тп. Доказать, что m — 1 : р — 1. Утверждения задач 619, 622, 623 показывают, что справедлива Теорема. Число m является числом Кармайкла тогда и только тогда, когда т — свободное от квадратов составное число, обладающее тем свойством, что т — 1 делится нар — 1 для произвольного простого числа р, делящего т.
93 624. Составьте алгоритм и напишите программу нахождения чисел Кармайкла. 8.4. Формула обращения Мёбиуса В этом параграфе будем считать, что мультипликативная функция определяется соотношениями: /(1) = 1; (n, m) = 1 => f{nm) = /(n)/(m). 625. Мультипликативная функция однозначно определяется своими значениями на степенях простых чисел. Доказать. Функция Мёбиуса д(п) вводится следующим образом. л(1) = 1; если п делится на квадрат простого числа, то ц(п) = 0: если п свободно от квадратов и представимо в виде произведения з раз- личных простых чисел, то д(п) = ( —1)С 626. Вычислить д(п) для n = 1, 2,..., 12. Определим функции Е(п), I(n), J(n): E(n) = 1 Vn; I(n) = n Vn; J(n) = ( еСЛИ П = J’ v v v [ 0, если n > 1. Мультипликативность этих функций очевидна. Произведением Дирихле функций f(ri) и g(ri) называется функция d|n 627. Доказать коммутативность и ассоциативность операции о. 628. Доказать, что из мультипликативности функций /(п) и g(n) сле- дует мультипликативность fog (п). 629. 1) Проверить, что J о f = f для любой функции 2) Пусть /(п) — мультипликативная функция. Доказать, что существу- ет такая мультипликативная функция f'(n), что f о f = J. Замечание. Последние три задачи показывают, что мультиплика- тивные функции образуют коммутативную группу с единичным элемен- том J и произведением Дирихле в качестве групповой операции. 630. Доказать, что из мультипликативности функций /(п) и f о д(п) следует мультипликативность д(п).
94 Глава 8. Дополнительные задачи Функция F = f о Е называется сумматорной функцией для Таким образом, F(n) = J2d|n/(d), где суммирование ведется по всем де- лителям числа п (включая 1 и п). 631. Доказать, что /(п) — мультипликативная функция тогда и только тогда, когда мультипликативна её сумматорная функция F(n). т(п) — число (натуральных) делителей числа n, s(n) — их сумма. 632. Проверить, что Е о Е = тД о Е = s. 633. Доказать, что ц о Е = J. 634. Найти ц о т и ц о s. 635. Доказать, что tpoE = I, т.е. что сумматорная функция для функ- ции Эйлера имеет вид: 52 99(d) = п. d\n 636. С помощью задач 635 и 631 доказать мультипликативность функ- ции Эйлера. 637. [Формула обращения Мёбиуса.] Доказать, что F = foE <=> f = Fop. Таким образом, найдено выражение функции f(n) через её сумматорную функцию: d\n 638. С помощью задач 635 и 637 доказать: ^(п) = п52^ = пп(1-р\ d|n р\п 639. Убедившись в справедливости равенства JJd = nT(n)/2, d\n найти сумматорную функцию для натурального логарифма.
95 640. Пусть A(n) = ( еС™ " = pt' ГДе Р - ПрОСТОе ’к 6 "• [О в противном случае. Доказать: 1) Л о Е = In; 2) Л = 1под. На множестве (линейных) последовательностей длины п, элементы ко- торых принимают значения из некоторого конечного множества, зададим отношение эквивалентности (®1! • • • ! dn) (flii O'i + l 1 • • • j ; • • • ; 1) — !>•••> • Класс эквивалентности назовем циклической последовательностью. Подсчитать число различных циклических последовательностей не так просто, поскольку в разных классах эквивалентности может быть разное число (линейных) последовательностей; например, постоянная цикличе- ская последовательность порождается одной линейной последовательно- стью, а циклической последовательности из п попарно различных элемен- тов соответствует п линейных последовательностей. Период циклической последовательности (а1;..., an) — наименьшее число d такое, что ai+a = сц для всех i, где сложение ведется по модулю п (т.е. если i + d > п, то i + d заменяется на i + d — n). 641. Пусть M(d) — количество циклических последовательностей дли- ны и периода d, элементы которых могут принимать к различных значе- ний. Доказать, что kn = Y^d\ndM(d). 642. С помощью задач 641 и 637 найти количество циклических после- довательностей длины и периода п. 643. Доказать, что общее количество циклических последовательно- стей длины п равно T(n) = 52 wo = ^52^^^ d\n d\n 644. Составляются ожерелья из бусин трех цветов. Каждое ожерелье состоит из 1) 5; 2) 6; 3) 7; 4) 8 бусин. Не будем различать ожерелья, полу- чающиеся друг из друга поворотом в плоскости. Пользуясь результатом предыдущей задачи, найти число различных ожерелий.
96 Глава 8. Дополнительные задачи 8.5. Бинарные операции и отношения 645. Пусть на множестве S определена коммутативная и ассоциатив- ная операция *. Имеется разбиение S на множества Т и U (т.е. Т U U = S; Т A U = ф), обладающее свойством: Vti, ^2; ^3 £ Т t\ * ^2 * ^3 Т' V^i, U2; 6 U Ui * U<2 * u3 E U. Доказать, что по крайней мере одно из множеств Т или U замкнуто отно- сительно операции *. 646. На множестве X задана бинарная операция * такая, что для лю- бых х, у, z Е X выполнено равенство х * (у * г) = у * (г * х), а из равенства х * у = х * z следует у = z. Доказать, что операция * является коммута- тивной и ассоциативной. 647. На множестве действительных чисел определена бинарная опера- ция *, обладающая свойствами: для любых х, у, z х*(х*х) = x;x*(y*z) = (х *у) + z. Найти все такие бинарные операции. 648. На множестве действительных чисел определена бинарная опера- ция *, обладающая свойством: для любых х, у, z, t (x*y)z+y(z*t) = (xz)* (y*t). Доказать, что для найдётся такая постоянная С, что х*у = С(х — у). 649. На множестве X задана бинарная операция * такая, что для лю- бых х, у Е X выполнено равенство (х*у)*х = у. Доказать, что имеет место и равенство х * (у * х) = у. Разрешить относительно х уравнения х * a = b и a * х = b. Пусть к тому же X — конечное множество, и его мощность не кратна трем. Доказать, что в X найдётся идемпотентный элемент, т.е Эх Е X х * х = х. 650. Бинарная операция на множестве целых чисел п * тп = — п — тп коммутативна и не ассоциативна. При этом имеют место тождества (п * тп) * тп = п, тп * (тп * п) = п. Пусть в произвольной алгебраической структуре < X, * > Vx, у Е X (х * у) * у = х, у * (у * х) = х. Доказать, что операция * является коммутативной. 651. Пусть S — множество упорядоченных троек (а, Ь, с) различных элементов конечного множества А. Выполняются следующие условия:
97 1. (a, b, с) E S 4=> (b,c,a) E S', 2. (a, b,c) E S 4=> (c, b,a) (£ S; 3. (a, b, c), (c, d, a) E S <=> (&, c, d), (d, a, b) E S. Доказать, что существует такая функция д : А —> R, что из двойного неравенства д(а) < д(Ь) < д(с) следует: (а, Ь, с) 6 S. 8. 6. Разные комбинаторные задачи 652. Предположим, что сейчас в России имеется ПО млн телефонов. Можно ли ввести 10-значные номера таким образом, чтобы ошибка в на- боре одной цифры обнаруживалась и исправлялась? (Например, если име- ется номер 240-877-2082, а по ошибке набрано 240-872-2082, то все суще- ствующие номера, кроме первого указанного, должны отличаться от на- бранного номера не менее, чем двумя цифрами). 653. В квадрате отметили 10 точек и соединили их непересекающи- мися отрезками друг с другом и с вершинами квадрата так, что квадрат разбился на треугольники. Сколько получилось треугольников? 654. Выпуклый n-угольник разбит на треугольники диагоналями, не пересекающимися внутри n-угольника. Сколько получится треугольников и сколько диагоналей? 655. На какое количество частей делят плоскость п прямых общего положения (т.е. среди них нет параллельных и никакие три прямые не пересекаются в одной точке)? 656. На какое наибольшее количество частей могут разделить про- странство п плоскостей? 657. На какое наибольшее количество частей могут разделить плос- кость п окружностей? 658. На какое наибольшее количество частей могут разделить про- странство п сфер? 659. Сколькими способами можно из цифр 2, 7, 9 составить три таких n-значных числа, что их сумма равна 2(10™ — 1)? 660. Сколько существует квадратов с вершинами в узлах квадратной сетки (п — 1) х (п — 1) клеток?
98 Глава 8. Дополнительные задачи 661. Сколькими способами можно расположить в ряд к\ единиц, /с2 двоек, ..., кп чисел п так, чтобы любому числу (к + 1) предшествовало по крайней мере одно число к? 662. Сколько существует перестановок (с повторениями) 2п чисел 1,1, 2, 2,..., п, п таких, что для любого i все числа, расположенные в перестановке между двумя вхождениями числа г, больше г? 663. Сколько существует перестановок чисел 1, 2,..., п, в которых для любого числа г, стоящего не на первом месте, хотя бы одно из чисел i — 1 или i + 1 стоит левее г? 664. На правой чашке весов лежит груз массой 11111 г. Весовщик последовательно раскладывает по чашкам гири в 1, 2, 4, 8,... г. В какой-то момент весы оказались в равновесии. На какую чашу весов попала гиря в 16 г? 665. Некоторые из подмножеств n-элементного множества М — ’’осо- бые”. Найти наименьшее возможное число особых подмножеств, если из- вестно, что любое подмножество М представимо в виде пересечения осо- бых. 666. Доказать, что среди п4-2 подмножеств n-элементного множества найдутся два, не сравнимых по отношению включения (т.е. ни одно из этих двух множеств не является подмножеством другого). 667. Какое наибольшее число подмножеств n-элементного множества можно указать таких, что любые два из них не сравнимы по отношению включения? 668. Среди 250 сотрудников международной фирмы в любой паре со- трудников каждый знает язык, который не знает другой сотрудник из этой пары. Какое наименьшее возможное число языков знают (в совокупности) сотрудники фирмы? 669. При фиксированном чётном числе к > 4 найти наименьшее п, при котором в п-элементном множестве можно выделить такие к подмножеств, среди которых нет одинаковых и взаимно дополнительных, что каждый элемент исходного множества входит ровно в половину этих подмножеств. 670. Какое наибольшее число тупоугольных треугольников можно со- ставить из 16 различных отрезков, если отношение длин самого длинного отрезка и самого короткого не больше 2?
99 671. Найти число умножений при вычислении определителя n-го по- рядка с помощью рекурсивной процедуры разложения по элементам столб- ца. 672. Найти формулу общего члена последовательности, определяемой соотношениями хп = (п - l)(zn_i + хп_2), П > 3; = 0, х2 = 1. 673. Пусть {xi, х2,..., хп} = {1, 2,..., п}. Найти как функцию от п (п > 2) наибольшее значение выражения Sn = ХГХ2 + Z2Z3 + • • • + Хп_]Хп + ХпХ1. 674. Пусть а1; а2,..., ап — различные действительные числа. Доказать, п что величина £iaii где £i = ±1, принимает не менее (п2 + п + 2)/2 раз- i=i личных значений. Показать, что эта оценка точная. 675. Пусть N — наибольшее число пар различных натуральных чисел таких, что суммы чисел в парах различны между собой и меньше нату- рального числа п. Доказать, что N < (2п — 3)/5. 676. Для строки S, состоящей из единиц и нулей, обозначим через Д(5) разность числа единиц и нулей. Например, Д(1001001) = —1. На- зовем строку S сбалансированной, если для любой подстроки Т (последо- вательных символов) S —2 < Д(Т) < 2. Так, строка 1001001 не является сбалансированной, так как содержит подстроку 00100. Найти число сба- лансированных строк длины п. 677. Дан правильный треугольник с длиной стороны п > 2. Каждая его сторона разбита на п равных частей. Через точки деления проведены прямые, параллельные сторонам. 1) На сколько маленьких треугольников (со стороной 1) разбит исход- ный треугольник? 2) Сколько получилось треугольников со стороной 2? 3) Сколько всего есть треугольников, ограниченных проведенными ли- ниями? 678. п мальчиков имеют по некоторому количеству яблок. Первый мальчик взял из своих яблок половину и ещё одно яблоко и разделил эти яблоки поровну между остальными мальчиками. Затем так же последо- вательно поступают и остальные мальчики (каждый берет половину всех
100 Глава 8. Дополнительные задачи накопившихся у него яблок и ещё одно яблоко и и делит эти яблоки по- ровну между остальными мальчиками). В результате у всех оказалось по а яблок. Известно, что все яблоки остались целыми. Каким наименьшим может быть а и сколько при этом яблок было у каждого мальчика перво- начально? 679. [Задача Иосифа2.] По кругу стоит 300 человек, пронумерован- ных последовательно по часовой стрелке числами от 1 до 300. Удаляют, начиная считать с первого человека, каждого третьего по ходу часовой стрелки до тех пор, пока не останется один человек. Каков его номер? 680. Пусть по кругу стоят тп человек, а удаляется каждый k-й. Напи- шите программу на любом известном вам языке программирования, нахо- дящую номер последнего оставшегося человека. 681. Некто, имея п монет одинаковой массы, хочет при помощи ча- шечных весов без гирь доказать окружающим, что все эти монеты весят одинаково. Какое наменыпее число взвешиваний ему придется сделать? 682. За круглым столом сидят п игроков. Каждый из них первоначаль- но имеет по одному рублю. Первый игрок передает рубль второму, после чего второй передает два рубля третьему. Затем третий игрок передает рубль четвертому, а четвертый два рубля пятому и т.д. Игроки пооче- редно передают рубль или два рубля следующему игроку, у которого ещё есть деньги; игрок, лишившийся денег, выбывает из игры и покидает стол. Найти бесконечное множество таких п, при которых игра заканчивается тем, что у некоторого игрока оказываются все п рублей. 683. В школе работает 6 кружков. Каждый из 20 учеников класса может посещать любое количество кружков — от 0 до 6. Верно ли, что обязательно найдутся такие 5 учеников и такие 2 кружка, что все пяте- ро либо посещают оба кружка, либо не посещают ни один из этих двух кружков? 8.7. Тождества 684. Доказать тождество ^-1 ~ т- 685. Пусть 1 < т < п. Доказать тождество (—^)kkmC^ = 0. 2О происхождении названия задачи см. [25].
101 686. Комбинаторными рассуждениями доказать тождество ЕГ=»(-1тс™у = о. 687. Доказать формулу для матрицы, обратной нижнетреугольной ма- трице биномиальных коэффициентов: / Со С? с} Г*1 Г*2 ° 2 °2 ° 2 а<2 °3 °3 °3 cl -1 ( eg -C? c2« -Cg -c2] c2 °3 °3 °3 \ с;° сД с2 cl • ) ((-ire; C™ ) 688. Пусть ап = ^к=оСпЬк- Доказать, что bn = ELo(-1)n кСпак- 689. Известно (см. [9]) следующее соотношение между числами Стир- линга I и II рода: п j=o ( 1, если п = к; [ 0, если п к. Пусть ап = Ylk=o k)bk- Доказать, что bn = ^2k=o s(.n’ ^ак- 690. Доказать, что число хп = (д/2 + д/З + v/5)2n+1 имеет вид хп = + bnV?> + Спл/б + dny/30, где ап, bn, сп, dn — целые числа. Найти lim an/dn. 691. Числа от 1 до 4m2 — 1 разбиты на две группы: в первую собраны числа, для каждого из которых ближайшим к нему квадратом является квадрат нечётного числа, а во вторую — все остальные. Доказать, что суммы чисел в группах совпадают. 692. Пусть п — чётное натуральное число. Запишем числа 1, 2,..., п2 в клетки квадрата п х п так, что к-я строка есть (к — l)n + 1, (к — 1)п + 2,..., (к — 1)п + п. Раскрасим клетки в чёрный и белый цвет таким образом, чтобы в каждой строке и каждом столбце чёрных и белых клеток было поровну. Доказать, что для любой такой раскраски суммы чисел на чёрных и белых клетках равны. 693. Доказать тождество = (ПРИ п > 2).
102 Глава 8. Дополнительные задачи 694. Пусть тип — взаимно простые числа, большие 1. Доказать, что (m — 1)(п — 1) 2 ’ 695. Пусть N — натуральное число. Вычислить для каждого N > 2 l<m<n<N, m+n>N, (m,n)=l 1 mn 696. Доказать: [^] ^(1 + [10ft(2rh k=l [log2 n] [log4 n] )]) = E (-i)‘(n/2‘i = E k=0 k=0 П 2 • 4fc 697. Пусть s(rr) есть расстояние между числом х и ближайшим к нему целым числом. Для произвольного натурального п вычислить 698. Пусть am,n есть коэффициент при хп в разложении многочлена (1 + х + х2')т. Докажите, что для всех целых к > 0 о < = 22(-1)ч-м < 1- г=0 699. Пусть п — натуральное число, с — действительное число. После- довательность (хк) определяется соотношениями xq = 0, Xi = 1 и Хк+2 — схк+1 - (п - k)xk к > 0. Пусть при фиксированном п число с принимает наибольшее значение, при котором xn+i = 0. Выразить хк через п и к, где 1 < к < п.
103 8.8. Две классические задачи В данном разделе на основе формулы включения-исключения решаются две классические комбинаторные задачи — задача мажордома и задача о супружеских парах. В [2] приводится три решения первой из этих задач. Вторая задача была сформулирована в 1891 г., но окончательно решена была лишь в 1934 г. 700. Задача мажордома. К обеду за круглым столом приглашены п пар враждующих рыцарей (п > 2). Требуется рассадить их так, чтобы никакие два врага не сидели рядом. Каким числом способов это можно сделать? 701. 1-я лемма Капланского. Число способов выбрать из п точек, расположенных на одной прямой, к точек так, чтобы никакие две из них не были соседними, равно ЖЧ = с„‘_1+1, 702. 2-я лемма Капланского. Число способов выбрать из п точек, расположенных на одной окружности, к точек так, чтобы никакие две из них не были соседними, равно 9М = ^кС^ 703. Задача о супружеских парах. К обеду за круглым столом приглашены п супружеских пар. Требуется рассадить их так, чтобы муж- чины и женщины чередовались и никакие двое супругов не сидели рядом. Каким числом способов это можно сделать? 8.9. Теорема Рамсея 704. По двум ящикам размещаются п предметов. Каким должно быть число п,чтобы обеспечить попадание р предметов в первый ящик или q предметов во второй ящик? 705. Доказать, что в любой компании из шести человек найдётся трое попарно знакомых или трое попарно незнакомых.
104 Глава 8. Дополнительные задачи 706. В графе К6 каждое ребро покрашено в красный или синий цвет. Доказать, что существует по крайней мере два одноцветных треугольника. 707. В графе Kg каждое ребро покрашено в красный или синий цвет. Доказать, что в графе есть два одноцветных треугольника без общих вер- шин. 708. В графе Кп каждое ребро покрашено в красный или синий цвет. Доказать, что в графе есть одноцветный треугольник. 709. Во всякой группе из 9 человек, в которой нет трёх попарно незна- комых, найдутся четверо попарно знакомых. Доказать. 710. В простом графе G 18 вершин. Доказать, что Кд С G или Кд С G. 711. Пусть = 2, an = n-an_]+l. Доказать, что в графе порядка ап+1, рёбра которого окрашены в п цветов, найдётся одноцветный треугольник. Показать, что ап = [п!е]. 712. [Теорема Рамсея.] Пусть p,q,r Е N, причем р > г, q > г. Существует число N(p, q, г) со следующим свойством. Для произвольного множества S из п элементов рассмотрим произ- вольное разбиение множества всех его г-элементных подмножеств Т на две части: Т = си U /3, аП /3 = ф. При п > N(p, q, г) • найдётся p-элементное подмножество А С S, все г-элементные подмножества которого принадлежат а, или • найдётся q-элементное подмножество В С S, все г-элементные подмножества которого принадлежат /3. Доказательство. В силу задачи 704 можно положить N(p, q, 1) = р + q — 1. Покажем, что в качестве N(r, q,r) подходит q. Действительно, если а не пусто, то любое r-элементное подмножество из а годится для роли А. Если же а = ф, то все r-элементные подмножества содержатся в /3 и В = S. Аналогично N(p, г, г) = г. Три найденных соотношения служат базой индукции. Теперь, предположив существование N(p— 1, q, г), N(p, q — 1, г) и N(p', q', г — 1) для любых р' и q', докажем, что существует N(p, 5, г) (это и будет индукционный шаг). Считая, что г > 1, р > г и q > г, положим Pi = 7V(p - 1,5, г), 5i = N(p,q - l,r), N(p,q,r) = N^p^q^r- 1) + 1. Достаточно доказать теорему для случая п = N(p, q, г).
105 Пусть xESn S'= S\ {x}. По произвольному разбиению T = ckU/З по- строим следующее разбиение множества Т' = a' U /3' всех г — 1-элементных подмножеств множества S': а' = {X | XU {z} С а},/3' = {X | X U {ж} С /3}. По предположению индукции • найдётся pi-элементное подмножество А' С S', все (г — 1)-элементные подмножества которого принадлежат а', или • найдётся Qi-элементное подмножество В С S, все (г — 1)-элементные подмножества которого принадлежат /3'. Рассмотрим подробно первый случай (для второго рассуждения аналогич- ные). Поскольку pi = Х(р — 1, q, г), • существует (р — 1)-элементное подмножество А" С S', все r-элементные подмножества которого принадлежат а (тогда множество А = A” U {ж} — искомое); либо • найдётся (/-элементное подмножество В С S', все r-элементные под- множества которого принадлежат /3 (в этом случае В и есть искомое множество). Теорема доказана. Наименьшее число N(p,q, г) из формулировки теоремы Рамсея назы- вают числом Рамсея и обозначают R(p, q, г). 713. Доказать, что R(p, q, 1) = р + q — 1. 714. Доказать, что R(r, q, г) = q, R(p, г, г) = р. 715. Доказать, что Я(3, 3, 2) = 6. 716. Доказать, что R(p, q, 2) < С^_2, где р, q > 2. Ниже рассматриваются геометрические задачи, последняя из которых решается с помощью теоремы Рамсея, а первые две имеют характер вспо- могательных лемм. Всюду будет предполагаться, что точки лежат в одной плоскости. Пусть рассматривается некоторое множество точек. Будем говорить, что это точки общего положения, если никакие три из них не лежат на одной прямой.
106 Глава 8. Дополнительные задачи 717. Из любых пяти точек общего положения можно выбрать четыре, являющиеся вершинами выпуклого четырёхугольника. Доказать. 718. Даны п точек общего положения, причем любые четыре из них являются вершинами выпуклого четырёхугольника. Доказать, что эти п точек — вершины выпуклого п-угольника. 719. [Теорема Эрдёша-Секереша.] Для любого натурального п можно указать такое натуральное число к = к(п), что из любых к точек общего положения можно выделить п точек, являющихся вершинами выпуклого п-уголъника. Доказать теорему, положив k = R(n, 5,4). 8.10. Ожерелья Данный параграф служит продолжением темы последнего примера §6.5 учебного пособия [9]. 720. Доказать, что цикловой индекс группы подстановок на множе- стве вершин правильного n-угольника, порождённых его вращениями в плоскости, есть PG(xl,x2,...,xn) = 71 d\n 721. Составляются ожерелья из щ бусинок 1-го вида, п2 бусинок 2- го вида, ..., nm бусинок m-го вида. Не считаются различными ожерелья, которые могут быть получены друг из друга вращением в плоскости. До- казать, что число различных ожерелий равно / п________(п/dY________ \n1/dy.(n2/d')\...(nm/d')r где суммирование ведется по всем числам d, одновременно являющимся делителями чисел п2,..., nm. 722. Найти цикловой индекс группы подстановок на множестве вершин правильного n-угольника, порождённых группой его симметрий.
107 8.11. Графы 723. Простой граф, изоморфный своему дополнению, называется са- модополнительным. Доказать, что число вершин самодополнительного графа при делении на 4 дает остаток 0 или 1. Найти все самодополнитель- ные графы с числом вершин меньше 6. 724. Доказать, что самодополнительный граф связен и его диаметр равен 2 или 3. 725. Диаметр графа равен 2, степень каждой вершины не больше 3. Каково максимальное число вершин графа? 726. Какие из платоновых графов являются а) эйлеровыми; б) гамиль- тоновыми? Указать в каждом случае эйлеров или гамильтонов цикл. 727. Реберный граф L(G) гамильтонов тогда и только тогда, когда в графе G есть цикл, содержащий хотя бы по одной вершине из каждого ребра G. Доказать. 728. Доказать, что граф К2п распадается на п гамильтоновых цепей. 729. Доказать, что граф K2n+i распадается на п гамильтоновых ци- клов. 730. На плоскости проведено 100 прямых общего положения. Докажи- те, что части, на которые они делят плоскость, нельзя обойти, переходя каждый раз в соседнюю (по стороне), побывав в каждой части ровно один раз и вернуться после этого в исходную часть. 731. В простом графе порядка 2п + 1 у любых п вершин есть общая смежная вершина. Доказать, что радиус графа равен единице. 732. Пусть в графе G для любых двух вершин число общих смежных вершин нечётно. Доказать, что 1) G — эйлеров граф; 2) число вершин G нечётно. 733. При каких п и k > 1 существует простой граф с п вершинами, в котором любые к вершин имеют ровно одну общую смежную вершину? 734. Найти порядок регулярного степени 6 простого графа, в котором у любых двух вершин ровно две общих смежных вершины. 735. Пусть в графе G любые две смежные вершины не имеют общих смежных вершин, а любые две несмежные вершины имеют две общие смежные вершины.
108 Глава 8. Дополнительные задачи 1) Доказать, что G — регулярный граф. 2) Пусть п — порядок графа (S', а к — степень каждой его вершины. Доказать, что n = 1 + к + С%. 3) Пусть А — матрица смежности графа G. Доказать, что если Л - собственное значение А, отличное от к, то Л2 + 2Л — к + 2 = 0. 4) Найти кратности собственных значений матрицы А. 5) Доказать, что число к представимо в виде к = 1 + т2, где тп — целое число. 6) Найти графы, удовлетворяющие условию задачи при m = 1, 2, 3.
Ответы .У казания. Решения 15. 50. 16. 2 рыцаря. 17. 1/2. 28. Пусть Р(х) — многочлен из условия задачи. Очевидно, что он чёт- ной степени. Пусть с — его наименьшее значение. Все действительные корни многочлена Q(x) = Р(х) — с имеют чётную кратность. Поэтому для некоторого всюду положительного многочлена R(x) имеем Q(x) = Т2(ж)Р(гг), где Т2(х) — произведение всех множителей многочлена вида (а? — а) (каждый множитель берется столько раз, какова кратность соот- ветствующего корня). Поскольку R(x) имеет степень меньше, чем Р(х), можно воспользоваться предположением индукции. Осталось заметить, что Р(х) = Q(x) + (у/с)2. 35. 140. 36. 70. 39. а) 10!2; б) 10!2 • 210. 41. 8! 42. 2 • 88 - 8! 44. 4п2/4. 45. а) 2П — 1; 1, 2, 22,..., г*1"1; б) (Зп - 1)/2; 1, 3, З2,..., З71"1. 46. 271-1. 47. 57599 = 2402 - 1 = 239 • 241. 51. 1 и 2. 52. Пусть d — общий делитель чисел тп и тп2 + п2. Тогда d делит и (т ± п)2. Если {m + п,тп — n) = 1, то ((m + n)2, (т — n)2) = 1, откуда d = 1. Если (т + п,т — п) = 2, то m и п нечётные числа (чётными они быть не могут, так как взаимно просты) и ((т + п)2, (т — п)2) =4. Таким образом, d — нечётный делитель четырёх, то есть d = 1. 53. 1 и 2. Используйте тождества 2m2 = (m2 + n2) + (т + ri)(m — п) и 2n2 = (m2 + n2) — (m + ri) (m — n). 54. 1) 3; 2) 57; 3) 11; 4) 1. 57. Положим Pi(x) = x, Pn+i(x) = P(Pn(x)). Тогда an = Pn(0), an = Pn_m{am) (при n > m). В силу теоремы Безу Рп(х) = Pn(0) + xQn(x) = ап + xQn(x), где Qn(x) ~ некоторый многочлен с целыми коэффициентами. По- скольку ап — Pn—mfam) — т P^mQn—m(^m) 1 ИМввМ (tin, (Zm) — (dn_m,(Zm). Из последнего соотношения следует, что (an,tzm) =
по Ответы. Указания. Решения 59. 218 • З8 • 54 • 72 • 11 • 13 • 17 • 19. 67. См. доказательство бесконечности множества простых чисел в [9]. 68. Пусть среди чисел 1, 2, 3,..., 2П содержится г < к простых чисел Рх,р2, • • • ,Рг- Тогда каждое число, не превосходящее 2П, представимо в ви- де - Ргг-> где, очевидно, каждый показатель степени st не больше п. Однако чисел такого вида (по правилу произведения) (1 + п)г, что меньше 2П. Полученное противоречие доказывает утверждение задачи. Поскольку, как известно из анализа, показательная функция ’’растет быстрее” степен- ной, и для любого (сколь угодно большого) к при достаточно больших п неравенство 2n > (l + n)fe имеет место, получено доказательство бесконеч- ности множества простых чисел. 69. Среди чисел {1, 2,..., п} имеем не более п/р2 чисел, делящихся на р2. Поэтому количество чисел, делящихся на квадрат простого числа, не больше \ п п \ п п 1 1 \ Зп р2 < 4 к(к + 1) 4 + П к к + 1 4 —2 к—2 Утверждение задачи позволяет получить не только доказательство беско- нечности множества простых чисел, но и оценку сверху для к-го простого числа Pk- Первые (по возрастанию) к — 1 простых чисел порождают 2fe-1 чисел, свободных от квадратов. Поэтому среди чисел от 1 до 4-2fc-1 = 2fe+1 содержится по меньшей мере к простых чисел (в противном случае доля чисел, свободных от квадратов, была бы менее четверти), т.е. Pk < 2fe+1. 70. 1) 1; 2) 1; 3) 5. В последнем примере имеем: Ю10 = З10 = 95 = 25 = 4(mod7); Ю100 = (Ю10)10 = 410 = (-3)10 = З10 = 4. Далее по индукции легко доказать, что V/c 1О10 = 4(mod7). 75. Поскольку 1980 = 20-99, достаточно доказать делимость на 20 и на 99. Первое очевидно. Для доказательства второго утверждения заметим, что 100m = 1 (mod 99). Поэтому 1920... 80 = 19 + 20 + ... + 80 = 0(mod99). 76. Нет. Получившееся число обязательно делится на 11. 77. Нет. Получившееся число обязательно делится на 37. 78. 8. 80. При к = 2 и 11. 81. 11-1014. Четвёртая степень целого числа при делении на 16 даёт остаток 0 или 1.
Ill 82. Да. Представив число п в виде п = т • 2г • З-7, где т не делится на 2 и на 3, будет относить его к одному из трёх подмножеств в соответствии с остатком от деления на 3 числа i — j. 84. Пусть N = qi •... qn, (ц = N/qi. Из условий задачи следует, что кратно а,. Положив i, = из второго свойства числа имеем, что для некоторого целого yi выполняется равенство aiXi = 1 + qiPi. Получили ли- нейное диофантово уравнение, в котором коэффициенты при неизвестных взаимно просты. 85. х = 86. 37. 87. По китайской теореме об остатках найдётся такое число х, что х = ri(mod(?i), х = Г2 — l(mod<?2)> ,х = гп — п + l(modgn). 89. 1) 1; 2) З102 = З100 • З2 = 9(mod 101); 3) 8900 = 828'32 • 84 = 84 = 62 = 7(mod29); 4) 1. 92. (п^2~ — l^n2^- + 1) \р. 98. 1) п = 2) п — 1 = a^Z?2 = (ар-1 — 1)а +1\ В силу малой теоремы Ферма из последнего соотношения получаем: п — 1 : р. Покажите, что п — нечётное число. Отсюда п — 1: 2р и а”-1 — 1: а2р — 1. В то же время из определения числа п следует: а2р — 1 : п. Таким образом, а”-1 — 1 делится на п. Это и требовалось доказать. 99. Поделим многочлен /(ж) на многочлен хр — х: = (хр - x)q(x) + г(х), где степень остатка г(х) не превосходит р — 1. Из малой теоремы Ферма следует, что f(x) = r(a;)(modp). 100. Доказательство можно найти в [3]. 101. Пусть q — простое число, являющееся делителем числа п. Так как 2 < q < п — 1, число q является делителем (п — 1)!. В силу задачи 49 (п — 1)! + 1 не делится на q, а, значит, и на п. 102. Пусть п — составное число. Может представиться два случая. а) п не является квадратом простого числа. Тогда п представимо в виде произведения двух различных целых чисел: п = щ п,2, каждое из которых больше 1. Заметим, что при этом п — 1 > щ и п — 1 > п^-
112 Ответы. Указания. Решения Значит, (п — 1)! содержит в качестве множителей и nj, и п2, в силу чего (п — 1)! : щ • п2 = п. б) п является квадратом простого числа: п = р2. При п > 4 выполняется неравенство п — 1 = р2 — 1 > 2р и, следовательно, (п — 1)! в числе своих множителей содержит как р, так и 2р. Таким образом, вновь (п — 1)! : п(= р2). 103. Если р = 2, то доказываемое утверждение проверяется непосред- ственно. Будем считать, что р > 2. Введем в рассмотрение многочлен f(x) = rrp-1 — 1 — (х — 1)(х — 2)...(х — (р — 1)). Легко видеть, что степень f(x) не превосходит р — 2. По малой теореме Ферма rrp-1 — 1 делится на р при х = 1,2, ...,р — 1. Тем же свойством обладает и многочлен /(я). Итак, сравнение ггр-1 — 1 — (ж — 1)(ж — 2)...(ж — (р — 1)) = O(modp) имеет степень не выше р — 2, в то время как число его решений не меньше р — 1. В силу задачи 100 все коэффициенты многочлена f(x) кратны р, в частности, его свободный член, равный — (1 + (р — 1)!). Таким образом, (р — 1)! + 1 делится на р. 106. По теореме Вильсона 1 • 2 • 3 •... • (р — 1) = — l(modp). 1) В левой части сравнения каждый чётный множитель 2k следует заме- нить на нечётное число — (р — 2k). 2) В левой части сравнения каждый нечётный множитель 2k — 1 следует заменить на чётное число — (р — 2k + 1). 107. Сначала заметим, что х2 = T/2(modp) тогда и только тогда, когда хотя бы одно из чисел х + у и х — у делится на р (поскольку р — простое число). Если не равные друг другу числа хну берутся из множества А = {1,2,.. . ,р - 1}, то 0 < |ж - у\ < р и х2 = p2(modp) <=> х + у : р. Поскольку А — приведенная система вычетов, квадрат любого числа из А сравним по модулю р с некоторым числом из того же множества А. Разо- бьем множество А на пары вида {к,р — к}. Квадраты любых двух чисел из разных пар не сравнимы по модулю р, а квадраты двух чисел из одной
113 пары сравнимы по модулю р. Таким образом, каждой паре соответству- ет свой ’’индивидуальный” квадратичный вычет, а квадратичных вычетов столько же, сколько и указанных пар, то есть (р — 1)/2. 108. Рассмотрим три возможных случая. • а кратно р. Здесь утверждение тривиально. • а — квадратичный вычет. Для некоторого числа Ь, не кратного р, имеем а = &2(modp). Тогда, используя малую теорему Ферма, полу- чим а^~ = b2(^ = bp~' = l(modp). ( p-i\2 • а — квадратичный невычет. Поскольку (аг) = ap~l = l(modp), число а^~ сравнимо по модулю pci или —1. Мы уже выяснили, что сравнению х^ = l(modp) удовлетворяет квадратичных выче- тов. В силу задачи 100 других решений у данного сравнения нет. Значит, для квадратичного невычета остается лишь возможность = — l(modp). 110. р = 4/с + 1. 111. Сначала убедимся в том, что 5 — квадратичный вычет по модулю 19: 5 19/ = 59 = 64 • 5 = (—2)2 • 5 = l(modl9). Итак, 59 = 1 (mod 19). Отсюда 510 = 5 (mod 19) и х = ±55 = ±9 (mod 19). Аналогично решаются сравнения вида х2 = a(modp), где р — простое число вида р = 4=к + 3. 112. Убедитесь в том, что когда число к пробегает по F, то и число ч акек пробегает по Р. Пусть К = П к. Тогда fc=i д К = Ц акек к=1 = aqK К JJ^(modp). fc=l W' k=l / \ 9 Сократив на К, получим I - 1 Д Ek = l(modp), откуда и вытекает требу- 'р' k=i емое.
114 Ответы. Указания. Решения 113. р = ±l(mod5). Примените лемму Гаусса, перебирая возможные остатки от деления р на 10. 114. 1) Решений нет; 2) х = — 1 ± 7t, у = 2 — 13i, t 6 Z; 3) х = -7 + 19i, у = 8 - 21i; 4) x = 56 + 171i, у = 653 + 1994i. 115. ab. Решение. Если m = ab, to x : b, у : a и ax ± by > 2ab. Пусть теперь m > ab. Общее решение уравнения ax ± by = m в целых числах: x = xo — bt, у = yo + at, где axo ± byo = тп. Покажем, что найдётся такое целое число t, что х > 0 и у > 0, то есть bt < х0, at > —у0, или —уо/a < t < Xq/Ь. Выразив yQ через ж0, имеем у — < t < Осталось заметить, что интервал (^ — имеет длину, большую 1, и значит, содержит целую точку. 116. (а — 1)(Ь — 1)/2. Решение. Все натуральные числа > ab, представимы указанным образом. Проверьте, что для каждого из чисел 0, l,...,ab — 1 указанное представление единственно (если оно существует). Рассмотрим на координатной плоскости прямоугольник с вершинами 0(0,0), А(Ь, 0), С(Ь, а), В(0,а). Пусть ах ± by < pq. Тогда точка (х, у) лежит в прямоугольнике О АВС ниже диагонали АВ. Несложный подсчет: всего прямоугольник содержит (а ± 1)(Ь+ 1) точек с целыми координатами, на диагонали АВ точек, отличных от А и В, нет (в силу взаимной простоты а и Ь), стало быть, ниже диагонали расположено (о+1)(ь+1)-2 точек Каждой точке соответствует число ах + by < ab (и разным точкам — разные числа). Значит, чисел, не представимых в виде ах + by с неотрицательными х и у, будет ab — аЬ+а+ь~1 = (а~1Хь~1). 117. 5 коров, 1 свинью, 94 овцы. 118. 15 коров, 7 свиней, 78 овец. 119. Андрей женат на Ольге, Иван — на Анне, Степан — на Екатерине. 122. 1) (0,0), (2, 2), (1,2), (2,1), (1,0), (0,1); 2) (0, 0), (±1010, ±10), (±100, ±100), (±6666, ±2). 123. 1) (498, 496), (78, 64); 2) m = 2, п = 4; 3) (m,n) = (3,5), (4, 7); 4) решений нет; 5) (3,1);6) (1,1,1), (4, 2, 2); 7) (13,2); 8) (3,1,2). Подроб- ное решение шестого уравнения приведено в первом издании настоящего сборника. См. также журналы ’’Квант”, 1998, №4, С.50 и ’’Математика в школе”, 1999, №3, С.60. 124. (7, 3, 3), (8, 5, 2), (13,4, 2). 125. 5, 49, 97. 126. Воспользуйтесь результатом задачи 93. 127. (-300,-2), (-36,-6), (24, 4). 128. 17. 129. 2000 и 3998.
115 130. 12 х 5 или 8x6. 131. а) нет; б) да; в) нет. Указание. Пусть х (у) — количество строк (столбцов), в которых знак менялся нечётное число раз. Тогда количество минусов в таблице равно гг (100 — у) + г/(100 — т). 132. 2, 8, 272, 1898. 133. Докажите, что знаменатель прогрессии — рациональное число. Контрпример: bi = 2т~г. 134. Для представления числа т/п указанной дробью ищите а, Ь, с и d в виде произведения степеней тип. 135. Если квадрат а2 и куб Ь3 совпадают, то утверждение задачи триви- ально. Рассмотрим различные случаи взаимного расположения квадрата и куба в прогрессии. I. Пусть а2 < Ь3. 1) Рассмотрим случай, когда b делится на а. Положив b = ка, будем искать натуральное х, для которого а2 + х(Ь3 — а2) = а2 + х(к3а3 — а2) = у6. Удобно положить, что у = at. Тогда х = Видно, что если t = к2, то х е N. Итак, при b = ка прогрессия содержит член (аА;2)6. Например, если прогрессия содержит 22 и 63, то в ней будет и член, равный 186. 2) Пусть а и b — взаимно простые числа. Сведем этот случай к преды- дущему, найдя в прогрессии число z3 такое, что z кратно а. Имеем а2 + x(b3 — а2) = z3, z3 — a2 z3 — Ь3 Ь3 — а2 Ь3 — а2 Пусть z = b + t(b3 — а2). Тогда z3 — b3 :z — b:b3 — а2 и х Е N. Подберем t так, чтобы z = b + tb3 — ta2 делилось на а. Для этого необходимо и достаточно, чтобы для некоторого у выполнялось равенство b + tb3 = уа, или ay — b3t = b. (1) Поскольку а и Ь3 взаимно простые числа, уравнение (1) разрешимо в на- туральных числах. Пример. Если а = 2, b = 5, из уравнения 2у — 125А = 5 найдём t = 1,у = 65, z = 5 + 1 • (53 — 22) = 126. Таким образом, вместе с 22 и 53
116 Ответы. Указания. Решения прогрессия содержит и 1263. Используя далее результаты предыдущего пункта, обнаруживаем в прогрессии член (632 • 2)6 (хотя можно найти в этом конкретном примере и меньшую шестую степень: 486). 3) Рассмотрим, наконец, общий случай. Пусть Pi,P2, • • • — все простые делители чисел а и Ь, a = Ц;л. Ь = П₽‘‘, d = №‘,«2) = Заметим, что если 3Aij = 2mj = Sj, то : 6 и все числа вида a2 + х(Ь3 — а2) делятся на (р^6)6. Поэтому вместо чисел а и b можно рассматривать числа а/p™' и Ь/р1?. Таким образом, без ограничения общности можно считать, что для любого г ровно одно из чисел b3/d и a2 /d делится на pt, отсюда ___«2______________________________J.3___________________________«2 d не делится на pi и число с = , взаимно просто с а. Пусть д = Пр^ — наименьшее натуральное число, куб которого де- лится на d. Легко убедиться, что для любого i выполняется неравенство ki > Si. Значит, b кратно д. Положим b = gb\. Найдём член прогрессии г3 с z, делящимся на а. Исходя из равенств ____________________________ J)^ b3 + х(Ь3 - а2) = z3, х = —_2 , Ь6 — ст будем искать z в виде z = gzi. Тогда = g3(z3 - Ь3) = д^_ z3-b3 d(b3/d — а2/d) d с Поскольку д3 : d, для того, чтобы х Е N, число zi можно определить как Zi = bi + ct. Если положить z\ = ay, то будем иметь уравнение ay — ct = bi. (2) Как уже было отмечено, коэффициенты а и с взаимно просты (и положи- тельны), поэтому уравнение (2) разрешимо в натуральных числах. По у находим z = дау: а такое, что z3 — член прогрессии. Вновь задача сведена к случаю 1). Пример. При а = 480, b = 72 имеем d = (b3, а2) = 29-32, д = 23-3, bi = 3, 243 z3 - З3 z3 - З3 ______. _t______= з . _1 • 29 • З2 З4 - 50_31 ’
117 Z\ = 3 + 31i = 480г/, 480?/ — 31i = 3. Одно из решений последнего уравнения у = 25, t = 387. Отсюда z = 24 • 480 • 25 = 600 • 480, и прогрессия содержит член (480 • 360000)6. II. Пусть Ь3 < а2. Будем искать в прогрессии куб, больший, чем а2 : b3 + x(a2-b3)=z3, z3 - b3 а2 — Ь3 Достаточно положить z = b+t(a2—63), где t нужно выбрать столь большим, чтобы z3 было больше а2. Таким образом, задача сведена к случаю I. 138. 20, 16, 100, 400000. 139. 96. 143. Числа, взаимно простые с п, разбейте на пары вида < к, п — к > . 144. Если бы числами pi,...,ps исчерпывался весь список простых чисел, то ни одно число, кроме 1, не было бы взаимно просто с их произ- ведением Р = pi ... ps, откуда tp(P) = 1, что противоречит формуле из условия задачи. 150. Пусть l = di<...<dk=n — все делители числа п. Количе- ство чисел из множества {1, 2,... ,п}, которые делятся на ф, равно n/di- Поэтому количество чисел, не превосходящих п и не являющихся взаимно простыми с п, не больше ^ + ^ • • • + ^ = <%к—х + <4-2 + + di = s(n) — п. Таким образом, п — ip(ri) < s(n) — п, что и требовалось доказать. 151. Используя обозначения из решения предыдущей задачи, имеем /с к п , лц , Ei х—' п 1 /-> / dx х . di = 2 ~т — п / ~ < П(1 + / —) = n(lnn + 1). • , . Щ I J] X 1=1 1=1 1=1 С другой стороны, s(n!) > n! 1/г. Множитель при п! может быть сколь угодно большим при достаточно большом п (как частичная сумма гармонического ряда). Поэтому ответ на вопрос задачи отрицательный. 154. Нет. 157. Произведение матриц AjAj. 173. Решение можно найти в [63]. 174. (1 — х)(1 + х + х2 + ... + жп-1) = 1. 177. Нулевой гомоморфизм и тождественное отображение. 179. Да. 180. Все участвовали. 181. Мать и младшая дочь. 182. Нет.
118 Ответы. Указания. Решения 183. Эта дорога ведёт в ваш родной город? Положительный (отрица- тельный) ответ означает, что указанная дорога ведёт в А (соответственно в В). Решение. Пусть х означает высказывание ’’встреченный путешествен- ником житель правдив”, а у означает высказывание ’’некоторая фиксиро- ванная (конкретная) дорога ведёт в А". Для того, чтобы составить вопрос, положительный (отрицательный) ответ /(х, у) на который означает, что указанная дорога ведёт в А (соответственно в В), составим таблицу истин- ности. X У Ожидаемый ответ f{x,y) 1 1 Да 1 1 0 Нет 0 0 1 Нет 0 0 0 Да 1 Из таблицы видно, что /(х, у) = х ~ у. Таким образом, нужно выяснить соответствие направления дороги и места жительства ’’аборигена”. 187. 22”-1. 188 . 22"-1. 189. ж, ж, y,y,z,z,xyV yzV zx,x 7/V у ZV Z X. 191. Если все переменные принимают значение ИСТИНА, то любая их комбинация, содержащая из связок лишь &, V или —истинна. Таким образом, отрицание не выражается через указанные логические операции. 193. х = х + 1; х У у = х • у = (х + 1) (у + 1) = (ж + 1) (у + 1) + 1 = ху + х + у; х у = х А у = (х + 1) А у = х + 1 + у + (х + 1)у = х + ху + 1. 194. 1, х, у, ху, 16 многочленов (0,1,..., 1 + х + у + ху) соответствуют всевозможным подмножествам 4-элементного множества одночленов. 195. 1) х + у + z + ху + xz + yz + xyz:, 2) ху + xz + yz\ 3) х + у + z. 196. Существование вытекает из 193. Приведем также явную конструк- цию, позволяющую получить многочлен от п переменных, если известно представление в виде многочлена любой функции от п — 1 переменных: f(xi, ...,хп) = /(О, х2, • • •, хп) + Х1 (/(О, х2,.. , хп) + /(1, х2,..., хп)). Проверьте! Единственность докажем двумя способами.
119 Комбинаторное доказательство. Из 2” одночленов от п перемен- ных можно составить 22" многочленов. Столько же имеется и различных логических функций от п переменных (поскольку таково число различ- ных таблиц истинности функции от п переменных). Так как любая функ- ция представима многочленом и число функций равно числу многочленов, каждая функция представима ровно одним многочленом. Доказательство по индукции. Индукция проводится по числу пе- ременных. База индукции (число переменных равно нулю) очевидна. Ин- дукционный шаг. Пусть для функций от п — 1 переменных единствен- ность представления многочленом установлена. Предположим, что неко- торая логическая функция от п переменных представима двумя много- членами: /i(xi,..., хп) и f2(xi,..., хп). Пусть f = /1 + /2. Из предположе- ния следует, что f — многочлен, при всех значениях переменных равный нулю. Нужно доказать, что f — пустой многочлен (не содержит непу- стых одночленов), поскольку это равносильно совпадению многочленов j\ и /2. Сгруппировав слагаемые, содержащие ад, получим /(ад,... ,хп) = Xiglxz,..., хп) + Л(а;2,..., хп). Подставив ад = 0, имеем h = 0. Подставив теперь ад = 1, имеем g + h = 0, откуда g = 0. Таким образом, задача сведена к многочленам от п — 1 переменных. 197. 1), 3), 4), 5), 6), 8). Указание. 2) а; Vу У z = (xVy)Vz; 7) функция х + у входит в указанное множество, а функция х + у + z не входит. 198. Все дело в штрихе Шеффера. 199. 1), 2), 6). 204. б) Покажите, что Ро С М U S. 212. Пусть высказывание а(&, с, с/, е) означает: ”А (соответственно B,C^D^E) присутствует”. Тогда заданным условиям отвечают следующие сложные высказывания: 1) $1 = tz&e -д & = ё V ё V /а: 2) s2 = ё > b&c = eV be; 3) s3 = tz&c V ё&ё = (a V c)(a V с) = ас V ac; 4) S4 = e d = ё V d; 5) s5 = (c —> (b —> e))&(&&c —> e&d) = (eV 6 V e) (& V с V ed) = = b V (с V e) (c&ed) = b V ced V её. Каждый вечер (с точки зрения присутствия на нем А, В, С, D, Е) мож- но описать совершенной элементарной конъюнкцией aai ba- саз dai еаъ, где — логические переменные, не обращающиеся одновременно в 0. Таким об-
120 Ответы. Указания. Решения разом, задача сводится к нахождению СДНФ для формулы f = S1&S2&S3&S4&S5. Исходя из высказывания S3, рассмотрим два возможных случая. • Пусть a = 1. Тогда с = 0, Si = е V b, s2 = е У b, s5 = b V е и f = (ё V Ь) (е V &) (ё V d) (b V е) = Ъ(е V d). • Пусть a = 0. Тогда с = 1, Si = 1, s2 = е, s5 = b V ed и f = е(ё V d)(bV ed) = ed(b V ed) = ebd. Значит, f = acb(e V d) V acebd = abcde V abcde V abcde V abcde. Ответ. 4 вечера: A, В, E\ A,B,D; A,B; B,C,E. 213. См. рис. 4, a). 214. См. рис. 4, 6). 215. См. рис. 4, в). 216. См. рис. 4, г). Рис.4. Схемы с минимальным числом контактов 220. АА = АА + О = АА + АА = А(А + А) = А I = А; А + А = (А + А) I = (А + А)(А + А) = А + АА = А + О = А. 221. А + 1= (А+ 1)1 = (Л + 1)(Л + Л) = А + 1-А = А + А = 1. 222. А + О = А + АА = А I + А-А = A(I+ А) = А I = А. 224. Обозначим X = (Л + В) + С, Y = А + (В + С). Нужно доказать, что X = Y. Имеем: АХ = Л((Л + В) + С) = Л(Л + В) + АС = Л + АС = Л; AY = А(А+(В + С)) = А + А(В_+С) = Л; _ _ _ АХ = Л((Л + В) + С) = _(ЛЛ +_АВ) + АС =^АВ + АС = А(В + С); AY = Л (Л + (В + С))_= ЛЛ_+ Л (В + С) = Л (В + С). Итак, АХ = ЛУ, АХ = AY. Поэтому X = (Л + А)Х = АХ + АХ = AY+ AY = (А + Л)У = У. Доказана ассоциативность сложения. Из закона двойственности теперь следует ассоциативность умножения.
121 238. Независимость аксиомы А от В и С будем доказывать указанием интерперетации, при которой аксиомы В и С и все выводимые из них формулы принимают одно значение, а формула А не всегда имеет это значение. Независимость А1 от А2 и АЗ. Будем считать, что каждая формула имеет одно из значений 0, 1 или 2, и при этом логические связки —> и ~ задаются следующими таблицами. В А 0 А 1 2 В А А 0 1 1 1 0 1 1 0 1 0 1 0 2 0 1 1 2 1 Проверьте, что при любых значениях переменных формулы А2 и АЗ при- нимают значение 1, а из формул, принимающих значение 1, по правилу modus ponens выводится формула, имеющая также значение 1. В то же время при А = 2 и В = 1 формула А1 имеет значение 0. Независимость А2 от А1 и АЗ. Будем считать, что каждая формула имеет одно из значений 0, 1 или 2, и при этом логические связки —> и “ задаются следующими таблицами. В А 0 А 1 2 В А А 0 1 1 1 0 1 1 0 1 1 1 0 2 2 1 1 2 0 Проверьте, что при любых значениях переменных формулы А1 и АЗ при- нимают значение 1, а из формул, принимающих значение 1, по правилу modus ponens выводится формула, имеющая также значение 1. В то же время при А = 1, В = 2, С = 0 формула А2 имеет значение 0. Независимость АЗ от А1 и А2. Будем считать, что каждая формула имеет одно из значений 0 или 1, и при этом логические связки —> и ~ задаются следующими таблицами. в А 0 1 А^В 0 1 1 1 0 1 А А 0 1 0 0
122 Ответы. Указания. Решения Проверьте, что при любых значениях переменных формулы А1 и А2 при- нимают значение 1, а из формул, принимающих значение 1, по правилу modus ponens выводится формула, имеющая также значение 1. В то же время при А = 1 и В = 0 формула АЗ имеет значение 0. 250. Нет. К аксиомам исчисления предикатов можно добавить без про- тиворечия невыводимую в нем формулу (Зх А(х)) —> (\/х А(х)). 253. 6) min(x, у) = х— (х—у)-, 7) тах(х, у) = у + (х—у). 255. 1) [х/у] = f>g(rr-z?/)-l; г=0 2) rest(a?,?/) = х-[х/у] - у- 3) div(z, у) = sgrest(x, у); 4) Pr(x) = sg(rr-l) • sg ( 52 div(z, г)—2) J ; 6) q(x) = 52 sg(z + 1—г2) — 1. \г=0 / г=0 259. Пусть д(х{, ...,хп) = . ,хп,у) = 0] < /г(ац,... ,хп), где h — примитивно рекурсивная функция.Тогда h(xi,...,xn) / i д(хх,...,хп) = sg I i=0 \?/=0 260. p(ri) = ц[|тг(т/) — n\ = 0]. Ограниченность сверху примитивно ре- курсивной функцией р(п) < 2п+2 установлена в решении задачи 69. 261. О'!(а;) = 2(х + 1); o^fx) = 2ж+2 • х + 2ж+3 — 2. 268. Пусть А — непустое примитивно рекурсивное множество, a - некоторый его элемент. Положим = х /л(ж) + a sgfA(x). Покажите, что множество значений функции д совпадает с А. 278. 6) (210 + С15о)/2. 280. 1) (7ф Каждая точка пересечения хорд однозначно задается (неупорядоченной) четверкой точек — концов этих хорд. 2) 1 + С2 + С^. Будем последовательно проводить хорды. Каждая новая хорда увеличивает количество частей на величину, равную количеству от- резков, на которые хорда разбивается ранее проведёнными хордами, т.е. на единицу больше, чем число её точек пересечения с ранее проведёнными
123 хордами. Поскольку при этом каждая точка пересечения считается ровно один раз, количество частей, первоначально равное единице, после про- ведения всех хорд увеличится на сумму числа точек пересечения и числа хорд. Интересен ответ к задаче при п = 1,2,..., 6. Он таков: 1, 2, 4, 8, 16, 31. Физик из известного анекдота на основании первых пяти результа- тов заявил бы, что общий ответ — 2ге-1, а число 31 возникло в результате погрешности эксперимента. 282. п = 10, к = 5. 283. 26^ = 70. 286. 1)Ср; 2)Cf = 126; 3)С|° - С53Ср - 5 = 10431. 287. 5151. 288. Левая часть тождества — это число наборов i0, ..., in, удо- влетворяющих неравенствам т > in > in_i > ii > i0 > 1. Пусть x0 = ?0 lj 2>1 — ^0, > ^n— 1j ^n+1 — ^n- Тогда 27g, X\, . . . , 27ra_|_i - неотрицательные целые числа, чья сумма равна т — 1. Число наборов Оо, х1}..., zn+1) равно = С^. 289. Пусть Ак = {1, 2, 3,..., кп — 1). Если а = х™ < кп, где Xi - неотрицательные целые числа, то Vz Xi < к — 1. 1) Из п натуральных чисел, каждое из которых не превосходит (к — 1), можно составить (к — 1)п упорядоченных выборок. Поэтому в множестве Ak по крайней мере кп — 1 — (к — 1)п чисел не представимо в виде х™. Число кп — 1 — (к — 1)п может быть сколь угодно большим за счет выбора числа к. 2) Можно считать, что речь идет о представлении числа в виде суммы ровно п слагаемых если допустить, что некоторые из них могут быть равны нулю. В указанной сумме каждое из п слагаемых принимает одно из к значений: 0,1, 2П, Зп,..., (к — 1)п. Поэтому можно образовать не более С£ = С^+п_1 таких сумм. Заметим, что откуда кп _ z-»n кп~1(к °fc+n-l к щ к + 1 2 Значит, не менее кп 1 • ^-А- чисел не представимы в указанном виде. 290. 1) (С2+1)2; 2) п(п + 1)(2п + 1)/6. 292. С^. 294. Составьте отношение ак+\/ак.
124 Ответы. Указания. Решения 295. Покажите, что число равнобедренных треугольников с фиксиро- ванным основанием не больше 3. Поэтому число треугольников, не явля- ющихся разносторонними, не превосходит С±2 • 3 = 198. 296. 52!/(13!)4. 299. 1) Подсчитайте двумя способами сумму мощностей всех подмно- жеств n-элементного множества. 306. (n — I)3 при нечётном n; (n — I)3 + 1 при чётном п. 307. С27 — 6 • Cf7 + Cg • Cl- Решение. Пусть U — множество последова- тельностей (а1; а2, , ^б), составленных из шести неотрицательных целых чисел с суммой 27; а для каждого i множество А/ С U состоит из таких последовательностей, в которых щ > 10. Для решения задачи нужно вы- числить | П Ai\. Заметим, что \U\ = Cg7, |Д| = Cg7, |AjП А,| = Cl (i j), а пересечение двух и большего числа множеств Д пусто. Замечание. Как отменно в [9], ответ к данной задаче даёт число счастливых 6-значных билетов (в которых сумма первых трёх цифр равна сумме последних трёх цифр). 310. Используйте оценку остаточного члена ряда из признака Лейбни- ца. 311. 14833. 314. 6) (k - I)2 = 2!С^ - 1!С£ + 1. 315. М£ = D£ = 1. 317. fn. 318. 7) Можно доказать индукцией по т (или п). Существует также комбинаторная трактовка этого тождества, связанная с ’’задачей о пры- гуне”. 319. О, 1, 12. 320. Примените индукцию. 325. 1) С помощью соотношения fn+k = fn-ifk + fnfk+i проверьте ма- тричное тождество (fk—1 fk ] | fn fm—k \ | fn+k fm | 1 0 / \ fn+1 fm—k+1 / \ fn fm—k J Теперь вычисление соответствующих определителей даёт желаемый ре- зультат. 2) Сложите тождества из п. 1) при т = п, к = 1 и т = п,к = 2. 335. 2иа/5/п = (1 + л/5)га - (1 - 336. 1) Используйте результат предыдущей задачи, а также следующие соотношения:
125 2Р 1 = l(modp); С* : р при 1 < к < р. 2) Следствие 1). 337. Используйте задачу 323. 338. а) Да. Если условие задачи выполняется для последовательности ..., то это же будет верно и для последовательности 1 1 1 1 П1П2 ’ ni (ni +пг) ’ 712(711+712) ’ ’ Пк(П1+П2) ’ б) Нет. Покажите, что если бесконечная последовательность У-, ... удовлетворяет условию задачи, то Vn 6 N ^2п+2 — < f2n+l hn+i An Переходя к пределу в двойном неравенстве по п, отсюда получаем,что 339. /(ж) = кх, где к = 340. fn - n-е число Фибоначчи. Решение. Обозначим число искомых подмножеств fn. Выпишем минимальные э-множества при п < 6: {1}, {2, 3}, {2,4}, {2, 5}, {2, 6}, {3, 4, 5}, {3,4, 6}, {3, 5,6}. Прямой подсчет показывает: А = А = 1,А = 2, /4 = 3, А = 5, А = 8. Возникает предположение, что для для любого натурального п fn+2 = fn+l+ fn- Убедимся в этом. Разобьем все мэ-подмножества множества {1,2, ..., тг + 2} на два клас- са: множеств, содержащих число п + 2, и множеств, не содержащих это число. Всякое множество из второго класса является мэ-подмножеством множества {1,2,...,п + 1}, поэтому во втором классе A+i множеств. Заметим, что в минимальном э-множестве, содержащем к элементов, не могут присутствовать числа меньше к (из-за минимальности), и так как множество — эгоцентрично, то число к — его элемент. Легко проверить, что верно и обратное: произвольное ^-элементное множество с минимальным элементом к будет мэ-множеством. Возьмем теперь произвольное множество из первого класса (пусть к - число его элементов; ясно, что к > 2), удалим в нем элемент п + 2, а все остальные элементы уменьшим на единицу. Получим к — 1-элементное
126 Ответы. Указания. Решения множество с минимальным элементом к — 1 — это мэ-множество, при- чем максимальный элемент его не превосходит п. Установлено взаимно однозначное соответствие между множествами из первого класса и мэ- подмножествами множества {1, 2,..., п). Таким образом, первый класс со- держит fn множеств. Соотношение fn+z = fn+i + fn доказано. Замечание. Из чисел 1, 2,..., п можно составить ^-элементных ми- нимальных э-множеств (к числу к нужно добавить ещё к — 1 больших к чисел). Поэтому /n = c'°_1 + c'L2 + c'L3 + -- [^] = У c'Lfc-i к=0 Другие доказательства данного тождества см. в задачах 320 и 330. 356. А(х) = В(х + Р). 357. 3) ЛМ = 4) °» = 2?s ((! + (^)“ + (V5 - 1) (^)“); / / п / П\ 1. _ 1 I (3+л/51 _ [ 3-л/51 \ п— л/5\Д2 ) \ 2 ) ) ' 362. 2- (—1)П + 2П. 372. Sn+2 = 14Sn - Sn_i; rn+2 = 4rn+i - rn. 375. 927. Указание. Обозначим через an число двоичных последова- тельностей длины п, удовлетворяющих условию задачи. Найдите началь- ные условия и рекуррентное соотношение для последовательности (ап). 378. 4) 55252. 379. (16п — 4п)/12. Решение можно найти в [58]. 380. Сделайте замену уп = 2ап + 1 и докажите, что yn+i = 4уп — yn-i. 381. Обозначив левую часть доказываемого тождества через Sn, дока- жите, что Sn+i = Sn — pqSn-г. 382. 99 руб 99 коп. Если бы у шулера было 100 руб, то он был бы всегда в выигрыше. 383. Нет. Решение. Каждый шаг удваивает длину последовательно- сти. После очередного шага она имеет вид (для некоторого п) Ср, 02, , 1 — Hi, 1 — О2 Д — а3, Д — а2п-
127 1°. Докажем индукцией по к, что \/к а2к = 1 — или °2fc + «2*—i = 1- База индукции очевидна. Пусть доказываемое утверждение верно для к < 2п. Докажем его справедливость для к, удовлетворяющего двойному неравенству 2П < к < 2n+1. Действительно, «2~+2г = 1 - «2г, a2"+2i-l = 1 “ й2г-1 (W 1 < 2г - 1, 2г < 2П), Откуда й'2п+2г + «2теН-2г—1 = 2 — (<72г + «2г—1) =2—1 = 1. 2°. Вновь по индукции докажем, что \/к a2fc-i = ак- Для к = 1 это утвер- ждение тривиально. Пусть оно выполняется для к < 2п. Тогда при г < 2п имеем Щ = й2г-1, O-2n+i = 1 ~ di, Й2"+2г-1 = 1 — «2г—1, откуда a,2n+l+2i-i = т.е. доказываемое соотношение выполняется и при к < 2n+1. 3°. Пусть ап — периодическая с некоторого места последовательность и Т - период. Докажем, что число Т — чётно. Если Т нечётно, найдётся период, первый элемент которого имеет нечётный индекс. Рассмотрим этот период и следующий за ним: «2j —1, ^2j, , a2j+T-2', d2j+T-l, , <^2j+2T-2- В силу 1° этот отрезок последовательности содержит поровну нулей и единиц, этот ’’паритет” должен иметь место и для каждого периода, что, однако, невозможно из-за нечётности Т. 4°. Пусть Т = 2s — наименьший период. Тогда ЗА; о \/к > ко ак+т = ak- В силу 2° \/к >к0 ак = a2k-i = a2fc-i+T = a2(fc+o-i = Uk+s, то есть s - также период последовательности, что противоречит минимальности Т. Доказано, что последовательность не является периодической. 390. (п — 1)! 391. 98475. 395. 16 и 11 соответственно. 401. |(С84 + 2С26 + ЗС42 + 2(С48 + С^8 + С4С28 + С^8 + С8)) = = 553 332 533. 402. |(224 + 2 • 26 + 3 • 212 + 2 • 24 • 210) = 2 102 800. 403. 13 звёзд и 88 мечей. 406. Существует 11 простых графов с 4 вершинами и 34 — с 5 верши- нами. 407. 1) 1; 2) 2.
128 Ответы. Указания. Решения 411. Пусть v — вершина максимальной степени; k = р(г). Если ни одна из к вершин, смежных с v, не является висячей, то степени этих вершин принимают значения от 2 до А;; поэтому по крайней мере две из них имеют одинаковую степень, хотя соединены цепью длины 2. 415. I 52Li Pi вершин и \ 52Li(Pi “ Pi) Рёбер. Решение. Вершина рёберного графа L(G), отвечающая ребру vpjj в G, имеет степень (рг + pj — 2). В сумме А = ^{pi + pj — 2), где суммирование ведётся по всем рёбрам G, степень каждой вершины pi встречается ровно Pi раз. Поэтому удвоенное число рёбер L(G) равно А = Pi ~ 2m, где 2m = 52 А удвоенное число рёбер исходного графа G. Другое решение. Ребра, инцидентные г-й вершине в графе С, образу- ют С2. пар смежных рёбер, каждой паре соответствует ребро в рёберном графе. Суммируя CPi по всем г, вновь найдём общее число рёбер С (С). 416. Достаточность очевидна. Необходимость. Из предыдущей за- дачи следует: 527=1 Pi = 2n; 52”=1 р! = 4п> откуда ^!i=i(Pi ~ 2У = °> ъе- \/г pi = 2. Связный регулярный граф степени 2 есть циклический граф. 441. п2 + п. 445. 1) а) Р4(ж1, ж2, ) = 21М + 9z2z2 + 8хз + б) Р5(з71,372, • • •) = (^}° +1037^2+20^1 +30x2^4-1- 15z2+20zJZ3Z5+24Z2). 2) Положив вес ребра равным х, а вес неребра равным 1, получим следую- щие производящие функции ^2атхт, где т — число графов с т рёбрами: а) = ^W(F) = Р4(1-|-а7, 1+ж2, ...) = 1+х+2х2+Зх3-\-2х4+х5+х6-, б) 52 атХт = 52 (^) = Р5 (1 + 37, 1 + Z2, • • •) = 1 + 37 + 2з72 + 4з73 + 6х4 + 6х5 + 6х6 + 4а77 + 2а78 + 379 + 3710. 451. При нечётных п. 453. 3 раза. См. задачу 452. 454. 1) Да; 2) нет. 459. В гамильтоновой цепи и —> ... —> г, где вершины и и г не смежны, произвольная вершина, смежная с и (обозначим её и'), не может следовать за вершиной (например, v'), смежной с v. Действительно, гамильтонова цепь и v' и' —> т легко преобразуется в гамильтонов цикл и —> ... v' —> v —> ... и' —> и. Поэтому число вершин, не смежных с и, не меньше числа вершин, смежных с v, то есть п — 1 — р(и) > p(v), или р(и) + р(у) <п — 1. 460. Предположив, что граф не является гамильтоновым, будем после- довательно добавлять к нему рёбра до тех пор, пока он не станет гамильто-
129 новым. Удалив последнее добавленное ребро uv, получим полугамильтонов граф G', не являющийся гамильтоновым. В нем существует гамильтонова цепь к г;, причем вершины и и v не смежны. Применение резуль- тата предыдущей задачи даёт: p'(u)+p'(w) < п—1, где р'(и), p'(v) — степени вершин и и v в графе G'. Осталось заметить, что р'(и) > р(и), р'(у) > р(у), откуда р(и) + р(г) < р'(и) + р'(т) < п — 1 < п. Получено противоречие с условием. 461. 38. 462. Упорядочим степени вершин графа по неубыванию: pi < р2 < ... < рп- Условие теоремы теперь можно сформулировать так: п — 1 2 V/c Рк > к. Предположив, что граф негамильтонов, будем, пока это возможно, доба- влять к нему рёбра так, чтобы не получить гамильтонова графа. В резуль- тате получим граф, в котором любые две несмежные вершины соединены гамильтоновой цепью, и, как показывает задача 459, сумма их степеней не больше п — 1. Для степеней вершин полученного графа G' условие (*) сохраняется. Выберем в G' пару несмежных вершин и и v, сумма степеней которых принимает наибольшее значение. Пусть к = р(и) < р(т). Тогда (поскольку р(и) + р(т) < п — 1) выполняется неравенство к < Кроме того, степень любой вершины, не смежной с г, не превосходит р(и) = к. Вершин, не смежных с v, не меньше, чем вершин, смежных с и (см. задачу 459), т.е. не меньше к. Таким образом, не менее, чем к вершин, имеют сте- пень не больше к. Поэтому pk < к. Полученное противоречие доказывает теорему. 466. Рассмотрите вершину, смежную с концом самой длинной цепи. 469. г = Р±±] . 471. Нет. 478. п71"1. 479. к + 1. 482. 1 + кщ — Y^k=inic- Из общего количества вершин нужно вычесть количество внутренних вершин. 483. С&/(п + 1). 484. Наибольшее число вершин равно 2n+1 — 1, а наименьшее /п+з — 1. Указание. Пусть ап — наименьшее число вершин в сбалансированном дереве высоты п. Тогда а0 = 1, щ = 2, ап+2 = 1 + ап + ап+1. Отсюда для разности ип = ап — ап-1 имеем щ = 1, = 2, ип+2 = ип+1 + ип, то есть ип = fn+i — n+1-e число Фибоначчи. Значит, ап = un + un_i + .. , + ui + ao = fn+1 + fn + + /2 + fi- Осталось использовать тождество 3) задачи 318.
130 Ответы. Указания. Решения 485. 42. 486. к < 4. 490. 2m = sf = p(m — f + 2). 491. Пять платоновых графов, циклические графы и графы с двумя вершинами без петель. Указание. Если в графе из условия задачи более двух вершин, то степень каждой вершины не превосходит 5. Используйте далее тождество предыдущей задачи. 492. Нет. 493. Если у планарного графа п вершин и тп рёбер, то выполняется неравенство m < Зп — 6. 494. Индукция по числу вершин. 495. Нет. Указание. Докажите связность графа. Применяя лемму о рукопожатиях, легко найти число рёбер графа: m = 25. Если бы граф был плоским, то (по формуле Эйлера) он имел бы 17 граней. Так как каждая грань ограничена не менее, чем 3 рёбрами, для тп имеем неравенство 2m > 3-17, а оно не выполняется. 497. Доказательство можно найти в [44], с. 100. 498. Возьмём грань с наибольшим числом сторон. Сколько сторон мо- жет быть у смежных граней? 500. См. рис. 502. 1) В Кп имеем полных подграфов с 5 вершинами. Каждый из них даёт хотя бы одно скрещивание, а каждая пара скрещивающихся рёбер входит в п — 4 таких подграфов. Отсюда Сп = n(n-l)(n-2)(n-3)(n-4) = п — 4 5! • п — 4 5 2) В Ктп имеем подграфов вида К3;3. Каждый из них даёт хотя бы одно скрещивание, а каждая пара скрещивающихся рёбер входит в (т — 2)(п — 2) таких подграфов. Отсюда сг(7Ст>га) > СЗ 7^3 _____т ' (т — 2)(п — 2) '~'т ' 9
131 503. cr(A”6) = 3; сДАДД = 2; сх(К±д) = 2. 505. 4 cr(G)/n — это среднее число скрещиваний, приходящихся на рёбра, инцидентные вершине графа. 506. На рис. приведено изображение графа Петерсена с двумя пересе- чениями. Для доказательства того, что одним пересечением при изображении графа обойтись нельзя, используйте утверждение задачи 505. 508. п/2п~х. 510. В исходном графе существует эйлеров цикл. 511. Перенеся доказательство теоремы Эйлера на случай графа из условия задачи, можно построить замкнутый путь, содержащий все ду- ги орграфа. 515. Произвольно ориентируем рёбра графа. Поскольку сумма полу- степеней исхода (пси) всех вершин равна числу рёбер и, значит, есть число чётное, число вершин с нечётной пси чётно. Возьмём произвольную цепь, соединяющую две такие вершины, и поменяем ориентацию всех входящих в нее дуг. Теперь пси этих вершин станут чётными, в то время как пси остальных вершин не изменятся. Повторяя эту процедуру, пока она воз- можна, получим в конце концов требуемую ориентацию дуг. 522. Пусть в турнире п вершин. Исходя из соотношений = и ^Г(г) + ~fr(v) = п — 1 (Vt), получим: — У'Т^2(г’) = - У М)(УМ + У И) = (п -1) Е(У(О - УМ) = 0. 524. Постройте соответствующие примеры при п = 3ип = 6и приду- майте, как от турнира с п вершинами перейти к турниру с п+2 вершинами. 529. Примените индукцию по числу вершин. 530. Примените индукцию по длине цикла. 532. Из сильной связности следует, что всякая вершина входит в неко- торый цикл; например, щ входит в цикл щ —> г>2 —> щ длины п. Если п > 3, то можно указать цикл меньшей длины. Действительно,
132 Ответы. Указания. Решения если в графе есть дуга г3г>1, то это цикл щ —> v3 —> v1; в противном случае — цикл щ —> г3 vn . 536. Рассмотрите остовный лес графа. Он является двудольным гра- фом. 538. 1, 2, 2, 4, 16. Пример минимального доминирующего множества при d = 7: 0000000, 0000111, 0011001, 1100001, 0101010, 1010010, 0110100, 1001100, 1111111, 1111000, 1100110, 0011110, 1010101, 0101101, 1001011, 0110011. 540. Так как диаметр графа равен 2, доминирующим множеством бу- дет Г (г) — множество вершин, смежных с т, где v — произвольная вершина графа. Поэтому 5(G) < min„6y p(v). При 5(G) = 1 доказывемое утверждение очевидно. Считая, что 5(G) > 2, положим m = 5(G) — 1. Тогда Vw р(у) > тп. В силу того, что ника- кое множество из т вершин не является доминирующим, для любых т вершин графа G найдётся вершина, не смежная с этими т вершинами. Поскольку всего существует G™ множеств из т вершин, найдётся такая вершина и в графе G, для которой существует не менее С™/п неупо- рядоченных наборов из т вершин, ни с одной из которых эта вершина не смежна. Действительно, обозначив через число т-элементных мно- жеств вершин, не пересекающихся с T(rj), получим ki > С™, откуда max ki > С™/п. С другой стороны, так как р(и) > 6(G) = т + 1, для дан- ной вершины и найдётся п — 1 — р(и) < п — т — 2 вершин, не смежных с ней. Из указанных вершин можно составить не более С™_т_2 наборов по т вершин. Таким образом, выполняется неравенство G"Lm_2 > С™/п, или G"Lm_2/G™ > 1/п. Последнее неравенство запишем в виде (п — т — 2)(п — т — 3)... (п — 2m — 1) п(п — 1)... (п — т + 1) т + 2 п — т + 1 В полученном произведении первый множитель больше всех последующих и поэтому больше среднего геометрического всех т множителей, которое, в свою очередь, не меньше, чем 1/ х/п. Получено неравенство т + 2 п 1 х/П
133 Прологарифмируем его: 1п(1 — п^) > — Применив (справедливое для О < х < 1) неравенство 1п(1 — х) < —х, получим теперь — , или т2 + 2т < nlnn, откуда т < —1 + \/nlnn + 1. Таким образом, 3(G) = т+1 < у/nlnn + 1, что и требовалось доказать. 546._______________________________________________________ граф G Cn Kn Wn ^n,m граф Петерсена Uq(G) 1 1 । 1 M max(n, m) 4 MG) n — 1 n~ M min(n, m) 6 &i(G) 1 1 1 1 №|3 1 M min(n, m) 5 MG) »- ^1 » - M max(n, m) 5 549, 550. Доказательство можно найти в [7]. 551. Используйте теорему О. Оре (задача 460). 552. Доказательство ведётся индукцией по п. 553. Число вершин в кубическом графе чётно. Гамильтонов цикл рас- падается на два совершенных паросочетания. Оставшиеся рёбра составля- ют ещё одно совершенное паросочетание. 555. См. рис. 556. Если совершенное паросочетание одно, то ’’структура” графа оче- видна. Пусть теперь число совершенных паросочетаний, на которые рас- падается граф, не менее двух и в графе есть мост — ребро uv. Рассмотрим ту компоненту, которой принадлежит вершина и после удаления ребра uv. В этой компоненте имеется одна вершина чётной степени, а остальные вер- шины нечётной степени. Из леммы о рукопожатиях получаем, что в этой компоненте связности нечётное число вершин. Значит, после удаления ре- бра uv граф не содержит совершенного паросочетания. По условию ребро uv входит в одно из совершенных паросочетаний; других, стало быть, нет. Полученное противоречие доказывает утверждение задачи.
134 Ответы. Указания. Решения 557. Данную задачу решают организаторы спортивных турниров, про- ходящих по круговой системе (когда каждый играет с каждым). Если вершинам графа К^п сопоставить участников турнира, а рёбрам - встречи между ними, то каждому туру состязания соответствует неко- торое совершенное паросочетание (участники образуют п пар). Шахматисты поступают следующим образом. Если участник турнира получил при жеребьевке номер i < 2п, то он последовательно встречает- ся с игроками, имеющими такие номера: 2п + 1 — i,2n + 2 — i,... ,2п — 1,1,2,... ,i — 1, 2n, i + 1,..., 2n — i. Для иллюстрации приведем расписа- ние встреч для турнира с 8 участниками. В каждой клетке следующей таблицы проставлен номер тура, в котором встречаются соответствующие участники. N участника 1 2 3 4 5 6 7 8 1 2 3 4 5 6 7 1 2 2 4 5 6 7 1 3 3 3 4 6 7 1 2 5 4 4 5 6 1 2 3 7 5 5 6 7 1 3 4 2 6 6 7 1 2 3 5 4 7 7 1 2 3 4 5 6 8 1 3 5 7 2 4 6 В турнирах по настольному теннису принят иной алгоритм распределения встреч по турам. В качестве примера приведем расписание встреч для 8 участников (каждая колонка отвечает очередному туру). 1 - 8 2 - 7 3 - 6 4-5 1 - 7 8 - 6 2 - 5 3 - 4 1 - 6 7- 5 8 - 4 2 - 3 1 - 5 6-4 7-3 8-2 1 - 4 5 - 3 6 - 2 7-8 1 - 3 4 - 2 5 - 8 6 - 7 1 - 2 3 - 8 4-7 5 - 6 При переходе к очередному туру 1 стоит на месте, а любое другое число сдвигается против часовой стрелки на одну позицию либо (после ’’встречи” с единицей) на две позиции. Читателю предлагается самостоятельно доказать корректность описан- ных алгоритмов. В книге [39] (гл. 8, §3) приводится ещё один способ составления распи- сания встреч кругового турнира.
135 558. Ребра одного цвета должны составлять совершенное паросочета- ние. При нечётном п ответ отрицательный, при чётном п — положитель- ный (см. предыдущую задачу). 564. Решение задачи о свадьбах существует тогда и только тогда, когда любые к юношей из данного множества знакомы в совокупности не менее, чем с к девушками. 569. Нет. Инвариант — объём тетраэдра, в вершинах которого сидят мухи. 570. Нет. Разность числа хамелеонов разного цвета каждый раз либо не меняется, либо меняется на 3. 571. а) да; б) нет. Изучите остатки от деления на 3. 573. Нет. После каждого броска меняется ориентация треугольника. 574. Нет. Инвариант — дискриминант квадратного трёхчлена. 575. Только при нечётном п. Инвариант — сумма номеров деревьев, занимаемых чижами, по модулю п. 576. Пронумеруем лампочки, начиная с зажённой, числами от 1 до 12. Подсчитайте, как может измениться за каждый ход число горящих лампочек в множестве {1, 2, 4, 5, 7, 8,10,11). 578. Сумма попарных произведений соседних чисел увеличивается с каждой операцией. 579. Полуинвариант — сумма квадратов чисел. 580. Разделим сначала парламент на две палаты произвольным обра- зом. Если перевести в другую палату того, у кого в данной палате более к врагов, то общее число пар врагов в парламенте уменьшится. 581. Нет. 582. Нет. 583. 1) Да; 2) нет. 584. Пусть а = л/п. Для приведения к противоречию предположим, что при повороте второго многоугольника на угол к а его вершина с номе- ром ед, совпадает с вершиной первого многоугольника, имеющей тот же но- мер (к = 1,..., 2п). Тогда первоначально эта вершина занимала позицию, чей ’’номер” сравним по модулю 2п с числом — к. Заметим, что числа Q принимают все значения от 1 до 2п; числа с^ — к образуют полную систему вычетов по модулю 2n. С одной стороны, 53fc=r (cfc — к) = ск — 52 = О- С другой стороны, 5Zfc=i(cfc — к) = (2п + l)n 0(mod2n). Противоречие получено.
136 Ответы. Указания. Решения 585. Число п должно быть нечётно. Если п = 2k +1, то на диагонали, идущей с верхней правой клетки к ниж- ней левой клетке, стоят неповторяющиеся числа 2k + 1, 2k - 1, 2k - 3,..., 1, 2k, 2k - 2,..., 2. Пусть n = 2k. Обозначим через элемент таблицы, стоящий в г-й стро- ке и j-m столбце. Имеет место соотношение = j — i + l(modn). Если {?!,..., гп} = {ji,..., = {1,..., п} (т.е. aiujl,..., ainJn — числа, взятые по одному из каждой строки и каждого столбца таблицы), то 52aifcjfc = ~ ik + 1) = п = O(modn). Если бы все числа j,,..., cnn,jn были бы разными, то выполнялось бы сравнение ^2 ач.,3k = "Нг ‘ п = (* + 2k)k = k(mod 2к), противоречащее ранее найденному. 586. Занумеруем вершины по ходу часовой стрелки числами от 1 до 2п. Звенья ломаной i — j и к — I параллельны тогда и только тогда, когда г + j = к + I (mod 2п). Пусть идущие по порядку вершины ломаной име- ют номера ai, а2,..., ап, ai. Рассмотрим остатки от деления на 2п чисел а, + йг+i- Если все они различны (что равносильно отсутствию в ломаной параллельных звеньев), то они принимают все значения от 0 до 2п — 1 и их сумма равна п(2п — 1), что сравнимо с п по модулю 2п. С другой стороны, 22^!(а, + ai+i) = 2^2^ г = 2п(2п + 1) = 0(mod2n). Полученное противоречие доказывает утверждение задачи. 587. За 2п + 1 ход. Решение. Пусть к (тп) — разность числа ходов волка вправо и влево с горизонтальной составляющей п (соответственно n+ 1); г (s) — разность числа ходов вверх и вниз с вертикальной составляющей п (соответственно п + 1). Если волк передвинулся в соседнюю сверху клетку, то кп + тп(п + 1) = 0; (1) гп + s(n + 1) = 1. (2) Из (1) следует, что к : п + 1, тп : п. Если к = 0, то тп = 0, a s чётно; равенство тп = 0 влечет чётность г; но тогда левая часть (2) чётна, что невозможно. Поскольку к ф 0 и тп ф 0, имеем: Д| > п+1 и |т| > п. Значит, общее число ходов не меньше 2п + 1. Покажем, как за 2п + 1 ход попасть в нужную клетку. Для этого можно сделать п ходов типа (п, — (п + 1)), один ход (n, n+ 1) и п ходов типа (~(п + 1), п), где (а, Ь) означает ход на а клеток по горизонтали и b клеток по вертикали, а знак минус соответствует движениям влево или вниз.
137 ^2fc+l — 588. Докажите, что любые две противоположные (п — 1)-мерные грани соединяет ребро с покрашенными концами. 589. 2. 591. Нет. 592. При чётном п число ап делится на 101; при нечётном п имеем разложение 102fe+1 - 1\ /102fc+1 + 1 9 J V П 595. тп = 1,п Е N; тп = 2, п = 3. 605. Используйте метод математической индукции. В тождестве ж4 + х2 + 1 = (ж2 + 1 — х) (ж2 + 1 + х) положите ж = 22" 1. Числа 22" — 22" 1 + 1 и 22"+22П +1 взаимно просты, так как если бы у них был общий (нечётный) множитель то их разность 22" +1 делилась бы на q, что неверно. 606. 1 000 001 = 101 • 9901; 1 000 009 = 293 • 3413; 1000 049 = 353-2833; 1000169 = 197-5077; 1000 361 = 97-10 313; 1 000 441 = 13 • 41 • 1877; 1 000 529 = 29-34 501. 608. 13 717421 = (102 + 7 • 232) • (602 + 7 • I2) = 3803 • 3607. 609. Доказательство будем вести методом от противного. Предполо- жим, что а]Ь1, а2Ь2, , апЬп — полная система вычетов. Если при г j щ = 0(modn) и bj = 0(modn), то afii = ajbj = 0(modn) и числа tzxfri, a2 ,b2,..., anbn не образуют полной системы вычетов по модулю п. По- этому без ограничения общности можно считать, что ап = bn = 0(modn). Можно также положить, что Vz а, = г. Рассмотрим два возможных слу- чая. • Число п свободно от квадратов. Пусть п = ртп, где р — наибольший простой делитель п. Тогда р > 2 и тп не делится на р. Если j :тп, то ajbj = jbj-.m и остаток от деления ajbj на п кратен тп. Для полной системы вычетов aibi, а2, Ь2,..., апЬп остатков такого вида должно быть ровно р (включая нулевой). По- этому для любого j, делящегося на тп, число bj также кратно тп. Вычислив произведение чисел ajbj для j = тп, 2тп,..., (р — 1)т, по- лучим р-i ПдьАт = тп • 2тп • ... • (р - 1)т = [р - l)!mp-1(modn). t=i
138 Ответы. Указания. Решения С другой стороны, р— 1 р— 1 р— 1 Yfakmbkm = 'ПЛ'"' = ((р~ l)!mp-1)2(modn). fc=i fc=i fc=i Таким образом, (р — l)!mp-1 = ((р — l)!mp-1)2(modn), или (р— l)!mp-1((p— l)!mp-1 — 1) -.pm, откуда в силу простоты р следует, что (р — l)!mp-1 — 1 : р. По теореме Вильсона (р — 1)! = — l(modp). Стало быть, mp-1 + 1 :р. В силу малой теоремы Ферма mp~1 = l(modp), и, значит, 2 : р, что невозможно, так как р > 2. Получено противоречие. • Число п делится на квадрат некоторого простого числа. Пусть п = p2s, где р — простое число. Тогда aj = j - Р •<=> : Р •<=> ajbj : Р (в противном случае среди чисел а^, а^, Ь%, , anbn будет ’’больше, чем нужно” (для полноты системы вычетов) таких чисел, которые при делении на п дают остаток, кратный р). Таким образом, если cijbj : р, то djbj : р2, и среди чисел , а2> &2> • • • > апЬп нет, например, имеющего остаток р от деления на п = p2s. Вновь пришли к проти- воречию. Замечание. В [41] рассмотрен частный случай этой задачи, когда п — простое число. п цифр 610. Обозначим ап = 22 и bn = ап — an_i. Докажем сначала, что в последовательности (6П) каждый член (начиная с 63) делится на предыдущий (значит, и на все предыдущие). Действительно, Ь^ = 22 — 2 = 2; &з = п3 — п2 : 2 = &2- Предположим, что &fc+i : bk. Тогда bk+2 = ак+2 - ак+1 = 2^+* - 2ак = 2ак(2ак+1~ак - 1) = 2ак(2Ьк+1 - 1). Поскольку последовательность (ак) возрастающая, 2ak : 2a'=-1. Так как (по предположению индукции) bk+i '-Ьк- имеем также 2^+1 _ 1 : 2Ьк - 1. Таким образом, Ьк+2: 2ак~1(2Ък - 1) = Ьк+Х.
139 Перейдем к непосредственному доказательству утверждения задачи. Оно также проводится методом математической индукции. База индукции очевидна (62 = 2). Пусть теперь для любого к < п выполняется Ьк : к. Представим число п в виде п = 24, где t — нечётное число. Легко проверить, что при п > 3 имеет место неравенство пп-2 > п. Отсюда bn : 2ап~2 : 2п : 2s. Теперь осталось доказать, что 2Ьп-1 — 1 : t. Применим теорему Эйлера. Заметим, что <p(n) = <p(2s) -<^(i) = 2s-1<^(i). Поэтому <^(i) < <Дп) < n — 1. Для любого к < п — 1 имеем bn-i : bk : к. В частности, bn-\ : <р(£). Отсюда следует, что 2bn~1 — 1 : 2^ — 1. В силу теоремы Эйлера 2^ — 1 : t. Из последних двух соотношений и вытекает, что 2Ьп-1 — 1 : t. Таким образом, Ьп : п. 611. ак - 1 = (а - l)(afe-1 + ак~2 + ... + 1). Так как а = l(modA;), то = l(modfc) и cd : к. 612. Доказательство проведём в несколько этапов. 1°. Применив п раз утверждение задачи 611, получим следующий резуль- тат: Если а — 1 : кт, то акп — 1 : кт+п. 2°. Докажем, что Vn un+i — ип : 2п. При чётном а число ип = а"71-1 делится на 2п (так как un-i > п). Пусть теперь а нечётно. Доказательство будем вести по индукции. База индукции очевидна: и? — щ : 2. Пусть ик+^ — ик = s 2к (stN). Тогда iZfc+2 — tZfc-i-i = Uk+i(aUk+1~Uk — 1) : as'2k — 1. Осталось заметить, что as — 1:2 и поэтому (в силу 1°) (а5)2*" — 1: 2fc+1. Индукционный шаг доказан. 3°. Докажем, что Vn > 2 nn+i — ип : 5П-2. Если а кратно 5, то Vn ип = а"71-1 : 5П (так как ип-\ > п). Отсюда следует, что Vn nn+i — ип : 5П. Если а не делится на 5, то по малой теореме Ферма а4 — 1 : 5. Вновь применяя индукцию, предположим, что Uk+i — ик = s 2к 5к~2. Тогда (при к > 2) ик+2 ~ ик+1 = uk+1(aUk+1~Uk - 1) = uk+l(a4t'5k 2 - 1),
140 Ответы. Указания. Решения где t = s• 2fe 2. Так как а4 — 1:5, а4г — 1:5 и в силу 1° имеем: (а4^5* 2 — 1 : 5fc-1. Индукционный шаг доказан, а база индукции очевидна. 4°. Из 2° и 3° вытекает утверждение задачи. Замечание 1. Аналогично можно доказать,что Vm 3Z Vn > I un+i — un : mn 1 Замечание 2. Развитие темы двух последних задач см. в [56]. 614. Отметим сначала, что каждое слагаемое делится на р (С? = mp-iy— Т1елое число; числитель дроби кратен р, а знаменатель — нет (так как р — простое число)). Доказываемое утверждение равносильно делимости на р суммы к\ — . Проведем сравнение по модулю р: к\Сг А:! (р-1)(р-2)...(р-г + 1) _ (-1)^4! --- = —--------т-—76----------=-----;---(modp). р г (г — 1)! г Итак, задача сводится к проверке того, что По условию р не делится на 3. Рассмотрим два возможных случая. 1) р = Зиг + 1. Здесь m — чётное число (иначе р делилось бы на 2). Тогда А; = [6т3+2] = 2m. Имеем: = (Зиг + 1) : Зиг + 1 = р. 2) р = Зпг + 2. Здесь тп —нечётное число (иначе р делилось бы на 2). Тогда А: = [6та3+4] = 2m + 1. Имеем: 1 ~ 1 ~ 2 к'. (ЫД...+ 1 2m + 1
141 1 m + 1 2 2 1 2m + 1 . J=i 1 1 + j 2m + 2 — j (2т + 1)! / -----гтх-----х---г : 3m + 2 = р. (т + ])(2т + 2 - j) 615. не делится на 5 при только при п = 1,..., 4, 20,..., 24,100,..., 104,120,..., 124. Подробное решение см. в [63]. 616. Нечётно. Решение приведено в [63]. 620. ат = a(modm) => ар-1 = am(p-1)(modm). Так как к тому же т:р2, отсюда следует, что ар-1 = am(p_1)(modp2). Наконец, т ; р =} ат'Т,~') - 1 : - 1 = -Г:р2. 621. Нет. 623. Достаточно доказать, что наибольший общий делитель г = (m — 1,р — 1) равен (р — 1). Поскольку Vcz, (а, т) = 1 ат 1 — 1 : т : р, ар 1 — 1 : р, имеем: (ато-1 — 1, ар~1 — 1) = а(то-1’Р-1) — 1 = аг — 1 :р. Таким образом, при г < р— 1 сравнение xr — 1 = 0(modp) имеет р— 1 > г решений (1, 2,... ,р— 1). Осталось применить результат задачи 100 и получить противоречие. 624. В таблице приведены числа Кармайкла < 2 117 000. 561 10585 63973 188461 399001 656601 997633 1461241 1105 15841 75361 252601 410041 658801 1024651 1569457 1729 29341 101101 278545 449065 670033 1033669 1615681 2465 41041 115921 294409 488881 748657 1050985 1773289 2821 46657 126217 314821 512461 825265 1082809 1857241 6601 52633 162401 334153 530881 838201 1152271 1909001 8911 62745 172081 340561 552721 852841 1193221 2100901 Студент Алексей Гуйдо составил программу, которая нашла первые 336 чисел Кармайкла. Самое большое из них — 146 843 929. Пусть с(п) — коли- чество чисел Кармайкла, не превосходящих п. Известны ([49]) следующие значения с(п).
142 Ответы. Указания. Решения к 3456 7 8 9 10 c(10fe) 1 7 16 43 105 255 646 1547 Интересные сведения о числах Кармайкла можно найти в Интернете по адресу www.utm.edu/research/primes/giossary/CarmichaelNumber.html. 627. (/op) o/z(n) =/о (po/i)(n) = £ di d2d^=Tt 628. = £ /(ш4т)4т) = Еж4т)Еж4^ = rfl|n,d2|m W di\n d2\m = (JодЬ)) 629. Определим функцию следующими соотношениями. /'(1) = 1. (Тогда f о /'(1) = / • /'(1) = 1.) Для каждого простого числа р пусть f'(p) = (Отсюда следует, что f о /'(р) = 0.) Положив fW = -Шрп) + Ж’7'(р) + • • • + ЖЖ-1)), мы добьемся того, что Vn /о f'(pn) = 0. Пусть теперь /' — мульти- пликативная функция. Поскольку значения f определены на всех степе- нях простых чисел, свойством мультипликативности функция полностью определена. В силу задачи 628 функция Ф = f о f мультипликативна. По- скольку Ф(1) = 1 и для любого простого числа р и любого натурального п справедливо Ф(р") = 0, Ф(р?-...-р^)=Ф(рГ)-...-Ф(р?) = 0. Доказано, что f о f = J. 630. Пусть f — мультипликативная функция со свойством f о f = J. Тогда функция д = Jog = (f'of)og = f'o^fog^) также мультипликативна. 633. Пусть п = р\1 рг22... prkk, где k > 1. Поскольку р(с?) 0 только если число d свободно от квадратов, имеем: ^2d\np(d) = р(1) + Е P^Pi) + Ел(ад) + ... + ц(Р1 ...рк) = 1-к + С%-С% -... + (-!)* = (1 - l)fe = 0.
143 634. р от = Е, //os = I. 635. Пусть d — делитель п и Ad — множество чисел к < п таких, что (к, n) = d. Элемент Ad представим в виде к = dki, где число ki взаимно просто с числом n/d и не больше его. Поэтому |ЛЙ| = tp(n/d). Осталось заметить, что п = |ЛД 637. Следствие задачи 633. 639. InoE'(n) = Inn. 642. M(n) = /1(d) к г. 644. 1) 51; 2) 130; 3) 315; 4) 834. 645. Предположите противное. 647. Последовательно докажите: х * х = 0, х * 0 = х, х * у = х — у. 648. Определим С = 1*0. Из тождества задачи при х = 1, у = 0 имеем Cz = z * 0, а при z = 1, t = 0 соответственно х*у + Су = х*О = Сх, откуда х * у = С(х — у). 649. a*x = b=>x = b* a; x*a = b=}a = b*x=>x = a*b. Если в X нет идемпотентных элементов, то число клеток в квадрате Кэли должно быть кратно трем ввиду соотношения x*y = z=}z*x = y, у * Z = X. 650. х * (у * х) = (у * (у * ж)) * (у * х) = у; у *х = у* ((а? *у)*у) = х*у. 651. Пусть в множестве А п элементов. Из 2) следует, что ровно полови- на всех упорядоченных троек (различных) элементов А входит в S. Зафик- сируем некоторый элемент w 6 А и на множестве Л\{ш} введем отношение р: хру <==> (w,x,y) G S. Легко видеть, что оно антисимметрично: ес- ли хру, то неверно, что урх. Докажем, что данное отношение транзитивно. Пусть хру и ypz. Тогда (ш, х, у) Е S и (ш, у, z) Е S. В силу 1) и (у, z, w) Е S. В силу 2) S принадлежат и тройки (х, у, z), (z, w, х). Опять применяя 1), получим, что (ш, х, z) G S, т.е. xpz, что и требовалось доказать. Обозначим
144 Ответы. Указания. Решения через kj количество элементов х из множества А \ {w} = {щ, аг,..., an-1} таких, что (црх. Если dipcij, то из-за транзитивности и антисимметрич- ности ki > kj. (Действительно, с одной стороны, aipaj,Ojpx => (црх — этим обеспечивается нестрогое неравенство; с другой стороны неверно, что ajpai). Нетрудно видеть, что верно и обратное: если kt > kj, то aipaj (по закону контрапозиции). Положим g(aj) = п — 1 — ki, g(w) = п. Покажем, что функция д обладает требуемыми свойствами. Если д(а) < д(Ь), то apb, или (w, a, b) G S. Если с = w, то из того, что(с, a, b) G S в силу 2) следу- ет: (a, b, с) е S. Если же с w, то из д(Ь) < д(с) получаем: (w,b, с) G S, стало быть, (Ь, с, w) Е S. Поскольку при этом и (w, a, b) Е S, то в силу 3) (a, b, с) Е S. 652. Нет. 655. + 1. 656. rc3+|w+6. Решение этой и предыдущей задачи можно найти в [26], с. 46-47. 657. п2 — п + 2. 658. п(п2 — Зп + 8)/3. 659. 6"1. 660. Назовем любой квадрат со сторонами, параллельными линиям сетки, основным. Каждый квадрат вписан в некоторый основной квадрат, а в основной квадрат с длиной стороны к можно вписать к различных квадратов (с вершинами в узлах сетки). Несложно подсчитать, что имеется (п — к)2 основных квадратов со стороной к. Далее вычисляем: к • (п - к)2 = ^2(п - Д • j2 = | J2(A:(n - к)2 + (п - к)к2) = fc=i j=i fc=i 1 п-1 4 2 661- Пг=1 ^fc’+fci+i+...+fcn-l- Все возможные расположения чисел, удовлетворяющие условию задачи, можно получить в результате выполнения следующих действий. 1) Поместить 1 на первое место, а остальные единицы на любые к\ — 1 из к\ + к-2 + ... + кп — 1 оставшихся мест. 2) Поместить 2 на первое свободное место, а остальные двойки на любые к2 — 1 из к2 + ... + кп — 1 оставшихся мест. И т.д.
145 662. (2n — 1)!! О связи данной задачи с задачей 559, а также с за- дачей о монотонных плоских деревьях см. статью [14], в которой также приводятся новые результаты, связанные с числами Стирлинга. 663. 2”-1. 664. На левую. Решение. Пусть последняя гиря имела вес 2П г, а на левую чашку весов был выставлен вес в х г. Тогда имеем равенство х = 2n+1 — 1 — х +11111, или х = 2” + 5555. Заметим, что 2” + 5555 > 11111, откуда 2” > 5556 (и даже 2” > 8192). Теперь имеем х = 2” + 4096 + 1024 + 256 + 128 + 32 + 16 + 2 + 1. 665. п + 1. Решение. Ясно, что М — особое множество. Пусть М = {ад,..., хп}. Обозначим Mj = М \ {ад}. Если выбрать в качестве особых множеств М, Mi,..., Мп (всего п + 1 множеств), то условие задачи выпол- няется, поскольку произвольное множество А С М представимо в виде А= A Mi. xi£A Докажем теперь, что менее чем п + 1 особых множеств быть не может. Пусть их не больше п. Тогда отличных от самого множества М особых множеств имеем не более п — 1, а число пересечений, которые можно со- ставить из особых множеств не превосходит 2”-1, что меньше 2” — общего числа подмножеств множества М. 667. Решение Будем рассматривать подмножества множества U = {1,2, ...,п}. Пусть А С U и |Л| = к. Будем говорить, что переста- новка чисел от 1 до п порождена множеством А, если первые к элемен- тов перестановки принадлежат Л. Очевидно, что А порождает kl(n — &)! различных перестановок. Если А\В^фпВ\А^ф, то множества пе- рестановок, порожденных А и В соответственно, не пересекаются. Пусть теперь Ai, Л2,..., Ат — семейство подмножеств, удовлетворяющих усло- вию задачи; для каждого i положим kj = |Aj|. Тогда т ^ki\(n - ki)l < n! i=i Учтя, что А4(п — к)1 > ([п/2])!(п — [п/2])!, имеем т ([п/2])!(п — [п/2])! < п!, откуда т<Сп • С другой стороны, если рассмотреть все Сп подмно- жеств мощности [п/2], то они удовлетворяют условию задачи.
146 Ответы. Указания. Решения Замечание. Этот результат впервые получил Шпернер в 1928 г. Дру- гое решение (основанное на понятиях цепи и антицепи) см. в [34], с. 172— 173. С содержанием задачи тесно связана теорема Дилворта (1950 г.), её формулировку и доказательство можно найти также в книге [34], на с. 177-178. 668. 10. 669. Если k = 21, то наименьшее значение п равно 1 + 1, а если к = 21 — 2 или 21 < к < 2г+1 — 2, то наименьшее значение п равно 1 + 2. Решение (В. Белокобыльский). Пронумеруем элементы множества числами от 1 до п, и сопоставим каждому подмножеству двоичную последовательность длины п, в которой г-я цифра равна 1 (0), если г-й элемент исходного множества входит (соответственно, не входит) в данное подмножество. Для краткости двоичную последовательность будем называть маркером, а маркер длины п — n-маркером. Введём также следующие определения. Маркеры одинаковой длины — противоположные, если они различаются в каждом разряде. Набор маркеров одинаковой длины — уравновешенный, если по каждому разряду данных маркеров число единиц совпадает с числом нулей. Очевидно, что уравновешенный набор всегда состоит из чётного числа маркеров. Уравновешенный набор — правильный, если в нём нет совпадающих или противоположных маркеров. Очевидно, что в правильном наборе не менее четырёх маркеров. В новых терминах исходная задача формулируется так: При каком наименьшем п существует правильный набор из к маркеров длины п ? Пусть А — правильный набор n-маркеров. Обозначим через а маркер, противоположный к а. Пусть также А = {а | а Е А). Поскольку А и А не пересекаются, правильный набор содержит не более 2”-1 маркеров, то есть k < 2”-1, или п > log2 к + 1. (*) 1°. Покажем, что при к = 21, где I > 2, оценка (*), является точной. Действительно, с одной стороны, из (*) имеем п > Z + l. С другой стороны, рассмотрим всевозможные (/-|-1)-маркеры, среди последних трёх разрядов которых чётное число единиц. Легко проверить, что эти маркеры образуют правильный набор, и их ровно к = 21. 2°. Пусть 21 < к < 2l+l — 4, где I > 2. Из (*) имеем п > I + 2. Покажем, как построить правильный набор (Z + 2)-маркеров мощности к. Для этого сначала из 2г+1 маркеров длины I + 1 выделим к — 4 маркера таких, что в
147 каждом из них не все первые I — 1 разрядов совпадают, и каждый маркер выделен вместе со противоположным себе (поскольку к — 4 < 2г+1 — 8, а (Z + 1)-маркеров, у которых первые I — 1 разрядов одинаковые, как раз во- семь, и маркер, противоположный ’’запрещённому”, тоже ’’запрещённый”, то это всегда возможно). Получен набор, состоящий из пар взаимно про- тивоположных маркеров. Пронумеруем эти пары числами от 1 до К маркерам, входящим в пары с нечётными номерами, припишем справа единицу, а к остальным — ноль. При этом противоположных маркеров не станет (совпадающих, конечно, тоже не возникнет). Если к кратно четырем, то число указанных пар чётно, и полученный набор правильный. Дополним его такими (Z + 2)-маркерами: О ... 0000, 0 ... ОНО, 1... 1011,1... 1101. Если же к при делении на 4 даёт остаток 2, то число пар нечётно, и в по- следнем разряде единиц на две больше, чем нулей. В этом случае дополним набор такими маркерами длины Z + 2: 0 ... 0000, 0 ... ОНО, 1... 1010,1... 1101. В обоих случаях возникает правильный набор маркеров. 3°. Пусть к = 21 — 2. Из (*) следует, что п > I + 1. Покажем, что равенство здесь невозможно. Назовем два маркера одинаковой длины почти равными (п.р.), если они различаются только в последнем разряде, и почти противоположны- ми (п.п.), если у них совпадает только последний разряд. Для маркера а обозначим через а’ п.р. маркер, а через а п.п. маркер. Очевидно, что мар- керы а' и а — противоположны (значит, вместе с а в правильный набор одновременно они входить не могут), а множество всех маркеров разби- вается на четвёрки вида {а, а, а1, а}. Пусть теперь А — правильный на- бор к = 21 — 2 маркеров длины Z + 1. Тогда множество A U А содержит 2Z+1 — 4 (различных) маркеров. Заметим, что если не менее трёх маркеров (а, Ь, с,...) из А не имеют в Л ни п.р., ни п.п. маркера, то за предела- ми Л U Л остается не менее 6 различных маркеров (а1, а, Ь1, Ь, с1, с,...), что невозможно, поскольку всего имеется 2«+i маркеров длины Z + 1. Таким образом, Л состоит из пар п.р. и п.п. маркеров и ещё не более двух непар- ных маркеров (точнее, нуля или двух, поскольку общее число маркеров в правильном наборе чётно). Докажем, что на самом деле непарных маркеров нет. Действительно, в каждом из первых Z разрядов пары почти равных маркеров число единиц
148 Ответы. Указания. Решения равно 0 или 2, а по всем таким парам оно чётно. А для любой пары по- чти противоположных маркеров в каждом из указанных разрядов имеем в совокупности ровно одну единицу. Для уравновешенности набора об- щее число единиц в каждом разряде должно быть равно k/2 = 2/-1 — 1, то есть нечётно. Значит, в каждом из I разрядов общее число единиц по двум непарным маркерам должно иметь фиксированную чётность. В слу- чае чётного числа единиц имеем совпадающие или почти равные маркеры, а в случае нечётного числа единиц — противоположные или почти проти- воположные маркеры. В любом случае приходим к противоречию. Итак, непарных маркеров нет. Рассуждение, изложенное в предыдущем абзаце, говорит о том, что число пар почти противоположных маркеров нечётно. Рассмотрим последний разряд. По парам почти равных марке- ров число единиц в этом разряде совпадает с числом нулей. Значит, этот паритет должен иметь место и по парам почти противоположных мар- керов — количество таких пар, оканчивающихся единицей, должно быть равно количеству пар, оканчивающихся нулем. Но тогда количество пар п.п. маркеров чётно — противоречие! Мы показали, что в случае к = 21 — 2 выполняется строгое неравенство п > I + 1. Используя конструкцию из 2°, можно построить правильный набор (/ + 2)-марксров. 670. 360. Указание. Положим, что длина наименьшего отрезка рав- на 1. Пусть имеется т отрезков длины меньше у/2, п отрезков длины из промежутка [\/2, \/3). Тогда 16 — т—п отрезков имеют длину из промежут- ка [ч/З, 2]. Итак мы имеем три класса отрезков, квадраты длин которых принимают значения соответственно от 1 до 2 (1-й класс), от 2 до 3 (2-й класс) и от 3 до 4 (3-й класс). Тупоугольный треугольник характеризуется тем, что в нем квадрат большей стороны больше суммы квадратов двух других сторон. В условиях задачи тупоугольный треугольник не может иметь более одного отрезка из 3-го класса; поэтому число тупоугольных треугольников нс уменьшится, если считать, что все отрезки из 3-го клас- са имеют длину 2. Аналогично можно положить, что все отрезки из 1-го класса имеют длину 1,1, а из 2-го — 1,6. Тогда треугольник будет тупо- угольным в двух случаях: • он содержит по одному отрезку из каждого класса (таких треуголь- ников mn(16 — т — и)); • ровно две его стороны из 1-го класса (количество таких треугольни- ков С^(16 — т)).
149 Осталось максимизировать функцию F(m, п) = mn(16—т—п)+С^(16—т) при условии 0 < т, п, т + п < 16. 671. Пусть уп — число умножений при вычислении определителя п-го порядка. Тогда уг =0, у„ = пуп_л + п, откуда Для последовательности с общим членом ап = справедливы соотношения: Gi = 0, ап — an_i + („2Qi — Т) 2 + („22)! + (п-!)! = = 12/Li Таким образом, уп = n! Полезно заметить, что lim = е — 1. 672. хп = Dn = п\ ---число беспорядков. Рекуррентное соотношение можно переписать так: хп — пхп_\ — — (xn-i — (п — 1)т„_2). Пусть уп — хп — nxn_i. Тогда уп — —уп-\- Из начальных условий 2/2 = 1? поэтому предыдущее соотношение даст равенство уп = ( — 1)”. Итак, хп = nxn-i + ( — 1)”- Далее можно действовать, как при решении предыдущей задачи. 673 S ___2п3 -iin+18 Решение. Можно считать, что числа , а?2,... ,хп расположены по кругу и Sn — сумма попарных произведений соседних чисел. Докажем по индукции следующее утверждение: если Sn максимально, то число п стоит между числами п — 1 и п — 2. База, индукции (и = 3) очевидна. Индукционный шаг. Пусть утверждение справедливо при п — к. Если число к + 1 вставляется между числами а и Ь, то сумма попарных произведений изменяется так: Ak — sfc+1 — Sk — (& + l)(a + Z>) — ab. При фиксированном a Д& = (к + ^)а+Ь(к + 1 —а) тем больше, чем больше Ь. Аналогично при фиксированном b Д* увеличивается с ростом а. Поэтому Д^ < {к + 1)(А; + (к — 1)) — к(к — 1) = к2 + 2к — 1. По предположению индукции если Sk максимально, то числа к и к — 1 стоят рядом. Если к +1 вставить между этими числами, то получим мак- симальное значение = к2 -I- 2к — 1, а заодно и Sk+i = Sk + Ак, что и требовалось доказать. Теперь легко найти Sn. S'2 = l,2 + 2- l = 4. При п > 2 п-1 п п п Sn = -С|'2 + У Ак = S2 + У Ak-t = 4 + У ^(к2 — 2) = 4 — 2(п — 2) + к2 = k—2 к—3 к—3 к—З
150 Ответы. Указания. Решения \>-2„+3=°(П + 1)(2,, + 1) fcf 6 2n3 + Зп2 — 11 п -I- 18 — 2п + 3 = 6 676. 2(2^1 + 2^1-1). 677. 1) п2: 2) и2 - Зп + З; 3) j(j + l)(4j + 1)/2 при п = 2j, j(4.j2 - j - 1)/2 при п = 2j - 1. Указания. 1) Сравните площади треугольников. 2) Каждый треугольник со стороной 1, нс лежащий ”на границе” исходного треугольника, взаимно однозначно определяет треугольник со стороной 2. 3) Подробное решение содержится в первом издании настоящего сборника. 678. Подробное решение — в первом издании настоящего сборника. См. также журнал ’’Математика в школе”, 2001, №5, С.74. 679. 191. Решение. Пусть по кругу стоят п человек и ап — ’’счастли- вый” номер. Получим рекуррентное соотношение для последовательности (ап). При увеличении числа человек на единицу задача сводится к пре- дыдущей после удаления первого человека (имеющего номер 3), при этом отсчет начинается не с 1-го, а с 4-го человека. Поэтому новый счастливый номер на 3 больше предыдущего (по модулю п + 1). Отсюда «1 = 1: а2 = 1 + 3(mod 2) => а2 = 2; а3 = 2 + 3(mod3) => а3 = 2: «4 = 2 + 3(mod4) => а4 = 1; а5 = 4; а6 = 1: «7 = 4: а8 = 7: «9 = 1: йю — 4:...; G13 = 13:... Легко видеть: если ап = п, то ап+1 = 2; если ап = п — 1, то an+i = 1. Пусть as = 1, тогда равенство as+^ = «s+a,_i +3 будет выполняться, пока 1 + 3k < s + к, или 2к < s — 1. Поэтому положив при нечётном з Д = имеем «8+д = 2. Если же з чётно, то для Д = s/2 выполняется равенство а^+д = 1. Пусть теперь as = 2, тогда равенство = as+^-i + 3 будет выпол- няться, пока 2-1-3/? < s + /?, или 2к < s — 2. Отсюда при чётном з положив Д = s/2, имеем «8+д = 2. Если же з нечётно, то для Д = ^=4 выполняется равенство «8+д = 1. Последовательно применяя найденные соотношения, получаем: а6 = 1, Д = 3, а9 = 1, Д = 5,й14 = 2, Д = 7, «21 = 2, Д = 10, G31 = 1, Д = 16, «47 = 2, Д = 23, «7о = 1, Д = 35, «Ю5 = 1, Д = 53, «158 = 2, Д = 79, «237 = 2, «зоо = 2 + 3 63 = 191. 680. Повторим решение предыдущей задачи в общем виде. Пусть «^ - счастливый номер для круга из i человек (если удаляется каждый к-й человек). Тогда ai+i = «j + /?( mod г +1). Заметим, что если «$ +/? < г +1, то
151 выполняется простое соотношение a.j+i = сц + к. Болес того, при ai + кЛ. < 1 + Д (т.е. когда Д < выполняется равенство а;+д = а^ + йД. Если же в качестве Д взять наименьшее целое, большее то для вычисления а,+д нужно вычислять остаток от деления щ + АД на i + Д. При составлении программы будет излишним организовывать массив для значений сц, поскольку переход (г, ср) —> (г +Д, «г+д) не требует знания других элементов рассматриваемой последовательности. Ниже приведен текст соответствующей программы на языке Си. /* Решение задачи Иосифа */ /* По кругу стоят ш человек, выбывает каждый k-й */ /* happy(ш,к) - номер последнего оставшегося */ int happy(int m,int к) { int i; /* число человек в круге */ int h; /* номер последнего оставшегося */ int delta; /* приращение Ki */ if(k==l)return(m); for(i=h=l; i<m; ) { delta=l+(i-h)/(k-l); if(delta>m-i){h+-(m-i)*k; i-m;} else {i+=delta; h+=k*delta; h=h’/ei; if(h==0)h=i; } return(h); } 681. n - 1. 682. Решение можно найти в [63|. 683. Нет. Решение. Заметим, что Cg = 20. Пусть каждый ученик посещает ровно три кружка, причем у каждого набор кружков отличен от других. (В силу сделанного замечания это возможно.) Тогда и наборы непосещаемых кружков также будут разными у разных учеников. Если бы некоторые 5 человек посещали одновременно какие-нибудь два круж- ка, тогда из оставшихся четырёх кружков по крайней мере один пришлось бы посещать одновременно двум (принцип Дирихле), что приводит к про-
152 Ответы. Указания. Решения тиворсчию. Последняя фраза останется справедливой, если в ней перед словами ’’посещали’; ’’посещать” поставить частицу ”не”. 685. Левая часть тождества легко выражается через числа Стирлинга И рода ([9|, §5.11): — 1)kCkkm = nl(—iynS(m,n). Известно, что при п> тп справедливо равенство S(m, п) = 0. Замечание. Тождеству данной задачи посвящена статья |13|3, в кото- рой приводится ещё 4 способа доказательства данного тождества. Прочи- тать указанную статью весьма полезно начинающим математикам. 690. х/15. Указание. Примените полиномиальную формулу. Покажи- те, что если (х/2 + \/3 + \/5)2n+1 = anx/2 + Ьпх/3 + сп\/5 + (7„л/30, то (л/2 + </з — \/5)2'г + 1 = «п\/2 + 6П\/3 — сп\/5 — dn\/30. Данные соотношения и ещё два аналогичных составляют систему линейных уравнений относи- тельно a„,bn,c„,d„. 693. Обе части тождества выражают собой число решений в натураль- ных числах неравенства ху < п. 694. [к — это количество точек вида (А;,у), где 0 < у < к • Подсчитайте количество точек с целыми координатами, лежащих внутри прямоугольника [0,т] х [0,п], лежащих под диагональю. Другое решение (А. Бадзян). [к- = к~ — {к- Числа п, 2п,..., (т — 1)п образуют приведенную систему вычетов по модулю т. 695. 1/2. 696. Тождество возникает в результате решения различными способа- ми следующей задачи: Сколько нужно взять различных натуральных, чисел, не превосходящих п, чтобы среди них наверняка одно было вдвое больше другого? Подроб- ности — в первом издании настоящего сборника. 697, 698. Решения можно найти в |63|. 699. Запишем рекуррентное соотношение в виде (п - к)хк + (к + 1)^+2 = схк+],к = 0,1,... ,п - 1. 3Эта статья помещена также в сборнике |27], JV95.
153 С учетом условий хо = хп = 0, a?i = 1 получим матричное уравнение Ах — сх, где А — трёхдиагональная матрица, х — вектор-столбец: Теперь задача сводится к нахождению наибольшего (вещественного) соб- ственного значения матрицы А и к определению отвечающего ему соб- ственного вектора (с условием Ж] = 1). Для локализации спектра матри- цы используем круги Гсршгорина. Поскольку главная диагональ матри- цы состоит из нулей, а сумма модулей всех внедиагональных элементов каждого столбца равна (п — 1), заключаем, что собственные значения ма- трицы А расположены в круге (комплексной плоскости) радиуса (п — 1) и с центром в нуле. Легко проверить, что число (п — 1) является соб- ственным значением матрицы А и, значит, является искомым. Переходим к нахождению собственного вектора. Имеем: .щ = 1, х% = п — 1; х3 = i>i _ (л,-IX»,-2) _ ^2: Далее индукцией по к легко доказать, что для к = 1,2,..., п выполняется равенство Хь = C* *Z]. 700. (2п)! -I- ^2£=1( — l)fcC‘^2fc+1(2n — к — 1)!. Решение. Пусть А — мно- жество всевозможных расположений рыцарей за круглым столом, a Aj - множество расположений рыцарей за круглым столом, при которых ры- цари из г-й пары врагов сидят рядом (i = 1,..., п). Тогда требуемое число есть мощность множества Л] П ... П Лп, где, как обычно, Л, = А \ Л, Vz. Найдём |Лц П ... П Л^| — число расположений рыцарей, при которых рыцари из к фиксированных пар сидят рядом враг с врагом. Каждое такое расположение можно получить с помощью следующих действий. • Разместить первую пару. Это можно сделать 4п способами (пару со- седних мест из 2п мест за круглым столом можно выбрать 2п спо- собами, после чего два рыцаря могут разместиться на этих местах двумя способами). • Разместить всех остальных рыцарей, рассматривая каждую из к — 1 пар как один объект. Имеем к — 1 пар врагов и ещё 2п — 2к рыцарей -
154 Ответы. Указания. Решения всего (2п — k — 1) объектов, которые можно переставить (2п — к — 1)! способами. • Определить для каждой из к — 1 пар, кто из двух рыцарей сидит слева от своего врага. Данное действие выполняется 2fr 1 способами. Применяя правило произведения, получаем: |Дц П... nAJ = 4п(2п — к — 1)!2*-1. Данная формула годится и для случая к = 1. Учитывая теперь, что к пар из п можно выбрать способами, получаем ответ. 701. Поставим в ряд п — к нулей. Теперь для расстановки к единиц имеется п — к + 1 возможных позиций. 702. Справедливость доказываемого утверждения для п = 2, 3 легко проверяется. Зафиксируем некоторую точку и разобьем наборы точек, сре- ди которых нет соседних, на два класса в зависимости от того, попадает в набор данная точка или нет. Очевидно, что классы содержат соответ- ственно f(n — 3, к — 1) и/(п—1,/г) наборов. Поэтому 9(n, А) = fin - 3, к - 1) + fin - 1, t) = Ck„:l., + с ‘ , По известному комбинаторному тождеству С^_к = откуда име- ем: AL, = АЗЛ- Стало быть, д(п, к) = С„‘Щ + П = 703. 2n!n=„(-l)^C‘„.t(n - ft)! Решение. Разместим сначала жён. Это можно сделать 2п! способами (сна- чала определяем, на чётных или нечётных местах будут сидеть жёны, а затем размещаем п жен по п предназначенным им местам). Идя по часовой стрелке, пронумеруем жён числами от 1 до п; каждому мужу присвоим такой же номер, как у его жены. Идя по часовой стрел- ке, начиная с 1-й жены, пронумеруем свободные места числами от 1 до п. Теперь нам предстоит разместить мужей по свободным местам так, что- бы 1-й муж не занимал тг-го или 1-го места, 2-й муж — 1-го или 2-го,..., п-й муж — п — 1-го или тг-го. Обозначим через Г' событие: г-й муж си- дит на i-м месте; через Р- — событие: i-u муж сидит на i — 1-м месте (г = 2,..., п), через Р{ -событие: 1-й муж сидит на п-м месте. В цикли- ческой последовательности Р', Pi, Р^, Р?,..., Р'п, Рп- Р{ любые два соседних события нс могут осуществляться одновременно, а любые два нссоссдних — могут. Пусть А — множество всевозможных расположений мужей, а At — мно- жество расположений мужей, при которых г-й муж сидит рядом со своей
155 женой, Ai = А \ Ai (i = 1,..., п). Тогда число расположений мужей (при фиксированном расположении жён) есть мощность множества Л1 П ...П Ап. Для того, чтобы иметь возможность применить формулу включения- исключения, найдём Ий П ... П Aik | (1 < ii < г2 < • • • < 4 < п). Мужья из некоторых к пар могут сидеть одновременно рядом со своими жёнами, если события, отвечающие этим мужьям, не противоречат друг другу. Число способов разместить к мужей есть число способов выбрать из циклической последовательности Р{, Р], Р£, ^2, • • •, Р'т Рп, Р{ к попарно нссоссдних элементов, т.е. д(2п, к) — 2^k^2n-k (согласно 2-й лемме Кап- ланского). Оставшиеся п — к мужей на свободных местах размещаются (п — к)1 способами. Таким образом, Ий О ... О Aik | = оРкк(-'2п-к(п ~ Применяя формулу включения-исключения, получим: IX п... nXI = - *)! 704. п > р + q — 1. 711. Индукция по п. При п = 1 утверждение очевидно. Пусть п = к + 1. Зафиксируем некоторую вершину и. Среди (к + 1)а*. +1 рёбер, инцидент- ных и, найдётся + 1 рёбер одного цвета (скажем, синего). Если из + 1 концов этих рёбер, отличных от и, по крайней мере две (например, v и ш) соединены синим ребром, то получаем синий треугольник uvw. В против- ном случае в подграфе, порожденном щ + 1 вершинами, синий цвет отсут- ствует, то есть рёбра покрашены в к цветов. По предположению индукции в этом подграфе найдется одноцветный треугольник. Положив Ьп = п п ' получим = 1 + р и Ьп = Ьп_! + откуда = 12 F и ап = п1. J2 F- Для fr=0 ' k=Q доказательства того, что ап — [п!е], с помощью оценки остаточного члена формулы Маклорена для функции ех при .г = 1 проверьте выполнение неравенства к—О А;—0 717. Если выпуклая оболочка точек Л, В, С, О, Е представляет собой пяти- или четырёхугольник, то утверждение очевидно. Пусть выпуклой оболочкой является треугольник АВС. Тогда точки D и Е находятся вну- три него. Прямая DE не может пересекать все стороны треугольника. Ес- ли она нс пересекает, например, отрезок ЛВ, то точки A,D,E:B будут вершинами искомого выпуклого четырёхугольника.
156 Ответы. Указания. Решения 718. Выпуклая оболочка п точек представляет собой многоугольник. Если число его вершин меньше п, то какая-то из заданных точек (скажем, £)) лежит внутри указанного многоугольника. Всевозможные диагонали многоугольника, проведенные из некоторой его вершины, делят выпуклую оболочку на треугольники, одному из них (например, АВС) принадлежит точка D. Точки А, В, С, D нс удовлетворяют условию задачи. Полученное противо- речие доказывает утверждение задачи. 719. Пусть k = R(n, 5, 4); множество а составляют четвёрки точек, вы- пуклые оболочки которых — четырёхугольники, множество 3 тогда состо- ит из четвёрок точек, чьи выпуклые оболочки — треугольники. Согласно теореме Рамсея 1) существует п точек (из заданных к) таких, что любые четыре из них являются вершинами выпуклого четырёхугольника, либо 2) найдутся такие пять точек, что никакие четыре из них не могут быть вершинами выпуклого четырехугольника. Задача 717 показывает, что второй случай осуществиться не может, значит, имеет место первая ситуация. Осталось воспользоваться результа- том задачи 718. 720. В |9| выведена формула PG(.rj, ж2, • • •) = У12"=14у(М‘ В полу' ченной сумме нижний индекс при х принимает значения всех делителей числа п. В качестве упомянутого индекса число d встретится столько раз, сколько есть чисел j от 1 до п, для которых (и, у) = n/d. Такие числа имеют вид j = kn/d, где число к взаимно просто с d и не превосходит d. Значит, d будет в качестве нижнего индекса ровно </;(d) раз. 721. Применив теорему Пойа, выражение для циклового индекса из предыдущей задачи и полиномиальную формулу, получим: EX---) = -ЕХ) 12 г ki T...+km—n/d
157 Нас интересует коэффициент при w^.-.w^1. Указанное произведение возникает лишь при d, делящем одновременно щ,... ,пт. Дальнейшее просто. 722. Если п чётно, то Pg = xnJdtp{d') + ^х\х^2Х + тугг^2). Если п нечётно, то PG = + пхАх22 ). 724. Пусть С — граф, дополнительный к G. Расстояния в G и G' будем обозначать соответственно d и d'. Докажем, что если диаметр G нс меньше 3, то диаметр G' нс больше 3. Пусть d(u, v) > 3. Тогда ни одна вершина х графа G не может быть одновременно смежна с и и ц, так как в этом случае d(u,v) < d(u,x) + d(x,v) — 2. Если вершины х и у не смежны с одной и той же вершиной (скажем, и), то d'(x, у) < d'(x, u) + d'(u, у) = 2. Если же х и у не смежны с разными вершинами (например, х с и, а у с г»), то, поскольку и и v также несмежны, имеем d'(x, у) < d'(x, u)+d'(u, v)+d'(v, у) = 3. Итак, расстояние между двумя произвольными вершинами графа G' не превосходит 3. Для самодополнитсльного графа G справедливо: d(G) = d(G'). Пре- дыдущее рассуждение показывает, что d(G} < 3. С другой стороны, если d(G) = 1, то G — полный граф и нс может быть изоморфен своему до- полнению (пустому графу). Поэтому 2 < d(G) < 3. Осталось придумать примеры самодополнительных графов диаметров 2 и 3. 725. 10. 728. Обозначим вершины графа числами 1,2,..., 2п. Рассмотрим цепи к —± к + 1 к — 1 —А; Ч- 2 —> А: — 2 —> А + 3 —>...—> к — гг -Е 1 —> к + п (к = 1,..., п), (к каждому неположительному числу добавляем 2п). Каж- дая такая цепь является гамильтоновой, поскольку числа к — п + 1,к — п + 2,... ,к + п — 1,к + п образуют полную систему вычетов по модулю 2п. Так как в А-й цепи сумма номеров соседних вершин сравнима с 2к или 2А + 1 по модулю 2п, данные цепи нс имеют общих рёбер. Осталось заметить, что общее количество рёбер в указанных цепях равно и(2п — 1) и совпадает с количеством рёбер графа Ку,,.- 729. Гамильтонову цепь к —> ... —> к -I- п из решения предыдущей задачи дополним до гамильтонова цикла добавлением цепи к + п —> 2п + 1 к. 730. Используйте результаты задач 655, 20 и 448. 731. Докажите, что в графе есть полный подграф порядка п + 1.
158 Ответы. Указания. Решения 732. 1) Связность графа очевидна. Осталось доказать, что степень каждой вершины чётна. Для произвольной вершины v рассмотрим (мак- симальный по количеству рёбер) подграф G, образованный вершинами, смежными с v. Степень каждой вершины этого подграфа нечетна, поэто- му число его вершин (суть степень v) чётно. 2) Пусть V — множество вершин графа; п — число вершин; Г(и) — мно- жество вершин, смежных с е; /Дм, и) — число общих вершин, смежных с и и V. Тогда при вычислении суммы S — 52uev\{i>] каждая вершина w, смежная с и, учитывается p(w) — 1 раз. В силу предыдущего пункта за- дачи |Г(г>)| — чётное число, a (p(w) — 1) — нечётное для любого w. Сумма чётного числа нечётных слагаемых S = 22wer(J)j(p(w) — 1) чётна. С другой стороны, S есть сумма (п — 1) нечётных слагаемых p,(u,v). Стало быть, число п нечетно. 733. Если к = 2, то п — нечётное число > 3; если к > 3, то п = к -I- 1. Указание. В случае к = 2, как следует из результата предыдущей задачи, п — нечётно (ясно, что п > 1). Примером графа с 2t + 1 вершинами, удо- влетворяющего условиям задачи, является объединение I треугольников с общей вершиной. Докажите, что 1°) у любых двух вершин ровно к — 1 общих смежных вершин; 2°) в графе найдётся полный подграф Kk+i- Пусть к > 3 и Y = {?/], у2,..., Ук+А} — множество вершин полного под- графа, a v Y. У вершин yi и у2 в силу 1° все общие смежные вершины входят в У, поэтому общая смежная вершина yi,y2 и v принадлежит У. Значит, v имеет смежную вершину из У. Если у v есть две смежные верши- ны из У, то у этих двух вершин будет не менее к общих смежных вершин (это оставшиеся к — 1 вершин из У и вершина v), что противоречит 1°. Та- ким образом, у v ровно одна смежная вершина из У — пусть это вершина х. Возьмем вершину у из У, отличную от х. У вершин v,x и у есть общая смежная вершина, и она принадлежит Y (так как все общие смежные вер- шины х и у из У). Но тогда у v сеть две общие смежные вершины из У. Полученное противоречие говорит о том, что множество У совпадает с множеством вершин графа, то есть граф — полный, при этом п = к + 1. 734. 16. 735. 1), 2) Возьмем произвольную вершину v. Пусть п — число вершин графа, к = p(v). Для любых двух вершин ъ\ и Vj, смежных с v, найдёт- ся общая смежная вершина Wij, не смежная с v. Причем все вершины wij, — 1, ,&, i ф j разные (иначе некоторая вершина, нс смсж-
159 ная с v, имела бы с ней более двух общих смежных вершин). Вершинами v, V],..., Vk, W12, • • •, wjjt,..., Wk-1,к исчерпывается все множество вершин графа. Значит, п = l+fc-l-*^"1). Это уравнение имеет (при фиксированном п) единственное положительное решение относительно к. Таким образом, степень каждой вершины определяется однозначно порядком графа. По- этому граф является регулярным. 3) Из условия задачи вытекает матричное равенство Л2 + 2Л + (2 — Л)7 = 27, (1) где 1 — единичная матрица размера п х п, J — матрица того же разме- ра, состоящая из одних единиц. Пусть х — собственный вектор матрицы Л, х' = (xi,..., хп). Умножив обе части матричного равенства (1) на х, получим п (А2 + 2Л + (2-А:))т = 2^^1 (2), г= 1 где j — вектор, все координаты которого единицы. Поскольку А / к, век- тор х нс коллинеарен вектору j (который как раз отвечает собственному значению к). Значит, равенство (2) возможно лишь при А2 + 2А + (2 - к) = 2 = 0. ?:-i 4) Матрица Л как матрица смежности связного графа удовлетворяет условиям теоремы Перрона-Фробениуса, согласно которой наибольшее по модулю собственное значение матрицы Л положительно и имеет алгебра- ическую кратность 1 (см. |33|). Таким образом, собственное значение к имеет алгебраическую кратность 1. Для определения кратностей Гц2 соб- ственных значений Ацг = —1 ± \/к — 1 нужно решить систему линейных уравнений Г1 4- г2 = п - 1, А1 Г] + А27’2 + к = 0. (Последнее соотношение есть следствие того факта, что сумма собствен- ных значений матрицы, взятых с учетом их алгебраической кратности, равна сумме се диагональных элементов). 5) вытекает из 4). 6) С4, граф Клебша (определение в задаче 443) и граф Гевиртца (|29|). Замечание. Рассмотренные в данной задаче графы относятся к так на- зываемым сильно регулярным графам (см. [29|, [61], |52|).
Литература |1| Башмаков М.И., Беккер Б.М., Гольховой В.М. Задачи по математи- ке. Алгебра и анализ. — М.: Наука, 1982. - 192 с. |2| Виленкин Н.Я. Комбинаторика. — М.: Наука, 1969. - 328 с. |3| Виноградов И.М. Основы теории чисел. — М.: Наука, 1981. - 176 с. |4| Гаврилов Г.П., Сапоженко А.А. Сборник задач по дискретной мате- матике. — М.: Наука, 1977. - 368 с. |5| Комбинаторный анализ. Задачи и упражнения /Под ред. К.А. Рыб- никова. — М.: Наука, 1982. - 365 с. |6| Куликов Л.Я., Москаленко А.И., Фомин А.А. Сборник задач по алге- бре и теории чисел. — М.: Просвещение, 1993. — 288 с. |7] Лекции по теории графов /В.А. Емеличев, О.И. Мельников, В.И. Сарванов и др. — М.: Наука, 1990. - 384 с. |8| Уилсон Р. Введение в теорию графов. — М.: Мир, 1977. — 208 с. |9| Эвнин А.Ю. Дискретная математика: Конспект лекций. — Челя- бинск: ЮУрГУ, 1998. - 176 с. ДОПОЛНИТЕЛЬНАЯ [10] Айерлэнд К., Роузен М. Классическое введение в современную тео- рию чисел. — М.: Мир, 1987. - 415 с. [11] Басакер Р., Саати Т. Теория графов и сетей. — М.: Наука, 1974. - 366 с.
161 |12] Березина Л.Ю. Графы и их применение. — М.: Просвещение, 1979. - 144 с. |13] Вагутсн В.Н. Числа многочлены, последовательности //Квант. - 1973. - №2. - С.27-34. |14] Васильев Н., Коганов Л. Разбиения, ГС-перестановки и деревья //Квант. - 1997. - №6. |15] Венгерские математические олимпиады. — М.: Мир, 1976. - 543 с. |16] Верещагин Н.В., Шень А. Логические формулы и схемы //Математи- ческое просвещение. - 2000. - Сер. 3. - Вып. 4. - С.53-80. |17] Верещагин Н.К., Шень А. Вычислимые функции. — М.: МЦНМО, 1999, - 176 с. Интернет: ftp://ftp.mccme.ru/users/shen/logic/comput. |18| Виленкин Н.Я. Популярная комбинаторика. — М.: Наука, 1975. - 208 с. |19] Воробьев Н.Н. Числа Фибоначчи. — М.: Наука, 1992. - 190 с. |20] Гаврилов Г.П., Сапоженко А.А. Задачи и упражнения по курсу дис- кретной математики. — М.: Наука, 1992. - 405 с. |21] Галочкин А.И., Нестеренко Ю.В., Шидловский А.Б. Введение в тео- рию чисел,. — М.: Изд-во МГУ, 1995. - 160 с. |22] Гашков С.Б, Чубариков В.Н. Арифметика. Алгоритмы. Сложность вычислений. — М.: Высшая школа, 2000. - 320 с. |23] Генкин С.А., Итснбсрг И.В., Фомин Д.В. Ленинградские математи- ческие кружки. — Киров: АСА, 1994. - 272 с. |24] Гиндикин С.Г. Алгебра логики в задачах. — М.: Наука, 1972. — 288 с. |25] Грэхем Р., Кнут Д., Паташник О. Конкретная математика. — М.: Мир, 1998. - 703 с. |26] Олимпиады ЮУрГУ для абитуриентов. Математика. Задачи и ре- шения /В.Л. Дильман, В.И. Заляпин В.И., Ю.Г. Малиновский и др. и др. — Челябинск: Изд. дом Обухова, 2000. - 100 с.
162 Литература |27| Задачник ’’Кванта”: Математика. — В трех частях/ Под рсд. Н.Б. Васильева. — М.: Бюро Квантум, 1997 (Прил. к журналу "Квант’: К°1, №3, №5). |28| Избранные задачи из журнала ’’American Mathematical Monthly”. — М.: Мир, 1977. - 598 с. [29| Камерон П., ван Линт Дж. Теория графов, теория кодирования и блок-схемы. — М.: Наука, 1980. - 139 с. |30| Кнут Д. Искусство программирования для ЭВМ. Т.1. Основные ал- горитмы. — М.: Мир, 1976. - 736 с. |31| Кострикин А.И. Введение в алгебру. Основы алгебры. — М.: Наука, 1994. - 318 с. |32| Кофман А. Введение в прикладную комбинаторику. — М.: Наука, 1975. - 480 с. |33| Ланкастер П. Теория матриц. — М.: Наука, 1978. - 269 с. |34| Липский В. Комбинаторика для программистов. — М.: Мир, 1988. - 213 с. |35| Нефедов В.Н., Осипова В.А. Курс дискретной математики. — М.: Изд-во МАИ, 1992. - 264 с. [36| Новиков П.С. Элементы математической логики. — М.: Наука, 1973. - 400 с. [37| Новиков Ф.А. Дискретная математика для. программистов. — СПб.: Питер, 2000. - 304 с. [38| Мендельсон Э. Введение в математическую логику.— М.: Наука, 1976. - 320 с. [39| Оро О. Приглашение в теорию чисел. — М.: Наука, 1980. - 128 с. |40| Оре О. Теория графов. — М.: Наука, 1980. - 392 с. |41| Полна Г., Сеге Г. Задачи и теоремы из анализа. — М.: Наука, 1978. - Т.2. - 432 с.
163 42 43 44 45 46 47 48 49 50 51 52 53 54 55 56 Рейнгольд Э., Нивергсльт Ю., Део Н. Комбинаторные алгоритмы. Теория и практика. — М.: Мир, 1980. - 478 с. Рудаков А.Н. Числа Фибоначчи и простота числа 2127 — 1 //Матема- тическое просвещение. - 2000. Сер. 3. - Вып. 4. - С.127-139. Саати Т. Целочисленные методы оптимизации и связанные с ними экстремальные проблемы. — М.: Мир, 1973. - 304 с. Ссндсров В., Спивак А. Малая теорема Ферма //Квант. - 2000. - >1. - С.9-16, 37; >3. - С. 11-17; №4. - С. 15-18. Сборник задач по алгебре /Под род. А. И. Кострики на. — М.: Факто- риал, 1995. - 452 с. Тараканов В.Е. Комбинаторные задачи и (0,1 )-матрицы. — М.: На- ука, 1985. - 192 с. Трост Э. Простые числа. — М.: ГИФМЛ, 1959. - 136 с. Уильямс X. Проверка, чисел на простоту с помощью вычислитель- ных машин //Кибернетический сборник. - Вып. 23. — М.: Мир, 1986. Харари Ф. Теория графов. — М.: Мир, 1973. - 302 с. Холл М. Комбинаторика. — М.: Мир, 1970. - 424 с. Цветкович Д., Дуб М., Закс X. Спектры графов. Теория и применение. - Киев: Наукова думка, 1984. - 383 с. Школа в ’’Кванте’": Арифметика и алгебра /Под ред. А.А. Егорова. - М.: Бюро Квантум, 1994 (Прил. к журналу ’’Квант”). - 128 с. Эвнин А.Ю. Девятнадцать доказательств теоремы Евклида, //Квант. - 2001. - №1. - С.35-38. Интернет: http://www.vivovoco.rsl.ru/quantum/2001.01/matkr_l_01.pdf. Эвнин А.Ю. Две заметки по комбинаторике //Математическое об- разование. - 2000. - К°3(14). - С.27-34. Эвнин А.Ю. Сверхстепени и их разности //Математическое образо- вание. - 2001. - №1(16). - С.68-73.
164 Литература |57| Эвнин А.Ю. Букет окрестностей одной задачи, или О методах сум- мирования //Математика в школе. - 2000. - №8. - С.64-67. [58| Эвнин А.Ю. Решение задач на возвратные последовательности //Математика в школе. - 2001. - №7. - С.69-70. |59| Эвнин А.Ю. Возвратные последовательности в олимпиадных зада- чах //Математика (прил. к газете ’’Первое сентября”). - 1999. - Я«36. -С.15-16. |60| Эвнин А.Ю. Элементарная теория чисел: Сборник олимпиадных за- дач. — Челябинск: ЧГТУ, 1996. - 76 с. [61| Юбо К. Сильно регулярные графы //Кибернетический сборник. — М.: Мир, 1987. - Вып. 24. |62| Яглом И.М. Булева структура и ее модели. — М.: Сов. радио, 1980. - 193 с. |63| http://www.zaba.ru/cgi-bin/tasks.cgi?tour=national.putnam.